Sei sulla pagina 1di 213

2011

Secondary School Preliminary


Examination Papers
Secondary 4 Express

Chemistry (Part 2)
Paper 1 & 2
1
2
3
4
5
6
7
8
9
10
11

Ngee Ann Secondary School


North Vista Secondary School
Outram Secondary School
Pasir Ris Secondary School
Peihwa Secondary School
Queensway Secondary School
Regent Secondary School
Springfield Secondary School
St. Anthonys Canossian Secondary School
Unity Secondary School
West Wood Secondary School

SA2
SA2
SA2
SA2
SA2
SA2
SA2
SA2
SA2
SA2
SA2

Name: .....

. . . .. . . . . . . . . . . . . . . . . . . . . . Register no: ..
'" Class:

NGEEANNSECONDARYSCHOOL
Building Character H H H;; ;;Expanding M inds;;;;;;;; ;;Shaping Lives

Preliminary Examination 2 2011


Secondary Four Express

Pure Chemistry
5072/1
Duration:
1h

Ngee Ann Secondary School Ngee Ann Secondary School Ngee Ann Secondary School Ngee Ann Secondary School
Ngee Ann Secondary School Ngee Ann Secondary School Ngee Ann Secondary School Ngee Ann Secondary School
Ngee Ann Secondary School Ngee Ann Secondary School Ngee Ann Secondary School Ngee Ann Secondary School
Ngee Ann Secondary School Ngee Ann Secondary School Ngee Ann Secondary School Ngee Ann Secondary School
Ngee Ann Secondary SchOol Ngee Ann Secondary SchOOI Ngee Ann Secondary School Ngee Ann Secondaty School

Monday

19 September 2011
Additional Materials:
OAS
Instructions to Candidates:

1. Write your name, register number and class at the top of this page and also on the Optical
Answer Sheet.

2. There are forty questions on this paper. Answer ALL questions.


3. For each question. there are four possible ~nswers A, B, C and D. Choose the one you
consider correct and record your choice in soft pencil on the separate OAS.
4. A copy of the Data Sheet is printed on page 16.
5. A copy of the Periodic Table is printed on page 17.

Signature

Date

After checking of answer scripts


Checked by
Student
Parent/ Guardian
DO NOT TURN THIS PAGE OVER UNTIL YOU ARE TOLD TO DO SO.
This Question Paper Consists of .!I Pages (including this cover page)

What can be deduced about two gases that have the same molecular

They have the same rate of diffusion at room temperature and

They have the same number of atoms in one molecule.

They have the same boiling point.

They have the same solubility in water at room temperature.

mass?

pressure.

widely separated,

sugar molecules

moving at random

close together,

water molecules

- -

Which of the following correctly describes the particles in a dilute sugar

moving at random

not moving

widely separated,

close together,

widely separated,

close together,

moving at random

widely separated,

not moving

moving at random

close together,

moving at random

solution at room temperature?

I D

moving at random

The table shows the melting and boiling points of four pure substances.

58

-35

boiling points/ C

At room temperature, which substance 1s a liquid and rapidly

evaporates if left exposed to air?

melting points/ C

225

-7

280

-100

-6

44

;j

Which technique could be used to obtain the elements X and Y from

crystallization

the ionic compound XY?


A
distillation

filtration

electrolysis

B
C
D

3-,

A gas X 1s insoluble in water and less dense than air. An impure supply

' 2-1
<--!<'J '

water

1, 3, 2,5

1,2, 3,5

1, 2, 3, 4

sodium

rubidium

chlorine

water

I s-~

of X contains water vapour and a water-soluble impurity.


gas
supply
olX

~
drying
agent

In which order should the pieces of apparatus shown be joined

c
1, 3, 2,4

together to collect a pure, dry sample of X?

Which of the following ions has the same number of electrons as a

xenon

krypton atom, atomic number 36?

State at room

temperature

conductivity at room

Electrical

aqueous solution

conductivity of

Electrical

Which substance in the table is an ionic compound?

temperature

solid

liquid

liquid

good

poor

good

good

good

poor

insoluble

~-

poor

solid

Substance

Fe2(N03)3

Fe(N03)3

fe3(N03)2

fe3N03

What is the formula or iron (Ill) nitrate?

key
o electrons of Y atom
x electrons ot Z atom

The diagram shows the arrangement of electrons in the outer shells of

the atoms in the compound YZ2 .

sulfur

oxygen

carbon

calcium

chlorine

hydrogen

sulfur

fluorine

Which pair of elements could be Y and Z?

10

11

12

13

16 g

32 g

"

64g

70 g

What is the mass of oxygen contained in 72 g of pure water?


A

solid sodium chloride

sodium metal

pure ethenoic acid

liquid <>xygen

Which of the following is the best conductor of electncity?

Solabon X turns acidified potassium dichromate (VI) from orange to

a reducing agent

an oxidizing agent

an ammonium salt

an alkali

green. What must solution X cootain?

14

15

a light blue solid and a colourless liquid

a bfue liquid only

a black solid and a colourless liquid

S02

-?

S03

-?

H2S04

The manufacture of sulfunc acid by the Contact process can be

-?

represent.ad as foflows .

in

6
5

1
0

the bonds breaking is greater than that of

2HC/ exothermic?

Which ciagram shows the oxidation state (ox1det1on number) of sulfur

it

at each stage of the process?

state

~00

j
~~

Energy involved

Why is the reaction H2 + C/2 A

bonds forming.

Energy involved in the bonds forming 1s greater than that of the

More bonds are brokenthan are formed .

More bonds are formed than are broken.

bond breaking.

peroxide to g ive oxygen and water. In a laberatory e)Qi>eriment,

Copper (II) oxide catalyses the decomposition of aqueous hydrogen

aqueous hydrogen peroX1de was shaken with copper (II) oxide in a


test-tube. What did the test-tube contain when the reaction was

a pink-brown solid and a colourless -liquid

completed?

16

17

magnesium hydroxide

T-

The rate of reaction always increases with temperature.

Which of the following does not react with dilute sulfuric acid?

magnesium metal

i8

carbon dioxide is given off. The graph shows how the volume of carbon

Ethanol is produced by the fermentation of sugar. During the reaction

dioxide produced per minute varies with temperature.

---,-----60

The rate of reaction reaches a maximum at about 40C.

20

is Q found?

iB

!C

-1-T-1

zinc carbonate

sodium carbonate

magnesium carbonate

copper (II) carbonate

alkali. What is X?

--t-

limewater. The residue reacts with dilute acid and also with aqueous

When heated, solid X gives off a gas which forms white precipitate in

compounds are coloured. In which position of the Periodic Table shown

states (oxidation numbers) of +2 and +3 in its compounds. These

Element Q has a melting point greater than 1000 C. It has oxidation

magnesium nitrate

50

The reaction is slowest at 0C.

calcium carbonate

1~

(\

40

/ \
~
tomperaturel"C

30

magnesium oxide

---20

vclumect
carboo dioxide
proouced per
minute

l
!---"
."--- --r
10

The reaction takes a long time to begin.

Using the graph, decide which statement is correct?

Under suitable conditions, hydrochloric acid reacts with each of the

lron(ll) hydroxide

fo llowing substances. Which reaction gives only a colourless solution?

potassium hydroxide
. silver nitrate

c
D

21

22

2'3

24

The diagram shows some of the stages 1n the manufacture of


2~ r

Aluminium is extracted from purified bauxite by electrolysis but iron is

Reduction with coke 1s cheaper than electrolysis.

Iron is less reactive than aluminium.

Haematite needs to be purified but bauxite does not.

extracted from haematite by reduction with coke.

ammonium sulfate. From which connecting pipe would a major leak

'\
ammonium

Why is iron not extracted by electrolysis?

' ammonia -

-.

I vessel I

~
PIPE A

Reduction with coke gives a purer product that electrolysis.

Aluminium has a lower density than iron.

Aluminium has only one valency.

of aluminium?

sulphate

most likely increase the pH value of rain?

Ollrogen
and
hyorogen
PIPE B
J L
~action L _
~r~e:sol
--

__

Old steel drums corrode quickly in a damp atmosphere but aluminium

Fluorine is the first element in Group VII of the Periodic Table.

Aluminium is above iron in the activity series.

c
26

Which statement is not true of fluorine?

Aluminium 1s protected by its oxide layer.

HSOJ
- __ ----~ction
reaction SO3 2

PIPli
S02+0z vessel
1 vesse1 r

~.JPIPE D

Fluorine exists in diatomic molecules.

cans do not. Which of the following statements explain this behaviour

Fluonne is less reactive than chlorine.

Francium is a metal that is more reactive than aluminium.

F;uorine forms negative ions.

27

Silver fluoride will be sensitive to light.

c
D

W hich reaction would produce francium?

heating francium carbonate

The ion XH/ can be represented by the 'dot and cross' diagram

IV

electrolyzing aqueous francium chloride

H-+

x~~j

xx

heating francium oxide with carbon

e1ectroiyzing molten francium chloride

Ill

shown.

To which Group in the Periodic Table does X belong?


A

Ir: the Periodic Table, how many periods include the elements of

atomic numbers 1 - 18?


A

28

11
Zinc reacts with hydrochloric acid according to the equation given
below:
Zn (s) + 2HCI (aq)-+ ZnCb (aq) + 1i2 (g)

Time taken (s)

Three experiments were carried out using exeess of zinc and different
concentrations of dilute hydrochloric acid at the same temperature. The
graph shows the results.

Total volume of
hydrogen evolved
(cm3)

Solution II

y
y

z
x

Solution Ill

I
I
I

x
x

100cm3 of 0.5 mol/dm 3 hydrochloric acid


50cm 3 of 1.0 mol/dm 3 hydrochloric acid
50cm 3 of 2 .0 mol/dm 3 hydrochloric acid

The solutions used were


Solution I
Solution II
Solution Ill

Solution I

Which cuFVe corresponds to the appropriate solution?

IA
'B
I c;
~

29

30

31

32

33

12

limestone

haematite

quartz

mica

Which naturally occurring substance is mainly silicon dioxide?

Which ions are d ischarged when concentrated aqueous sodium

hydrogen and sodium ions

chloride and hydrogen ions

chloride and hydroxide ions

ch loride is electrolysed?

hydroxide and sodium ions

e
0

When calcium carbonate is heated, the following reaction takes place.

CaC0 3 (s) 7 Cao (s) + C02 (g)

22dm3

24 dm 3

NH4N03

(NH 4 )2SO.

(NH4)3P04

carbon monoxide

methane

ethane

44 dm3

What volume of carbon dioxide at r.t.p. would be produced, if 50 g of

12dm3

ca lcium carbonate is heated?


A

NaN03

Which fertilizer provides the most nitrogen per mole?

ethanol

Which compound. on combustion, never forms soot?

34

35

36

Methane .s a gas which can be used as a fuel. What property is

It burns exothermically.

essential for this use?


A

It is odourless.

It is a gas.

C
It has a low boiling point

Which natural resource is being depleted by the manufacture of

air

plastics?
A

metal ore

fossI fuels

C
water

6
D

An organic compound X reacts with sodium hydroxide to give a

propanol

propanoic acid

propane

ethanol

compound with formula C3Hs0 2Na What is compound X?

37

38

H
H-C -- H

and

H-C --C- H

Cl

H -C-C- H

H - C C -- Ci

I
H

"

Cl

C= C

H--- C

c1.,

Cl

- C - Cl

and

C -C/

and

H
H-C -

Cl

"

/
H'\..,.
C= C
/

and

Which pair of compounds are isomers of each other?

Ci

CH2Cl2 +

+n

CHC'3 + HCI

CH2

C3H10H

C2H.Br2

Cir~

Which of these equations does not represent ar addition reaction?


A

C2H4 +

C3H6 + H20

nC2 H4 ~ +cH 2

Brr~

c
0

39

4.Q

l::>

CHa

CHa

CH-, CH2-CH-

CH2-CH--CH2 -

A polymer has the structure shown.


CH3

C2Hs

C2H4.
D

c
C3Ha

C3Hs

What is the molecular formula of the monomer?

Which polymer has the empirical formula CH?

0
H\ fHb-bH\ fHb-bHj
HH\ fHb-b-1
f !b-b-}
! Jn ! c~~/n ! cJH5~ ! c&Ls n

10

DATA SHEET

Colours of some common metal hydroxides

White
White
Light blue
Green
Red-brown
White
White

l. ";;:::::,;u d id .'"'. .,.:,-\1;:;; ~;;~,. 1 .. _,,,,,,;, "


;"';" ~;:QY .. rox ..e ~:.;:: .:::;~:;:~f\:t:~!? .O.l!~: .''".:,;;;.
Aluminium hydroxide
Calcium hydroxide
Copper(ll) hydroxide
lron(ll) hydroxide
lron(lll) hydroxide

Zinc hydroxide

Lh_E!ad(l_l)~Y~!()ldde

Name: ..

Register no:

Class:

NGEE ANN SECONDARY SCHOOL


Building Character H H H H #Expanding Minds H H H H #Shaping Uves

Preliminary Examination 2 2011


Secondary Four Express

Pure Chemistry
5072 / 2
Duration:
1h45 min

Date

Ngee Ann Secondary School Ngee Ann Secoodary School Ngee Ann Secondary SchOol Ngee Ann Secondaty School
Ngee Ann Secondaty School Ngee Ann Secondary School Ngee Ann Secondary School Ngee Ann Secondaty School
Ngee Ann Secondary School Ngee Ann Se<:ondaly School Ngee Ann Seccndary School Ngee Ann Serondary School
Ngee Ann Secondary School Ngee Ann Secondary School Ngee Ann Secondary School Ngee Ann Secondary School
Ngee Ann Secondary SchOol Ngee Ann Secondary School Ngee Ann Secondary School Ngee Ann Secondary School

Monday
12 Septeriber 2011

Instructions to Candidates:
Write your name. register number and class at the top of this page.
Section A : Answer ALL questions in the spaces provided.
Section B : Answer 3 questions in the spaces provided. The last question 1s in the form of
either/or
The number of marks is given in brackets [ ) at the end of each question or part question.
A copy of the Data Sheet is printed on page 25.
A copy of the Periodic Table is printed on page 26.

Signature

After checking of answer scripts


Checked by
Student
ParenU Guardian

DO NOT TURN THIS PAGE OVER UNTIL YOU ARE TOLD TO 00 SO.
This Question Paper Consists of 26 Pages (including this cover page)

A1

Secti on A

The maximum mark for this section is 50.

Answer all t he questions in this section in the spaces provided.

Silver carbonate

Zinc chloride

Copper (II) chloride

A luminium oxide

Bromine

Silicon dioxide

Calcium chloride

From the list of substances given below:

Helium

hydroxide solution.

2011 NAS Chemistry Prelim Paper 2

an element that is a liquid at room temperat ure.

a compound that reacts w ith both acids and bases.

. .. ' "' " 't.-

a compound which produces an alkaline gas when warmed with sodium

a salt that is prepared by precipitation.

a compound that is coloured.

Ammonium su lfate

Choose
(a)

(b)

(c)

(d)

(e)

Setter- PG

[1)

[1)

[1 I

[1 I

[1)

A2

A3

10 electrons and 8 neutrons.

state

in liquid

insoluble

water

dissolved in

[11

electro nic structure

The electronic structures of elements A to D are shown in the table below.

2,8, 18, 18, 8, 1


2,8, 18, 18, 9

A4

2, 8, 18, 27

(1]

2,8, 18, 18, 6, 3

in solid

poor

good

simple molecular solid;

................................. (1]

................................... [1]

Setter: PG

for a caesium atom?

much more reactive than a lithium atom.

2011 NAS Chemistry Prelim Paper 2

[2]

to water. Explain, in terms of electronic structure, why a caesium atom is

the surface of the water. A small piece of caesium explodes when added

When a small piece of lithium is added to cold water, it fizzes around on

[1 I

Which of these elements could possibly represent the electronic structure

(a)

poor

state

poor

good

(1]

......... 1 ................ ............. ... .... . .. ... . ...... . . . . .

l> ............................................. .
Predict the kind of bonding formed between A and carbon.

........................ .

poor

good

insoluble

soluble ionic solid;

2011 NAS Chemistry Prehm Paper 2

(b)

To which group of the Periodic Table does element A belong to?

Is particle A a positive or negative ion?

Particle A has 8 protons,

(a)

(b)

(c)

~-- .I .....

oc

poor

poor

good

Electrical conductivity

3 44

poor

good

insoluble

point/ .

-85
1382
good

poor

Bolling

The following table gives some information about several substances.

36
685
poor

point I

37
-114
614

2230

oc

-130
29
1610

Substances ] Melting

c
D
E

once, more than once or not at all.

(a)

pure liquid containing simple molecules

.................. (1]

Answer the following questions using one of the letters A to F. Each letter may be used

(b)

Using the list, write the letter of a substance which at 2nr. is a

(c)

Setter: PG

A~

s
Superglue' contains the following monomer:
H C := N

I I

C= C

H C - O-CH3

II
0

(1)

(1I

The monomer is rapidly polymerised by traces of bases on the surfaces of

group that is re.sponsible for producing this sweet smell.

2011 NAS Chemistry Prelim Paper 2

(1)

It was found that the glue formed has a sweet smell, name the functional

polymerised form of the glue.

Draw the full structural formu la to represent one repeat unit of the

What type of polymerisation takes place when the glue is in use?

objects to be stuck together, causing the glue to set solid.


(a)

{b)

(c)

Setter. PG

AS

by 50% from 1900 to the year 2050. Most of this increase is thought to be

It has been predicted that carbon dioxide levels in the atmosphere will increased

caused by the burning of fossil fuels in motor vehicles and power generating

Explain why the burning of fossil fuels produces carbon dioxide.

this so?

and nitrogen in the air.

Construct a balanced equation for the reaction between oxygen

an internal combustion engine.


(i)

.. ...................... .. ............

2011 NAS Chemistry Prelim Paper 2

(1)

Nitric oxide, NO, is formed when oxygen and nitrogen from the air react in

monoxide, CO.

fumes contain atmospheric pollutants such as nitric oxide, NO, and carbon

vehicles contribute to the poor quality of air in many cities. The exhaust

The exhaust fumes from the internal combustion engines of motor

[2]

Caibon 01oxide is sometimes considered an atmospheric pollutant. Why is

[1]

plants. An important effect of increased carbon dioxide levels is the 'greenhouse


effect'.

~)

(b)

(t)

Setter: PG

Sette1: PG

(ii)

(iii)

(iv)

What are the harmful effects of nitric oxide and carbon monoxide?

faster in the car engine. Explain why.

(2)

The rate of reaction belWeen oxygen and nitrogen in the air is often

(2]

Name another air pollutant and explain how it is released into the
air.

............. ........................ .. .... ...... ... .......... ................ [2]

201 1 MS Chemistry Prelim Paper 2

A7

combustion of butane in air.

Write a balanced chemical equation with state symbols. for the complete

1 mole of butane (C4H1o) on complete combustion liberates 2880 kJ of heat.


(a)

(2)

Calculate the amount of heat evolved from the complete combustion of 600

(2)

(2)

(b}

Using the given axes, draw a labeled energy profile diagram for the

cm 3 of butane, measured at room temperature and pressure.

(c)

Tim e

2011 NAS Chemistry Prelim Paper 2

complete combustion of 1 mole of butane in air.


Energy

Setter: PG

AB

(d)

Explain
in

terms of bond-breaking and bond-forming why heat is liberated

during the complete combustion of butane.

(l>J

$8.00

$4.00

Price per bottle

'I

10

Concentration

125

bottle (cm3 )

0.833

(mol/dm 3)

of ethanoic acid

250

vinegar in 1

Volume of

The prices of Brand X and Brand Yvinegar are giver. below:

["""'
I

Using your answer in (a), explain which brand of vinegar will be a better

(1 J

The following experiment was conducted by the Consumer Association of

(2)

(2]

--+

SO/'(aq) + 2r (aq} + 4 H-Caq)

Calculate the oxidation number of sulft.r n

so/ ?
(i)

Explain whether S02 is reduced or oxidized in the reactio n.

2011 NAS Chemistry Prelim Paper 2

(ii)

S02(aq) + l2(aq) + 2 H20(1)

aqueous iodine and the ionic equation is as shown below:

(a)

Setter: PG

(2]

[2]

:1 J

dioxide. The sulfur dioxide content of a wine can be found using its reaction witr

too little ar.d the wine reac ly goes bad, too Much and the wir.e tastes of sulfur

preservative. The amount cf sulfur dioxide added needs to be carefully calculated;

Wines often contains a small amour.I of sulfur dioxide that is added as a

buy.

AB

Singapore to compare the composition of two conmercial brands of vinegar.

25.0 cm3 of Brand X vinegar was titrated against 1.00 moVdm3 NaOH sclution,

CH 3 COOH + NaOH -+ CH3COONa + H20

2011 NAS Chemistry Prelim Paper 2

the percentage by mass of ethanoic acid, CH 3COOH in tre viregar.

Given that tr e density of Brand Xvinegar is 1.05 i;/cm3, calculate

Brand Xvinegar 'n mol/dm 3 .

Calculate the concentration of ethanoic acid, CH3COCH present in

using phenolphthalein as an indicator. The equation for neutralisation is as


shown below.

(11)

(I/

The volume of NaOH required for titration was found to be 26.25 cm 3


(a)

Setter: PG

11

Read the article below and answer all the questions that follow.
(a)

12

"The battery itself is simple, consisting of two electrodes - one positive, one

In the article mentioned above,

negative - immersed in a I/quid containing electrically charged particles, or ions. "

A18

Researc hers use ri ver w ater and salty ocean water to generate electricity.

.,-............_,,,, ..t -

Draw a labeled d iagram of the simple battery (simple cell).

In the school laboratory, a simple battery (simple cell) could also be setup.
(i)

Write two half equations for the setup you have drawn in (c) (i).

2011 NAS Chemistry Prellm Paper 2

(ii)

Setter: PG

(2)

(2]

(2)

11 I

ions'.

... .

Based on the article above, name all the ions present in the battery.

Give another name for the "liquid containing electrically charged particles, or

(b)

potential site for a power plant using such a battery. The battery itself is simple,

..... .......

(c)

Any place where freshwater that enters the sea, river mouths or estuaries, could be a

consisting of two electrodes - one positive, one negative - immersed in a liquid


containing electrically charged particles, or ions. In water, the ions are sodium and
chloride, the components of ordinary table salt.

Initially, the battery 1s filled with freshwater and a small electric current is applied to
charge it up. The freshwater is then drained and replaced with seawater. Because
seawater is salty, containing 60 to 100 times more ions than freshwater, it increases the

reap far more electricity than the amount used to charge the battery.

electrical potential, or voltage, between the two electrodes. That makes it possible to

"The voltage really depends on the concentration of the sodium and chloride ions you
have," said the researcher. "If you charge at low voltage in freshwater, then discharge at
high voltage in sea water, it means you gain energy. You get more energy than you put

2011 NAS Chemistry Prelim Paper 2

Step 4 , seawater 1s discharged and


replaced with river water, for the cycle to
begin anew.

Step 3, electricity is drawn from the battery


for use, draining the battery of its stored
energy. Ions return to electrodes.

Step 2, the fresh water is purged and


replaced with seawater. Note much higher
abundance of charged ions in the salt water.

In Step 1, a small electric current 1s applied


to charge the battery, pulling ions out of the
electrodes and into the water.

in. "Once the discharge is complete, the seawater is drained and replaced with

....

freshwater and the cycle can begin again.


~

'----- -...-...---...-....Setter: PG

J3

Section B
Answer all three questions from th is section.
The last question is in the form of ar. either/or ar.d only one of the alternatives should be

lla!k>

"""'

Tre rate of prod:.iction of hydrogen by the reaction of magnesium with 0.5 mol/dm 3
of excess hydrochloric acid was investigated using the apparatus shown below.

fresh page.

attempted. Write your answers in the writing paper provided. Start a new question on a

811

~![
!

Alit>erbang1
""""'""'
:.nagnesiumf~~---,,..-: -.-

ooN<>ne add

Time in minutes

Tota l volume of hydrogen in cm"(at r.t.p)

18.0

9.5

26.5

10

15

33.5

0.5

20

48.0
54.0

41 .4

59.0

2.5

3.5

3.0

40

62.0
63.0

45

I
2011 "JAS Chemistry Prelim Paper 2

63.0

The following results were obtained at room temperature and pressure (r.t.p)
using 0.0699 g of magnesium.

'

IT
Setter. PG

(a)

lb)

(cj

(d)

Setter: PG

14

Sketch a graph of voiume of hydrogen collected against time.

(2)

(1)

On the same graph, sketch and label the curve you would expect if

[1 ]

[2]

Calculate the volume of t.ydrogen gas that cou:d be obtained in this

experiment.

tre 1-.ydrochloric acid has beer" heated to 40C before the

(ii)

!hi)

experiment fro'TI 1 mole of magnesium

How does your answer compare with the 24 dm 3 normally accepted for the

[2]

molar volume of a gas? Suggest a possible reason for the discrepancy you

The experiment was repeated with 1.0 mol/dm 3 of excess hydrochloric

have found?

[2]

When a similar experiment was conducted with pieces of calcium

acid, how would the rate of reaction be? Explain your answer.

carbon dioxide was found. Why do you think the value is so low?

[1]

carbonate instead of magnesium, a value of 16.8 dm 3 for the volume of

[Total: 10)

2011 NAS Chemistry Prelim Paper 2

812
(a)

(b)

16

8 13 Eith er

15

Read the article below and answer all the questions that follow.

(1)

(3)

(2)
(Total: 10)

2011 NAS Chemistry Prelim Paper 2

Gasoline, diesel, and

vegetable oil have

different compositions. Gasoline and diesel are

each a mixture of hydrocarbons-molecules

composed of only atoms of hydrogen and

CH,

C~CH- ettz- C.- Cttt

CHa

often foun4 in ,uoi:ine.

are longer, they tend to bind more strongly than the

smaller molecules in gasoline. As a result, diesel is more

viscous than gasoline, that is, it is more resistant to flow

than gasoline. Especially in winter, diesel has a thick

honey.

consistency similar to the consistency of molasses or

Vegetable oil, by contrast, contains fat molecules, which

are composed of triesters. An ester is an organic

CH,-

OH

GlJUNll

fi

~ HO-C-

+,

n
n

3h.ttyAciclt

mar rm tie rie~,

~31Ul

(Wlte1~ A re;mtten~
a lo<-~ CCC d!iW1}

II

1)

ri

C-

ri

Cll,-~ - C - R

I
I

O!- G-

fal(lrig~"de)

~- 'G - G-R"

{-""' R, R', mt R' m:l'f

201 1 NAS Chemistry Prelim Paper 2

compound formed when an acid and an alcohol react,


releasing water.

Setter: PG

"-Oil
&.-OH
I

through weak forces called intermolecular forces. But because the molecules in diesel

In both gasoline and diesel, the hydrocarbon molecules are attracted to each other

respectively.

Figule 2. Cetane it a typical hydroarbon molecule loiind in diesel.


Examples of molecules present in gasoline and diesel are shown in Fig. 1 and Fig. 2 ,

CHx-~- Cli,- ~-CH,-~-CH~-Cft -Cl-i:-C~~-CHr-~- CH;--C!it-~

contains longer molecules wrth 10-24 carbons.

molecules with 5-12 carbons whtle diesel Figure 1. lsooctane is a hydrocarbon molmde

carbon. But gasoline contains hydrocarbon

CH!

Gasoline, diesel and vegetable oil; What's the difference?

and sodium chloride was formed.

!-chlorine

Sodium iodide

=-

State two observations which would be made during the reaction.(2)

jar 1s chlorine gas?

Describe a chemical test to confirm that the gas present in the gas

sodium

ln an experiment shown below, a piece of sodium was burnt in chlorine

(i)
(ii)

Sodium chloride

~
~QA
r;:: l)]'/l~llf
~

(1)

The sodium chloride formed was used in another experiment as shown in


the diagram below:

gentle heat
Name the type of reaction that produces P.
Give the name and chemical formula of P.

(1)

(ii)

any one of the reaction in part (b)

Wnte a balanced chemical equation, including state symbols for

Q and R.

element. Identity the element and give the colour of the element at

It was observed that Q and R are two different states of the same

(iv)

(iii)

(i)

Floorioog"

Setter: PG

Paper1

1
2

Paper2
Section A

ammonium sulfate

(1]

(1)

(1)
c)

alumnium oxide

[1]

d)

bromine

silver carbonate

e)

copper(l l} chloride

a}

31

b)

Chemist ry Prel im Paper Answers

2011 Ngee Ann Secondary School


A1

21

(1)

11

-D

c-

(1]

[1)

32

negative ion

[1)

a)

Covalent bonding

Group VI

22

b)

A2

c)

12

[1]

34

B
A

(1 ]

[1)

33

B
35

24

a)

23

A
25

b)

14

A3

c)

is more reactive than a lithium atom.

Addition polymerization

c:=N

I I

II

C-0-CH 3

c-c H

Ester functional group

2011 NAS Chemistry Prelim Paper 2

its valence electrons compared to the lithium atom [1]. Therefore, caesium atom

compared to the lithium atom (1) and hence it is easier for caesium atom to lose

[1)

The valence electron of the caesium atom is futlML~ from its nucleus as

Setter: PG

c)

b)

a)

a)

13

c
15

A4

A5

b)

4
D

37

26

36

B
27

16
17

B
D

6
7

39

38

40

29

28

c
-30

19

18

D
20

10

?01 1 NAS Chemistrv Prelim Paoer 1

AS

A7

Fossil fuels are carbon containing compounds. reacts with oxygen to produce

Carbon dioxide is a greenhouse house gas{1), traps heat on Earth, leading to

carbon dioxide. (1 )

N2 +Or> 2NO

a)

b)

(i)
Nitrogen oxides irritates eyes, may cause breathing difficulties by irntahng

[1]

A8

d)

a)

Mole ratio of CH3COOH:NaOH is 1:1

liberated in forming new bonds. [1)

The total amount of energy absorbed in breaking the bonds is less than that

(i)

No of mole of CH 3COOH

Mass of CH3COOH in 1dm3

= -2

=2.625 x 1021(25.0/1000)
=1.05 mol/dm3 (1)

has a higher concentration of

[1)

[1)

=1.05 mol x (12+3+12+16+16+1)

= 63.0 g

= 6.00%

Mass of vinegar in 1dm3 =1 05 x 1000


=1050g
=63 0/1050 x 100

% by mass of CH 3COOH

For the same price of the vinegar. Brand

Brand Xwill be a better buy (1)

(ii)

No of mole NaOH =1.00 x (26 2511000)


=2 625 x 102 mo!
[1]
=2 625 x 102 mol

b)

=+6

(1 )

let x be the oxidation number of S

ethanoic acid in the vinegar. (1)

(i)

X + (4X -2)

x-8=-2

so, to +6 1n so.2'[1 J

502 is oxidized (1)as the ox1dahon number of sulfur increased from+ 4 in

Electrolyte

(ii)

a)

Sodium ions, chlonde ions. hydrogen ions. hydroxide ions (0.5 mark each)

(1)
b)

a)

the lungs and even causing bronchitis. Nitrogen oxides also react with

(1)

[1)
(1)

A9

Concentration of CH 3COOH

limestone buildings, destroys aquatic life and plants.


Carbon monoxide combines with haemoglobin in the red blood cell to

leads to death. (1)

form carboxyhaemoglobin, which reduces the blood capacity to carrv

In the car engine, the high temperature causes the oxygen and nitrogen

~which

molecules to have kinetic energy which is higher than the activation

faster speed of reaction .

eneray(11 This lead to increase in frequency of effective collis1on(1)hence

Sulfur dioxide 11 J.Sulfur dioxide is produced during the combustion of

= (600 + 24000) = 0.0250 mol


= 72.0 kJ

0 0250 x 2880 kJ

A10

oxygen and dissolve in rainwater to form acid rain, which corrodes

global warming which eventually lead to more floods in low-lying areas(1].


c)

(ii)

(i)

(ii)
fossil (sulfur containing) fuel. (1)

No. of moles of butane burnt


Amount of heat evolved

-2880 kl

Time

[1) - balanced equation, (1)- correct state symbols (only if equation is balanced)

a) 2C.H1o (g) + 1302(9)--+ 8C02(g} + 10H20(1)

b)

c)

Energy

' C.H10 (11) 6~ O, (&J.

' 6H

(1) - correct shape (exothermic),


[1) - reactants and products written with Ea and 6H

c)

a)

Section B

B11

b)

(i)

Copper

[1]

(1] - diagram, [1] -fully labeled

Zn (s) - Zn2 (aq) + 2e


{1]

Zinc

Aqueous
sodium chloride

Use a piece of moist blue litmus paper would turn red then bleached[1)

(ii)
H 2 (g)

(i)

Sodium melts and bums with a brighVyellow flame.[1]

2H.(aq) + 2e -

(ii)

A white solid would be formed.(1j

(ii)

(i)

lodine(1), purple at

Chlonne and Cl,

Displacement reaction
[1)

[ 1)

Sodium decrease in size.[1]

(iii)
F2 (g) + 2NaCl(S} -

2NaCI (s) + 12 (g)/(s)

2NaF (s) + Cl2 (g)

a [1J and black at R[1 J

(iv)

OR
Cl2 (g) + 2Nal(s) -

<: f"hcl"nictn 1 Pralim PQnPr?

[1 ) - balanced equation. (1)-correct state symbols (only 1f equation 1s balanced)

1n11 f\lfl

B12
a)

(i)

[1] for steeper gradient.

s.s

OR

Time/min

1 mol of Mg . 63/ 0 .0029125

0 .0029125 mol of Mg 63.0

cm' of

"'0 0029125 mo

No of mole of Mg "' O 0699/24

[1 ] - correct shape, [1] - fully labeled (axes with units and values)

63.0

Vol of hydrogen/cm'

(u)
(tii)

Mg +2 HCI ~ MgCl2 + H2
0.0699gofMg 63cm' ofH 2
1 mol of Mg 1 mole of Hi
24 g of Mg 2 g of H2
0 0699 g of Mg Q 0699 x2
24

=0.005825 g of H,

= 21630.9cm1

It is less than 24 dm3(1) as Mg 1s not pure.[1)

=21 .6 dmJ(1]

b)

No of mole of Hi= 0.005825/2


= 0.0029125 mole
= 69.9crn3[1)
Percentage yield =53/69 9 x 100
90.1%
Vol of H2 produced= 90.1% x 24 dm 1
= 21 6dm;{1j

C)

Rate of reaction will be faster. (1) When a more concentrated hydrochlonc acid is
used, there are more W ions per unit volume. The frequency of effective collision
increases {1) and hence resulting in a faster rate of reaction.

2011 NAS Chemistry Prelim Paper 2

Carbon dioxide 1s slightly soluble in water as some of 1t is dissolved in the water.


[1)

d)

Setter. PG

813 Either
a)

C)

I)
lsooctane - CaH1s(1), Cetane - C1sH34(1]

ii) 2 CaH1a + 25 Or+ 16 C02 + 18 H2 0 (1)

C,eHJo<- C2Ha + 2C3Hs+ 2C.He[1)

b) Presence of ultraviolet lights. (1)

i)

H
I

H
I

H
I

H
I

H
I

H
I

H
I

I
I
I

I
H

c - c - c - c - c

c - c
I

- c
I

H
I

H
I

H -

C -

I
I
H

- c H

?011 NAS r.hP.m1~trv PrP.lim PAnAr?

C -

c
H

- c - c I

c
H

- c -

- c - c - c - c - c - c

- c - c

ii) Add aqueous bromine (1) Butene will rapidly decolovrise brown aqueous
bromine (1)

d) Any 2 of the following

Sl'ttAr pr,

SP.ttP.r pr,

- c -

- c -

- c - c -

- c

- c - c - c - c - c
I

C -

- c

H -

2011 NAS Chemistrv Prelim Paoer 2

- c - c - c - c - c - c -

- c - c - c - c - c - c

<"-............ _. nf'

H
I

H
H

H
H -

I
C -

H
H

I
H

- c -

H
H

2011 NAS Chemistry Prelim Papet 2

I
H -

c - c - c - c - c -

H -

H -

H -

10

manganate(Vll) tums colourless (1)

- c - c - c - c

~potassium

- c

I
H

I
I

C H

Prelim Paoer 2

Setter: PG

I
H

I
H

?011 NA~ C"'.hP.m1~trv

e) AC1d1fied

c I

H -

H
I

- c
I

- c - c - c - c - c I
I
I
I
I

I
C

I
H

I
H

H
I

- c - c - c - c - c

H -

- c

- c - c - c - c - c - c I
I
I
I
I
I

H
H

H
Q

813 Or

b)

Weigh the milk again after evaporating the water.

42.1

Carbo'.:_ _ _ _

l
I

142 1 / 12 =3.5

6.43

hydrogen

51.47 / 16 = 3 22

51.47

Oxygen

(1)

6.43 / 1 =6.43

Dry mass I original mass x 100% = 13.0 %

Mass
Mole

no of H atoms= 6.43 x 3.43 = 22.05 =22

(1)

no of O atoms = 3.22 x 3.43 = 11 .04" 11


Formula of lactose= C12H22011

[1)

Since lactose has 12 carbon atoms, n = 1213.5 = 3.43.

(1)

[1)

a) Weigh a sample of milk, then evaporate the milk completely.

OR
% of carbon= 42. 1% = 12 x 12 = 144 mass units

(1)

?011

N6~

t"'li#tomidn1 Pro1im

P:::.~,.?

By proportion, hydrogen which has 6.43% should have 22 mass units and oxygen which
So no. of hydrogen atoms = 22/1 = 22 atoms

has 51.47% should have 176 mass units.

I
l

H-C-H
H

11

H 0
I II
H-0-C-C-O-H

Formula of lactose= C12H22011

no. of oxygen atoms = 176/16 = 11 atoms

c) [1]

~~tt~r - P~

d) [2]

e)
(i)

(ii)

(1)

11

II

I II

H 0

H-C-H
I
H

.,

H -C- H
I
H

1?

[1)

?n11

NA~

II

r.homidrv Pr'3olim

H-o-c1~~]-c-o-H
:.~.

II

H-C-H
I
H

I
N-H

-O-C-C-O-C-C-O-C-C-0-

Amide linkage

I
H-N D

C:::.ott.cw Pt:

P~n~r?

North Vista Secondary School


Chemistry
Papaer 1

methane

hydrogen chloride

carbon monoxide

ammonia

Which of the following gases cannot be dried using concentrated sulfuric acid?

North Vista Sec

0
0

0
"Q

chromatogram A

first starting
line

rotate 90 clockwise

original
position of Q
chromatogram B

second
starting line

A chromatography experiment is performed to determine the number of amino acids


present in a sample of protein mixture Q.
Chromatogram A shows the separation or amino acids in the mixture using solvent X.
The chromatogram is then rotated 90 clockwise and placed inside a second solvent.
Y. The results are shown in chromatogram B.

Four identical gas jars are filled with different gases.


The lids are taken off the gas jars, and they are left open to the air for a few hours.

How many d ifferent types of amino acids are present in protein mixture Q?

first starting
line

t1 B B B
Which gas jar will then have the most air in it?

ssc.

In an experiment, the melting point of an unknown solid, X, was found to be 70C which
1s the same as an 11-carbon hydrocarbon. The experiment was repeated by using one
part of X with two parts of pure 11-carbon hyClrocarnon. The mi.xture starts tu 111ell al

X may contain 11-carbon hydrocarbon.

X is not pure 11-carbon-hydrocarbon.

X is pure 11 ~rbon hydrocarbon.

X is a mixture.

Which of the following is a correct

What is the best deduction from these observations?

Deuterium . ~O . is an isotope of hydrogen.


description or deuterium?

Deuterium 1s chemically more reactive than

Deuterium. D,, has a lower melting and boilirg point than hydrogen, H2 .

Deuterium has the same electronic structure as hydrogen.

Deuterium is more abundant than hydrogen on earth.

~ydrogen

A g iant molecular structure with covalent bonds between the carbon ions.

A giant molecular structure with covalent bonds between the carbon molecules.

Diamond is the hardest natural material known. The hardest diamonds c an only be
scratched by other diamonds It is generally chemically inert and has high melting and
boiling point.
Which types of structures explain these properties?
A

A giant molecular structure with covalent bonds between the carbon atoms.

A giant molecular structure with covalent bonds between the layers of carbon
atoms.

Cao

Si02

P.O,

cr20,

Fibreglass is used as " rP.inforc1ng agent in many polymer products. It contains a


mixture or ionic oxides and giant covalent oxides. Which of the following is not a
possible constituent of fiberglass?

72

150

"'""'l'l11t::ihl,. ...e.. 1 nm1f.lnP Punt/,j

130

160

The scent of banana is due to the chemical compound called isopentyl acetate. The
formula of this compound is C 7 H,.02
What is the relative molecular mass of isopentyl acetate?

n .... v"'"'"

10

11

12

13

14

A complex mixture of calcium compounds, proteins and other substances Is known to


be present in human bone. When a bone is heated strongly in a generous supply of
air, the only residue left Is calcium oxide.

10.0%
B

20.0%

c
28.0%

D
71 .4%

When a bone sample weighing 100 g is heated in air. 28.0 g of calcium oxide Is
obtained. What 1s the percentage by mass of calcium in the sample of bone?
A

~ll

People with kidney problems are advised against eating carambola fruit, commonly
known as starfruil, as it contains a significant amount of oxalic acid.

CHO

s.

CH02

roton n umber
20
17

54 g

C,H20

n uc l eon n umber
40
35

75g

110 g

C2H02

Oxalic acid contains 26.7% carbon and 2 .20% hydrogen by mass. The rest Is oxygen.
What Is the empirical formula of oxalic acid?

element

Element X and Y can form a compound Z.

37 g

What is the mass of one mole of Z?


A

14.0 g

B
28.0 g

74.7 g

151 .6 g

Motorcars are often fitted with airbags which rapidly inflate during an accident,
protecting the passengers. Tre bags are filled with nitrogen gas, which is produ::ed
from the decomposing of solid sodium azide, NaN,. into its e lements.
In a car crash, a typical airbag is filled with 84 dm3 of nitrogen. measured at r.t.p. What
is the mass of sodium azide, NaN,, needed to produce this amount of nitrogen?
A

Methane. CH . bums in oxygen to produce carbon dioxide and water.

50 cm 3
B

100 cm'

150 cm'

350 cm'

In a reaction, 100 cm 3 of methane was heated in too cm 3 of oxygen.


What is the total volume of gas, measured at r.1.p, left at the end of the reaction?
A

Tartaric acid is commonly found :n red wine . The molecular fonnula of tartaric acid is

c .HaOa. One mole of tartaric acid is neutralised by two moles of sodium hydroxide '

0.16 mol/dm'

0.04 moVdm3

0 .:?2 mol/dm3

O.C8 moUdm3

25.0 cm3 of a sample of red wine require<l 20.0 cm' of 0 .100 moUdm' sodium hydroxid,
for complete neutralization Assuming tartaric acid is the only acic! found in red wine,
what 1s the concentratJon of tartaric acid in the sample of red wine?

20

21

22

6
In an expP.rim1mt. 8 0 cm 3 of 1.0 moVdm3 of sodium carbonate solution was added to

4.0 cm3 of 1.0 mol/dm3 copper(ll) sulfate solution. What would the resulting solution

colourless solution

green precipitate in blue solution

contain?

green precipitate in colourless solution

white precipitate in colourless solution

heat with carbo'ii

Si
(impure)

II and Ill

I and II

1v/

I only

heatZith
hydrogen

heat with chlorine

(pure)

SiCJ,

distill! Ill

(impure)

SiC/,

The reaction scheme represents the process for obtaining pure silicon from sand.
S102

Si
(pure)

A
I and IV

In which stage(s) is the silicon reduced?

C'

!fll.\\.'tl< ~
7
znso.

2H20

H20

H2

H2so,
+

Zn
Zn SO,

co,

In which reaction does dilute sulfuric acid act as an oXld.lfil!g agent?

znso.

A
7

ZnO
H,so,

Zn(OH)2

H,so,

B
+

c
znso.

T't.;-L,- 1).,,..,/,.

H,O

H2 SO,

Zn CO,
+

- - n w.. -~ ..... H.I. 9

23

24

positive
electrode

A piece of litmus paper was soaked in a concentrated aqueous sodium chloride and
supported on a glass shde. The paper was connected to a d.c. supply as shOwn In the
diagram.

negative
electrode

litmus paper soaked in concentrated


aq;eous sodium chloride

red

blue

bleached

no change

n egative electrode

blue

bleached

no change

bleached

positive electrode

electrolyte

Which one of the following shows the correct observations near the negative electrode
and the positive electrode after the current had flown for some time?

Apparatus was set up as shown below.

met al X

zinc

iron

iron

copper

metal X

silver

magnesium

aluminium

ZIOC

metal Y

For which pair of metals would electrons flow in the direction shown?

Our Viston: .Responsible & 1'hinki11g Pupt!s

26

26

27

reactant'

E,
.6.H

The enthalpy diagram shows an uncatalysed reaction .

>.

e'

"'c:
"'

products

--L---

28

decreases

Ll.H

Using 0.5 g of granulated zinc at 20C.

Using 0.5 g of granulated zinc at 40C.

Using 1.0 g of granulated zinc at 20C.

Using 1.0 g of powdered zinc at 20C.

flammability

In the graph, curve X represents the results of the reaction between 1..0 g of g ranulated
zinc and an excess of acid at 30C.

vol of gas/cm 3

---- - -- -- -- - y

e.
unchanged

time

decreases

Which change will produce graph Y?

decreases

The reaction was repeated in the presence of a catalyst.


What effect does the catalyst have on the activation energy, E and the enthalpy
change. L:>.H?

A
increases

viscosity

The acidity of the elements increases across a period.

The oxidizing power of the elements decreases across a period.

The reducing power of the elements increases across a period.

The melting and boiling point of the elements increases across a period.

malaysium oxide would not react w ith thailandium.

singaporium oxide would only react with indonesium.

singaporium

thailandium ,

indonesium

thailandium,

malaysium

cambodium

singaporium

indonesium

singaporium

cambodium

malaysium_,. cambodium

cambodium

malaysium

malaysium

singaporium

thailandium

indonesium

thailandium

indonesium

From these results, deduce which of the following correctly lists the metals in the order
of i ncreasing ease of oxidation.

ii

An American scientist visited South East Asia and returned with five samples of metals
for further research. He named them singaporium, malaysium, cambodium, thailandium
and indonesium. He decided to place them in a reactivity series by investigating which
metals reacted with oxides of the other metals when heated. He found that he could do
this in just two experiments. His findings were:

Which of the following descriptions of the elements in the Periodic Table is correct?

increases
decreases

29

c
unchanged

boiling point

The mass of magnesium is decreasing.

30

A
melting point

Which physical property of the crude oil fractions does not increase as relative
molecular mass increases?

vol of gas/cm 3

When a strip of magnesium is dropped into excess hydrochloric acid, the rate of
reaction is followed and the results are plotted as shown below. Which statements
best explains the shape of the graph?

The solution is becoming hotter.

The magnesium is acting as a catalyst.

time

The surface area of the magnesium is increasing.

C
D

31

31

35

34

35

~
I

syringe

40 cm3

10

copper

beat

I
t I I I

:r
syringe

\:

80 cm 3

'

~
3

160cm

In an experiment, 200 cm3 of air were passed over heated copper until the reaction was
complete.

20 cm 3

What is the final volume of gas after cooling to the original temperature?
A

38

Y7

11

Ethanol is used in some after-shave lotions and deodorants. Which pair of properties
makes it suitable for these uses?

It mixes easily with water and is less dense than water.

It is flammable and mixes easily with water.

It is colourless and freezes easily.

It is a good solvent and vapourises easily.

II

One of the chemicals used to make the hard outer covering of golf balls has the
following structural formula:

Nitrogen is reduced by hydrogen.

It is an ester.

CH3

C=c ...c , o ,...cH3

It is not possible to obtain 100% yield of ammonia.

A higher temperature will increase the yield of ammonia.

It forms alcohol in the presence of steam at high pressure and with the use of
catalyst.

Ammonia is manufactured by the Haber process. Which of the following statements is


incorrect?

B
CH1

HCI (g)

C,H,C/ (g)

C1Ha

UV light is required for the reaction to take place.

It Is a subst1tut1on '"""lion.

C1H1

It contains many double bonds between carbon and hydrogen atoms.

It burns in air with non-smoky name.

It forms a solid compound on hydrogenation.

It will undergo complete corrbustion to form carbon dioxide and water only.

below.

It can undergo both addition and condensation polymerisation.

s~own

Compound w has the structure

H H H HI
t
t
I
H-CCC-C-O-H
t
I
l
I
H H H H

H- c-c-C-0H
I
I
H H
H-CH
t

Which of the following 1s an isomer of W ?

H H HI
I
I
H-c-c-co-H
I
I
I
H H H

I
A

I'UJ)l/.f

H H H

0-H
H H H
I
I
t
H-C-C-C-C-H
I
I
t
t

H HI HI
I
H-C-C-C-O-H
I
I
H
H
H-C-H
I

H
H
H
t
I
I

Which of the following statements about this molecule is incorrect?

c
Catalyst is used to increase the speed of the reaction.

38

C6Hs

The fourth member of a homologous series has the molecular formula CeH12. What is
the molecular formula of the first member of the series?
A

The reaction between the hydrocarbon C,H6 and hydrogen chloride can be represented
by the equation
C,H 6 (g)

The molecular formula of the hydrocarbon is C2H0

Which of the following statements about the above reaction is true?

The molecular formula of the hydrocarbon is C,H.

II produces margarine when it reacts with steam in the presence of nickel


catalyst.

A vegetable oil is described as polyunsaturated. Which statement about this vegetable


oil is correct?

nur Vi.t>ion: Resoonsible & ThinklnI!

39

40

CH3 COOH and C,H7 0 H


C2 H5 COOH and CH 3 0H
C2HsCOOH ,and C3 H70H
C3 H7 COOH and C2H0 0H

12
A food chemist wants to aeate the odour of pineapples for a product. An ester with
this odour has the formula C,H,C02C2Hs. Which pair Of reactants WOUid produce !Ills
ester?

A
C

0
Kelvar is a new product which is aromatic polyamide (aramid). It is five times stronger
than steel. It is made from hght atoms such as carbon . hydrogen and oxygen, hence it
is mueh lighter than other strong materials. It is useo in making bullet proof vests and
protective suits for motorists. Its structure Is:

ftoJ-z---0-zf

H2NOC- - - o -CONH2

H---0-H

HOOC---o-NH2

HOOC---0-cooH

and

and

and

and

H,NOC- - - 0 -CONHz

H,N---0-COOH

HOOC---o-NH 2

H,N---o-NH2

Which two molecules could be condensed to form Ketvar?


A

13

lead(ll) hydroxide

iron(ltl) hydroxide

iron(ll) hydroxide

copper(ll) hydroxide

calcium hydroxide

aluminium hydroxide

white

white

red-brown

green

light blue

white

white

Colours of Some Commo n Metal Hydroxides

zinc hydroxide

North Vista Secondary School


Chemistry
Papaer 2

A3

What would you see if aqueous ammonia is added to solution S?

(1)

121

Write the equation for the reaction between chlorine gas and iron(ll) bromide

structural formula:

Name liquid T and give the structural formula of the compound formed when it
reacts with but-2-ene.
[2]

Some chlorine gas was passed into aqueous iron(ll) bromide, the colour of the
Golution immediately changed from pale green to orange-red. When the orange-red
solution was heated, it gave off a brown vapour, leaving a yellow solution, S. The
brown vapour formed a dark red liquid, T, on cooling. When but-2-ene was added
to T, the red colour d isappeared.
(a)

(b)

(c)

[Total: 5 marks]

A4

O,(g)

N2(9)

2H20(g)

Hydrazine. N 2 H,, burns in oxygen according to the following equation.


N 2 H,(g)

945

bond

N-H
N N

158

498

bond energy/
k J/mol
391

O=O

Calculate the change of heat of reaction . l\H . for the combustio.n of hydrazine.

464

N-N

(a)

Is the reaction endothermic or exothermic? Explain your answer in terms or


the energy changes that take place during bond-breaking and bond-forming.

0-H

(b)

Draw the energy profile for the reaction. Include in the diagram. the value of
tiH as calculated in (a) and indicate the activa tion energy for the reaction. (4)

(2)

(2)

(c)

[Total: 8 marks]

AS
(a)

tor

I
pH

14

Identity

The labels of four c olourless solutions, sodium hydroxide, sodium chloride,


el hanoic acid and hydrochlor:c acid, we re M ixed up accide ntally. They were
relabelle d as solution A. B. C and o and the pH ot the solutions were tested .
solution
1

I
I

A
B

c
0

A6

Lead compounds are extensively used to provide the colour in paints and pigments.

c. 4.5%,

o. 18.0%

'W/11(" lead', used for over 2000 years as a white pigment, i s based upon le ad
carbonate. Analysis shows that lead carbonate has the following percentage
composition by mass.

Pb, 77 5%,

'Red lead'. is the pigment in paint used as a protective coating for structural iron
and steel. It is based upon tead(IV) oxide. Pb3 0,, a scarlet powder formed by
combining lead(//) oxide powder with oxygen.

Calculate the e mpirical formula of lead carbonate .

i2)

[Total: 5 marks]

(2)

[2]

(a)

Complete the table above to show the identity of the four solutions.
[2 1

(I)

Assum ing all solutions have the same concentration, explain why
elhanolc a cid and hydrochloric acid have 01fferent pH values
(2)

Write the ba lanced chemical equation for the formation of the 'scarle t powder'.
(1)

(ii)

(b)

Calculate the mas s of Pb 3 0, that cou ld be formed from 66.9 g of PbO .

Ca lci;late lhe rel ative molecular mass o f the carboxylic acid.

[1 ]

II(

(iii) Write the ionic equation for the reaction between solutions A and C

(c)

(I)

Identify the c arboxylic acid .

(1)

In a separate experiment, 25.0 cm 3 of a carboxylic acid completely neutralised


22 .5 cm 3 of 0 .1 mol/dm3 sodium hydroxide . The concentration cf the
c arboxylic acid is 6 .66 g/ dm'.

{ii)

[Total: 8 marks]

Our Vision: Resoonsible & Thinking Pupils

/l:

8
'Blister' copper ;s impure copper. The ;mpuri1y was a metal that is below copper In
tho reacti~ity series. The diagram bolo.., shows thP. apparatus usr.d for 1cfinirg lho

:.abel 1he two e1ectrodes ar.d the electrolyte to complete the set-u.

l1 I

131

copper.
- +

(a)

Write the equation for the reaction occumng at the cathode.

11---i

(b)

Whc.t happer.s f the ccncentratic n of the electrolyte during the electro'ysis?


Expla-, your answer.
2;

Explain ~1hy the anode decreased in mass.

what you would observe just t:.clow Ille ar.ode after eleclr'>1ys1s.

\Nh11t is '.l,e per..:entage i:urity of tte 'blister' copper?

Ex~lli'l

(H)

(I)

(2)

(1)

[1)

T~e loss of rr ass of the a'lode was 52 g and the gain ir rnass o1 the cathode
was 4'3 g.

(..:)

(d)

(e)

[Total: 10 marks]

Sect ion B

Read the fol'owing article on reactive oxygen sriccies (ROS) f-om Wikii:,ea:a and
answer the questions that follow.

Answer all t h ree qJestions from this section. The last question is ::1 the form of either/or
ard OOI/ one altern'llive should Lle attempted.
81

Reactive oxygen species (ROS} are ger.%711/ very small molecules and are
high;y 1eac1ive due to the presen..e of ui1,:,aired etectr::ms in their valence shells.
Supero1<iaes and peroxiues are t~o examples of ROS. They are formed as a
r.aiural by-product cf the r.ormal metabolism of oxyr;f:n in living thinr;s. They are
oxidising in nature and high tf!vals will resut in cell damage which may
eventually tea? to ir.ar.y forms ci canr:er and agmg.

A superoxide is the anion 0 2- w"ch occurs ..,,idely i.; nature. It ccntai;s an


oxygen--0x1gen single bond Wtth ""e unpaired electron, the superoxiue icn is a
highly reactive part:cle A peroxioe a1sc contatt'S an 01<ygen-cxygen single
bono. The simples1 stable peroxide 1s hydrogen peroxide.

\!)

Deduce the 1~pe >f bonding ir. 1he su:>eroxide anion and rycirogen
peroxide. Draw dot-and-cross' diagrams to s.1ow the 001C::'lg in each
one o' !hem You need to draw only the valence electrons.
[5]

:::letermine the oxidatior. state c;f oxygen in tl1e superoxide anion and
iydroger1 eroxide.
[2]

wh'le hydroger. peroxide has the

tj.)

Wit!" reference to the 'ciot-and-crcss' o:agram :n (ii) above for the


[2]
Guperoxide anion explain why it is OJ<idisir.g in nat ure.

2 ,

ill

The superoxide ;nin has the forrr1>':?, 0


formula H2 0 2

Hydrogei; peroxide can undergo d1sproportionat1on.


Disproportionation 1s
used to describe a spec1f1c type of redox reaction in which a species is
simuf3neously reouced ar.d ox1d1zed so as to form tiJo different products.
(a)

1\J)

Decompcsitlon of h~drogen peroxldu into "l<lit:r arn.J l:XJYt:ll is" uisproporliun


reactior . Construct a che11'cal equatio<i :o show the dsprooortioration and
expla in why tills is a redox reaction.
[3]

(Total: 1 2 marks}

li2

10

[1]

(Ii)

Explain, m terms of collisions between particles. the rate of this reaction


increases as the concentration of oxygen increases.
(1)

Construct a balanced equation for this reaction.

(Ii i)
Explain how carbon monoxide is formed in an internal combustion engine. (1)

Explain why the rate of this reaction increases as lhe engine


temperature Increases.
[1]

(i)

Nitric oxide, NO, is formed when oxygen and nitrogen from the air react in an
internal combustion engine.

Exhaust fumes from the internal combustion engines of motor vehicles contribute to
the poor quality of air in many cities The exhaust fumes contain atmospheric
pollutants such as nitric oxide, NO, and carbon monoxide. CO.
(a)

(bl

(I)
Construct a balanced equation for this reaction.

Explain why the catalyst should be in the form of a powder supported on


a mesh.
(1]

In a catalytic converter, nitric oxide and carbon monoxide in the exhaust


gases react together in the presence of a catalyst.

With effect from January 2001 , 1t 1s mandatory for all vehicles in Singapore to be
fitted with a catalyllc converter.
(c)

(ii)

[1 ]

(1111

Does a catalytic converter eliminate all pollution problems from motor


vehicles? Explain your answer.
[2)
[ Total : 8 marks)

EITHER

83

11

(a)

Draw a 'dot-and-cross' diagram for ethene.


valence electrons.

Construct an equation to show the cracking of dodecane to make ethene.

Etl1i;n., can be the starting material for the production of chemicals such o:i cth:inol,
ethano1c acid and ethane-1,2-diol. Ethene, c,H. is manufactured by the cracking
of long chain hydrocarbons such as dodecane, C12H20-

fb~

Describe how ethene is converted into ethanol industrially.

(1 ]

\ti

Ethanol reacts with hot acidified potassium dichromate(VI) to form ethanoic


acid.

(2)

You only need to draw the


(2)

(d)

(ii

Ethane-1,2-diol has the structure drawn below.

Describe the colour change that occurs during this reaction and draw the
structure of ethanoic acid.
[2)

between
(1]

\ti)

I I
H-o-c-c-o-H
I HI

Suggest the structure of the product of the reaction


ethane-1,2-diol and hot acidified potassium dichromate(VI).

Draw the structure of the liquid.

[1)

(1]

The product formed In (d)(ii) reads with ethanol to form a sweet smelling liquid.

(ol

Circle and name the linkage found in the liquid.

[Tot al: 10 mark s)

(fl

Our Vision: Resoonsible & Thinkina Pupils

OR
83

12

13

H H
H)::_,.C,(;_,.CH

iron(ll) hydroxide

copper(ll) hydroxide

calcium hydroxide

aluminium hydroxide

red-brown

green

light blue

white

white

Colours of Some Comm on Metal Hydroxides

'
'
H
H
butadiene

iron(\11) hydroxide

white

Styrene-butadlene Is a synthetic rubber. It is made by polymerising a mixture of the


monomers styrene and butadiene.

styrene

'c=c'
I
\

lead(ll) hydroxide

white

butadiene

styrene

butadiene

(a)

styrene

What type of polymerisation will take place when the monomers polymerise?
Explain your answer.
(2)

butadiene

Butane cracks to form butadiene and one other product.


equation to show this product.

:11

11I

Write an

\II/hen the mixture of styrene and butadiene polymerises. the polymer is


unlikely to contain only th is regular, repeating pattern. Explain why.
[1I

121

Give the full structural formula for the repeating unit in this polymer structure.

styrene

One possible structure for the polymer is shown below.

zinc hydroxide

(.b)

(c)

(i)

Give a use of the other product of this reaction.

2.90 kg of butane entered the cracking tower. After the reaction, 2.16 kg of
butadlene had been made. Calculate the percentage yield of butadiene.
i31

(ii)

Butadiene can be made by cracking butane in a cracking tower.


fd)

(e)

[Total: 10 mar ks)

c
7

B
29

19

39

10 B
20 D

3~~

40 A

Marks

1 _
1
1

1
1 OR
1
Total: 7
1
1

Total: 8

reaetarts and
produ:t 1

ShPO 1
Lab" of

6H1

E,1

Equal.on 1
Balancm<> 1
1
Total: s
2
1
1

1n eotiect
oos1bon a1v 1

2
fonlyC_,,., 0

Total: 7

,,

Max2

ans'f\er

any wrong

-1 morkfor

c
c
c

2011 NVSS Secondary Four Express Preliminary Exam Pure Chemistry Solutions
Paper 1 MCQ
1

c
28

A
12

11

c
c
c
18

27

17

16

26

24

14 A

38

B
D

15

c
37

D 1 25

23

I 35
D

36

34

13

D
1 22 A
33 A

1 11

D 1 32 C

12

1 21

1 31

neutron
3

.. Chara

electr on

Markin< Points
numWr~
p~rt1ete% ;;:,,_ name
proton
3

0
0

Paper 2 Structured and Free Response


Qn
A1(a)
b~-

~Nv

..

A1(b)

as lithium but 1dfferent number of

Grouo I
Metal. Onl:t metal forms eositive Ion.
No. It is an isotooe of lithium.

Bromine
Structural formula:

Band 0
Metal Bis most likelv lead.
Lead forms insoluble lead(l l) chloride w ith hydrochloric acid
which act as a pr otective layer on the surface of lead thus
oreventina further reaction.
Metal A can be extracted bv heaton<> with carbon.
Metal C is more reactive than carbon. therefore will not be reduced by
carbon.
Metal C is hlahly reactive thus can onlv be extracted bv electrolysis.

C, A,B. 0

neutrons.

II has the same number of proton

AHcl
A1<dl
A11e\

A2(a)

A2(bl
A2(c)

A2(d)

A3<al

Br H H
H-t-t-6-t.1:-H

3Cl 2 + 2FeSr2 -7 2Br 2 + 2FeCl,

H Br H

Red-brown orec1oitate insoluble 1n excess an ammonia

A3(C)

N,(9) + 2H, O(g)

f-l~ation ene~y

A31d\

01(;1

A4<al
A41bl

4(391\ + 158. 498 + -1945 + 41464\1= -581kJ


Exothermic (delta His ne11at1vej
More energy is released during bond forming than absorbed during
bond braakma

A4(c)
N:H..(g)

--------- -~!:i.. .....

Qn
AS(a}(i)

--

Marks
Every 1
mistake - 1
mark

M arklno Points
identity

Max 2

Sodium chloride

Hydrochloric acid

pH

sol ution

E~~a~

Sodium hydroxide

1-----+--~---+--------'--

A
B

14

'blister' copper

1
1
1
1
Total: 10

1
1
1

1
Total: 8
Correct
labeling 1
mark each
Max 3

.J----,- -

1
1
1
Total: 11

1
1

M, /

Scirution

+ copper(ll) sulfate

__

A5lallii) Ethanoic acid is a weak acid while hydrochloric acid is a strono acid
Ethano1c acid releases lesser I-+ ions in water as compared to
hvdrochloric acid.
ASlaH1ill H (SQ) + OH.(aQ) -7 H, 0(1)
ASlbHi) All carbox'tlic acids are monob!!sic
_
_
Concentration of the carboxylic acid = (22.5 x 0.11/25.0 = O 09 mol/dm
1-~-.,,--,1--M. of the carboxylic acid 6 .66 / 0.09 = 74
Prooano1c acid
ASlblliil

0 30
0.1
Therefore mass of Pb,Q, = 0 .1 x [13x207l + 14 x 1611

pure copper

.............. ...

=68.5

A6(a)
element

Pb .
~ "
C
1;,r," O
u,
mass/Ok
77 5
4 .5
18.0
A,
207
12
16
no.of mol
0.3744
0.375
1.125
Simplest
0.3744 =
0 .375 =
1 .125 = 3
1
1
rat io
0 .3744
0.3744
0.3744
f - -- - l Therefore the empiricai formula is PbC01
=c....:c.:.._ _ _
A6lb)
6Pb0 + 0 2 -7 2Pb,O,
A61c)
No. of mol of PbO = 66.9/(20716) = 0.30 mol
From the eouation: No of mole of
PbO
Pb,O,

A7(a)

A7Cbl
A7rc)
~7{cHil

A7lcll11l
A71dl

Cu1aq) + 2e -7 CuCsl
Remained unchanQed.
The copper(ll) ion that was discharged at the cathode is replaced by
another con=rllll ion from the 'blister' conn<>r.
The anode which is the 'blister' conflf>r dissolved into the electrolvte.
PercentaQe pUrity =(48/521 x 100 =92.3%
A solid which is the imountv Present in the 'blister' connAr
The 'bhster' copper dissolved and no longer holds the 1mpuritv.

Qn
B1(a)(i)

A1 (a)(ii)

E\1 (a)(iii)

B1(b)

B2(a)(i)
B2(a)(ii)
BZ(a )(i ii)

B2(b)
B2(c)(i)
82(C)Oil
62(c)(iiil

Marking Points
Oxidation state of oxvgen in:
superoxide anion = -Y.
hvdr ogen peroxide = - 1
Type of bonding in both the superoxide anion and hydrogen
peroxide is covalent
'Dot-and-cross' diagram for superoxide anion:

~x

.@
~~

'Dot-and-cross' diagr am for hydr ogen peroxide:

0~ @

It is oxidising in nature because one of the oxygen has only 7 valence


electrons so
it has a high tendeny to gain 1 more electron to achieve the noble gas
str ucture.
2H202 - 2H20 0 2
OS of 0 changed from -1 in H, O, to -2 in H,O therefore it is a reduction
OS of 0 changed from -1 in H2 0 2 to 0 in 0 2 therefore it is an oxidation

N, O, '2NO
Concentration increase -7 chances of particles collision increases -7
more effective collisions 7 faster rate of reaction
Engine temperature increases - temperature increases~ partic::les

have more kinectic energy 7 chances of particles collision increases


7 more effective collisions ' faster rate of reaction
Incomplete combustion of the carbon-containing fuel due to insufficient

conversion.

Marl<s

1
1

electron

extra

[1jfor
Sharing of
1 pair of
electron
[1] for
showing a
charge of
1 [1] for the

arrangeme

correct

(1) for

showing

nt of
atoms
[1] for
the no. of
electrons
shared
Max f51
1
1

1
1
1
Total: 12
t
1

1
1
1
1
1
Total: 8

--- ---

Larger surface area for more collisons to take place to speed up the

.~!L_

2NO + CO '
N, + CO,
rio. the greenhouse gas carbon dioxide is produced
causing global warming.

Qn
EITHER
B3(a)
B3(b)

B3(c)
B3(d)(i)

63(d)(ii)

OR
B3(a)

83(b)

B3(c)
,...B3Cdl!il
B3(d)(ii)
B3(e)

C12H26

Marking Points

C2 H. + C10H22 ior anv oth er combin ation)

'-H

C=C

Ethene is reacted with steam at a high temperature of 300"C, 60 aim


in the presence of concentrated phosphoric acid as catalyst.
Potassium dichromate/Vll colour chanaad from oranae to areen

II

H
O
1
,,
H-C-C
1
H 0-H

II

HO'-.c/c......_OH
0

"

0 0
H H
H H
II
I
I
I
I
H- c-c-o -c-c- o - c -c-H
I
I
I
I
H H
H H

Addition polymerisation
Both monomers are unsaturated.

/H

f "HHH:dH

9-9--9-9

H X

Possible for polymerisation to happen between styrene monomers or


butadiene monomers 1 " before they further polymerise with one
another.
c . H10 7 C,H. + 2H2
Use in the manufacture of ammonia in the Haber orocess
Use as fuel.
No. of mole of C,H, 0 : 2.90/ [4(12) + (10x1 )] 50 mol
From th equation:
no. of mole of
C 4 H 10 : C 4 H6
1
1
50 :
50
Therefore mass of C 4 H6 expected= SQ x [4(12)+ 6(1)1 = 2 .7 ka
Percentaae vield (2.16/2.7\x 100 = 80%

Marks

Max. 2

Else double
b ond [1)
Correct no. o1
electron sha fe<:I
between all C
and H atoms [1]

1
1
1
1

Max [2]
1 ester
linkage (1]

Total: 10

1
1

Max2
Able to show
the styrene and
1 double bond
from buladiene
polymeri.tJng {11

1
Any 1 Max
[1]
1

1
1
Total: 10

Outram Secondary School


Chemistry
Paper 1

6.

7.

8.

0.

9.

are coloured elements


have low melting points
have catalytic properties
form gaseous compounds

Which of the following statements about transition metals is true? They


A
B
D

Lower densities

Less reactive

Higher melting points

Higher densities

Burn to form acidic oxides

Group II

The table below compares metals in Group I and Group II. Which of these
comparisons is incorrect?
Group I

B
Lower melting points

A Burn to form basic oxides

D Very reactive

Cu(OHh (aq) and NaOH (aq)


Cu(OH)2 (s), Na2S04 (aq) and CuS04 (aq)
Cu(OH)2 (aq), Na2S04 (aq) and CuS04 (aq)
Cu(OH), (s) and NaOH (aq}

In an experiment, 4.0 cm3 of 0.5 mol/dm3 copper(ll) sulfate solution was


mixed with 4.0 cm3 of 0.5 mol/dm 3 sodium hydroxide solution ir. a test
tube. Upon shaking the test tube, it can be deduced that the test tube
contains

A
D

10 cm'
50 cm3
90cm3
100 cm3

What volume of water must be added to 1o cm 3 of 5.00 mol/dm3 sodium


hydroxide solution to make it into a 0.50 mol/dm3 solotion?
A
B

lead(ll) oxide
nitrogen dioxide
sodium oxide
zinc oxide

Which of the following will dissolve in water to form an aqueous solution of


pH> 7?

A
C
D

11 .

12.

10.0 %
11.1%
90.0%
95.0%

An impure sample of copper was


refined as shown in the diagram on
It was found that the loss
the right.
in mass of the anode was 50g and the
gain in mass of the cathode was 45g.
What was the percentage purity of
this sample of copper?

A
B

-t---1

impure
copper
electrode

uqueoos
<-'<lp pe:(Il)
Sltlphztc

The calcium carbonate is all used up.


The hydrochloric acid is all used up.
A n insoluble layer of calcium chloride is formed.
The calcium carbonate is covered by bubbles of carbon dioxide.

When an excess of calcium carbonate reacts with dilute hydrochloric acid,


the reaction gradually becomes slower and finally stops. Which statement
best explains why this happens?
A

ec

brown precipitate

Questions 13 and 14 refer to the reaction scheme shown below.

I metalM I
!

pale green
solution P

'l dilute hydrochloric acid


colourless
combustible gas

chlorine
brown

solution Q I aqueous sodium


hydroxide

13.

14.

15.

16.

aluminium.
calcium.
copper
iron.

Metal M could be
A
8
C
D

colourless.
yellow.
pale green.
blue.

dampn:d lilmu.

mnmoniw.ai ddorid:e

"'"" plug

~
loeac

In the reaction scheme above, solution Sis

It remains red.
It turns blue and then bleaches.
It turns blue ard remains blue.
It turns blue and then red .

When solid ammonium chloride is


heated, it decomposes into ammonia
and hydrogen chloride. What happens
to the damp red litmus paper?
A
B

C
D

'

The reaction shown has in recent years become very important.


2CO + 2NO ~ 2C02 + N2
Identify the type of reaction and where it commonly occurs.

Neutralisation

Red ox

Neutralisation

Car exhaust

Blast furnace

Blast furnace

Where the reacti on takes place

Car exhaust

Type of reaction

c
Red ox

1?.

Reduction

Oxidising agent

In which of the following are all the definitions correct?


Oxidation

Gain of electrons

Loss of hydrogen Oxygen donor

Loss of oxygen ,

Loss of e ectrons

Oxygen acceptor

Electron donor

Oxygen donor
Oxygen acceptor

Electron acceptor

Electron donor

Reducing agent

Gain of oxygen /

a catalyst.
a reducing agent.
an oxidising agent.
a dehydrating agent.

2Ag (s) + HiO (I) + 0 2 (g)

ti. H = - 92 kJ/mol

The reaction between nitrogen and hydrogen is represented by the


equation:

Concentrated hydrochloric acid using platinum electrodes


Copper(ll) sulfate solution l'sing graphite electrodes
Dilute sulfuric acid using platinum electrodes
Copper(II) sulfate solution using copper electrodes

In which one of the following examples is there no change in the


concentration of the solution during electrolysis?

A
8
C
0

In this reaction, hydrogen peroxide behaves as

Ag20 {s) + H202 (I)

Hydrogen peroxide reacts with silver oxide according to the following


equation:

Gain of hydrogen

A Loss of electrons Gain of electrons Electron acceptor

~c

18.

19

2:J

N2(Q) + 3H2{g) ~ 2NH3(g)

46 kJ of energy evolved
46 kJ of energy absorbed

184 kJ of energy evolved


184 kJ of energy absorbed

What will be the energy change when 48 dm3 of nitrogen gas reacts
completely with excess hydrogen gas at r.t.p?

A
B
C

21 .

22.

23.

absorbs sunlight energy.


gives out heat energy.
builds up complicated molecules from simple ones.
uses chlorophyll, which is a natural coloured pigment.

Photosynthesis is an endothermic process as it

A
D

In the diagrams, O and represent a nitrogen atom and an oxygen atom


respectively.
Which of the following diagrams represents a mixture of two compounds?

ii

n~-

~u

' i

0-

l n l
I o---.1
tP ~

6 n
--.
, _I1 ~:>-- ..:.-J1
1 ~ . _ ..

r------- .
- ,, .. .:,-,:__",_
. -1 \ 't'
l
cf :
Jt:'l
o...
i
lr.
cJ , I
I~u ~. L-.ID
' i}'fl!? ~ 'Q,,\i,,tJ C~I
A

~
~
\

statements

c=-:~
'\
can

:(~

, ' f-ortl

~-Solvent;

be deduced from the

-lue

---~

The chromatogram of the dyes used for the colouring of a drink is shown
in the diagram below.

r-

L\--

stsr.ing line

following

The colour of the drink is red.


The Rt value of the yellow dye is less than that of the blue dye.
The yellow dye is more soluble than the blue dye in the solvent
used.
The molecules of the yellow dyes are smaller than those of the blue
dyes.

Which of the
chromatogram?
A

J3

Z4.

25.

26

Observations
White precipitate dissolved in excess
alkali to give a colourless solution.

An aqueous sample of T shows these observations with the following


reagents:
Reagent
Aqueous ammonia

Aqueous acidified barium nitrate White precipitate formed.

aluminium chloride
calcium sulfate
zinc sulfate
sodium chloride

What is compound T?
A
B

31

How many electrons are present in an ion of , 5 y 3 -?

12
15

18

16

A
B

Ol

Solid State

x
x
..j

"

Liquid State

x
Insoluble in
water

""

Aqueous
solution

The table below shows the ele::trical conductivity of four substances W, X ,


Y and Z. In the table shown below "..j " indicates ability to conduct
electricity and "x" inability to conduct electricity.
Substance

x
x
x
..j

w
x
y
z

SubstanceW
Substance X
Substance Y
Substance Z

Which of the substances is likely to be hydrogen bromide?


A
l;3

c
D

JO

27.

28.

29.

30

11.5
23.0
39.0
46.0

The compound Y2S contains 58.9% of Y by mass. What is the relative


atomic mass of Y? (Relative mass of S is 32.)
A
B

23

Statement (i) only


Statement (ii) only
Statements (i) and (iii) only
Statements (ii) and (iii) only

It contains 6.0 x 10 nitrogen atoms.


It occupies the same volume as 17.0g of ammonia gas at room
temperature and pressure_
It has a mass of 14.0 g.

Which of the following statements concerning one mole of nitrogen gas 1s


correct?
(i)

(ii)
(iii)

A
B

Cathode
Sooon
Gold
Spoon
Gold

Anode
Gold
Sooon
Gold
Sooon

Electrol}'!e
Gold nitrate
Gold nitrate
Sulfuric acid
Sulfuric acid

---

To electroplate a spoon with gold, which of the following eleclrodes and


electrolyte would be used?

A
B

The reaction between chlorine and dilute sodium hydroxide at room


temperature is represented by the equation:
Cl2(g) + 2NaOH(aq) -) NaCl(aq) + NaCIO(aq) + H20(1)

Chlorine is oxidised and sodium hydroxide is reduced.


Chlorine is reduced and sodium hydroxide is oxidized.
Chlorine is simultaneously oxidised and reduced.
Sodium hydroxide is simultaneously oxidised and reduced.

Which of the following statements concerning this reaction 1s correct?


A
B
C
D

II

31

32.

33.

34.

Qieces of solid
larger
larger
smaller
smaller

acid
less concentrated
more concentrated
less concentrated
more concentrated

tem1?erature
lower
higher
lower
higher

A chemical reaction between a solid and an acid is too fast fer safe use in
the laboratory for student demonstration. Which set of changes is most
~kely recommended for safe use?

A
B

magnesium hydroxide, aluminium hydroxide, calcium carbonate


sodium hydroxide, potassium hydroxide, magnesium carbonate
magnesium hydroxide, potassium hydroxide, calcium carbonate
sodium hydroxide, aluminium hydroxide, potassium carbonate

Stomach disturbances are sometimes caused by excessive secretion of


stomach acid. It can be cured through the reduction of the acid by
consuming antacid tablets. Which of the following sets of substances
could likely be the active ingredients in the antacid tablets?

A
D

evaporate, crystallise, filter


evaporate. filter, crystallise
filter, evaporate, crystallise
filter, crystallise

Which one of the following sequences shows the correct order of


processes to obtain crystals of ammonium phosphate after neutralising
phosphoric acid with aqueous ammonia?

A
B
C
D

mass is greater than that of potassium. Which of the following statements

Rubidium 1s a Group I element in the Periodic Table and its relative atomic

Rubidium is a weaker reducing agent than potassium.


Rub1d1um is a soft metal.
Rubidium reacts violently with water.
Rubidium reacts with oxygen to form a basic oxide_

concerning rubidium is incorrect?

A
B

12

35_

36.

37.

38

39.

zinc is harder than tin towards scratching.


zinc is more reactive than tin.
tin reacts more readily with air than zinc.
zinc adheres onto iron better than tin does.

Galvanized iron is more rust resistant than tin-plated iron because


A
B

C
D

Pollutants
carbon dioxide
sulfur dioxide
photochemical smog
nitrogen oxides
Effect
global warming
dizziness and headache
eye irritation
form acid rain

Which of the following about the effects of pollutants is not correct?

The members can be represented by a general formula.


Boiling points increase down the series.
The members have similar chemical properties.
Succeeding members increase by a - CH - unit.

Which one of the following is not a characteristic of a homologous series?

A
B

Liquid P
propane
propyl ethanoate
propan-1 -ol
propan-2-ol

butan-1-ol
water
propanoic acid
butanoic acid

!JIDl!g_Q

Propane reacts with steam at 300C in the pre.s ence of phosphoric acid as
catalyst to produce a colourless liquid p _ On warming liquid P with
acidified potassium manganate (VII) solution, a colourless liquid Q 1s
produced. What is liquid P and Q?

A
B

Name
propyl ethanoate
propyl ethanoate
ethyl propanoate
ethyl propanoate

C2HsCOOC2Hs
CH3COOC3H1
C2HsCOOC2Hs
CH3COOC3H1

Formula

Propanol and ethanoic acid react to form a compound X. What is the


name and formula of compound X?

A
B

13

40.

One that does not contain enzymes


One that contains only organic compounds
One that decomposes quickly after use
One that is not broken down by naturally occurring bacteria

Sewers and rivers are sometimes polluted by 'non-biodegradable'


detergents. which cause large amounts of froth to build up. What is a 'nonbiodegradable' detergent?

A
B

End of paper

white

w hole

red-brown

green

light blue

whole

whole

Colours of Some Common Metal Hydroxides

aluminium hydroxide

calcium hydroxide

copper (II) hydroxide

iron (II) hydroxide

ron (Ill) hydroxide

lead (II) hydroxide

zinc hydroxide

Outram Secondary School


Chemistry
Paper 2

Ail.

1
Emt;

~
!

/
A----/

\
\

/f\
1

Si
.,
\

tI
i
I

11

'-Lo

\ .

1 \\

c!
>

~
PfcgreuOI~ - - - -

Name the substance(s) that occur at position A?

Name the substance(s) that occur at position D?

(1)

(1)

Study the following energy profile diagram for the conversion of carbon
monoxide and oxygen to carbon dioxide. The equation for the reaction is
2CO (g) + 0 2 (g) ~ 2C02 (g)

a.

b.

Which letter labels the arrow that represents the activation energy?
(1)

Which letter labels the arrow that represents the heat of reaction?
[1]

d.

Is this reaction endothermic or exothermic? Explain your answer


wrth reference to the diagram.
(2)

c.

e.

A4.

Sodium metal reacted with oxygen to form compound Q of composition


Na, 59.0%, 0, 41 .0% by mass. Compound Q has a molar mass of 78g.

On dissolving in water, compound Q reacted with water to form a solution


2.

iii.

ii.

i.

Hence, write the chemical equation for the reaction between


0 and water.
(1)

Predict the other croduct in solution R.

Deduce the formula of compound Q _

(1)

(2)

R which contained two products one of which was hydrogen peroxide,


H20

a.

I;).

A sample of Q was dissolved in water. The amount of hydrogen


peroxide produced was determined by titration with a solution
containing cerium(IV) ions. In this reaction , hydrogen peroxide was
converted into oxygen and the cerium{IV) ions became cerium(lll)
ions. It was found that 2 moles of cerium(IV) oxide reacted with 1
mole of hydrogen peroxide.

Calculate the mass of Q used.

[2]

The solution reacted exactly with 18.2cm3 of a 0.102mol/dm3


solution of cerium{IV) oxide (Ce02).
i.

Afr

ii.

solution

I
litmus paper

[:J

State, with reason, the property of hydrogen peroxide in the


titration process.
[2]

I colourles~
I
acidified silver nitrate

[5)

white pr&:1pitate

The diagram below shows some properties and reactions of substances


A, B, C, D and E.

aqueous
ammonia

pr~cipitate

blue-green solution

blue

excess aqueous
ammonia

Identify substances A, B, C, D and E.

black solid
A

a.
A
B

c
D

fJS.

b.

Describe the appearance of substance F.

[2]

A section of a condensation polymer called lactomer is shown below.

CH3 0

11

-CH - C-O-C~-C - O - CH-C-O - C~ - C-0-

II

11

11

Give the repeating unit for this polymer

Draw a circle around the linkage 1n the above polymer.

CH3 0

i.

Name the linkage in this polymer.

[1)
a.

ii.

Name another synthetic polymer w hich contains the same


linkage.
[1]

[1)

(1 )

b.

i.

ii.
Give one use of the synthetic polymer named in c.i.

[1]

[1)

Surgical stitches which are made of lactomer dissolve after a few


weeks because the polymer undergoes a reaction with water and it
is converted to its monomers. The structure of one of the
monomers is given below:

H - 0 - CH - C - 0 - H
I
II
CH3 0
Draw the structure of the other monomer.

A7.

experiment B

heat

=~~

sucrose 7 carbon + water

The figure below shows three experiments in which compounds are being
broken down into simpler subs1ances.

silver bromide 7 silver+ bromine

heat
experiment A

State one of the


decomposition.

11

heat

experiments w hich shows thermal


[1 ]

In which experiment, A , B or C , is a compound being broken


down into its elements?
[1]

experiment C

hydrogencarbonate

,0 ,~

sodium hydrogencarbonate 7 sodium carbonate + carbon dioxide +water

silver bromide

ii

ii.

10

A8.

ti.

c.

A coloured gas is produced in experiment A.

[2]

State and explain at which electrode, 1 or 2, this gas is produced.

~~

Explain why sodium hydrogencarbonate is both an 'acid' and a

a.

Explain why a reaction is observed only after the aluminium is


dipped in concentrated hydrochloric acid.
(2)

Explain why no reaction was observed when aluminium was first


added to the copper(ll) sulfate solution.
[1]

When aluminium was added to a solution of aqueous copper(ll) sulfate, no


reaction was observed. However, when the aluminium was first dipped in
concentrated hydrochloric acid, and then added to the copper(ll) sulfate
solution, reddish brown solid was observed.

b.

II

<.

Write an ionic equation with state symbols for the reaction between
aluminium and copper(ll) sulfate.
[1]

12

SECTION B (30 marks)


Answer all three questions from this section. The last question is in the form of an
either/or and only one of the alternatives should be attempted.

89.

i.

The mineral chalcopyrite, CuFeS2, is smelted in modern


works by heating with air:

Calculate the percentage of copper by mass in a sample of


rock containing 0.50% by mass of chalcopyrite, assuming
this is the only source of copper in the rock.
(2)

810.

a.

The water quality of drinking water can vary. The table gives
information about the concentration of ions in drinking water from
four different locations.

2.7

4.2

ca>+

0.3

0.6

2.4

Na

00

0.4

4.4

2.0

er

0.4

0.2

0.0

2.5

so,"

0.2

0.0

0.1

2.0

No,

6.7

8.2

7.6

7.1

pH

0.35

0.4

Concentration of ion, in mol

0.25

location

In which location would the drinking water not give a white


precipitate with acidified barium nitrate?
[1]

[1 ]
ii.

In which location is the drinking water acidic?

Ir. each case, state whether the rate increases or decreases


and explain your answer in terms of collisions of reactant
particles and the activation energy of the reaction.
(3]

Write the formulae of two compounds that give rise to the


ions found in the drinking water in location D.
[1]

V.

Describe one chemical test for water.

[1]

14

In whi1;h luc<:1tiun i~ the waler most likely lo be polluted by


fertilisers? Explain your answer.
[1]

iti.

The rate of the reaction changes if the CuFeS2 is ground up


and if the temperature is raised.

8CuFeS2 + 2102 -? D 8Cu + 4Fe0 + 2Fe203 + 16S02

ii.

One such copper mineral is chalcopyrite, CuFeS2, which contains


both copper and iron in the +2 oxidation state.

Many copper minerals are found in hydrothermal deposits. These deposits


were formed by crystallisation from very hot solutions which were trapped
underground at high pressures.
a.

t.

Explain why the reaction is considered to be redox.

IV.

Water draining from the waste heaps around a copper mine is often
blue due to the presence of hydrated copper(ll) ions. If this water
nms over iron metal (such as discarded tools or railings) a redox
reaction occurs_
Describe what you wou1d observe as the water runs over the
iron metal.
[2]

ii.

Hence, write an ionic equation for this redox reaction. [1]

[2]

iii.

13

Either
B11.

b.

a.

b.

Ammonia is used to make fertilisers, dyes, explosives and plastics.


Ammonia is manufactured in the Haber process by the reversible
reaction between hydrogen and nitrogen.

;t_

i.

Construct an equation for tr.e mak:ng cf ammonium sulfate


from ammonia.
[1 ]

Name two other essential elements that are needed by


plants.
[2]

Describe the essential conditions needed in the Haber


process.
[2)

Ammonium nitrate, ammonium sulfate and urea are all fertilisers


that can be made from ammonia. The three fertilisers a'I provide
plants with nitrogen.

iii.

Compound X has two isomers.


formula for each isomer of X.

[2]

Draw the full structural

The molecular formula cf an organic compound X is CoHJnO.


i.
Given that tne percentage of carbon by mass cf compound
X is 52.2., find the value of n.
(1)

Ji

Crude oil is a complex mixture of saturated hydrocarbons. One of


the fractions of crude oio contains heptane, C,H16- Briefly describe
how you would prepare propane! from heptane.

the names of processes described;

Your answer should ir,clude the following :


1.

the reagent and reaction conditions, if any, for each process;

[1]

ii

a chemical equation to represent the reaction taking place


for each process described.
[2)

Or
B11.

a.

b.

caesium chloride solution

hydrogen gas

A student used the set-up shown below to prepare hydrogen and


chlorine by e lectrolysis of caesium chloride sol ution. Contrary to the
student's expectation, gas X :nstead of chlorire gas was liberated
at the anode.

GasX

~l\,:i;!~lfo-j

carbon electrode

The experiment was then modified so that hydrogen and


chlorine gases were produced at the cathode and anode
respectively. Suggest how the experimer.t could be modified.
(1]

Suggest an ident ty of gas X . Hence, wnte an ionic equation


for the formation of gas X
[2]

Explain what cculd have happened to the set up of the


electrolytic cell wh ich causes gas X to be collected and not
chlorine gas at the anode
[1)

carbon electrode

II.

a,.

iv.

W hen chlorine gas was collected in the modified experiment,


the volume of gas obtained was significantly smaller than the
theoretical volume. Suggest an exp1anation for this
observallon.
[1)

Na2 CO,

BaCl2

Fo

4- student was provided with four bottles each containing an


aqueous solution of one of the following compounds.
HCI

Unfortunately, the labels had been removed from the bottles. Usmg
ONLY t~.e above solutions. devise a p lan by which you could
identify each compounc.

A student Is given two sample solutions A and B, one of which is


hydrochloric acid solution, while the other is ethanoic acid solution.
Describe a chemical test which will enable yoi. to distinguish
between the two acid solutions. In each case, state the ooservatiOI'
you would expect
[2]

16

[2]

1P

IS

Answ ers
Paper 1

1. A
2. C
3. C
4. C
5. C

6.C
7. A
8. B
9. C
10. c
(i)
(ii)
(iii)
(iv)
(v)

11. c
12.B
13. D
14. A
15.D
16.D
17.A
18. B
19.D
20.A

21. A
22. B
23 B
24. C
25. D

aluminium, iron. magnesium;


fluorine, chlorine:
magnesium and fluorine;
carbon and chlorine or fluorine
chlorine, fluorine:

26. B
27.B
28.B
29. A
30. c

31. A
32. A
33.A
34. A
35. B

36. B
37. D
38. c
39.B
40.D

Drawing consists of magnesium ion circle with 8 crosses and charge 2+;
Fluoride or chloride ions two circles of 7 dots and one cross and one
negative charge.
Excess lead(ll) oxide is added to nitric acid to ensure that all nitric acid
has reacted to form lead(ll) nitrate.
[1 ]
Pb(N0 3)2 + Na,SO.
2NaNOa + PbS04
[1]
Sodium sulphate and sodium nitrate
(2)
Lead(ll) sulph ate is an insoluble salt [1] so it needs to be prepared by
ionic precipitation from two aqueous solutions. Lead( II) oxide is insoluble
so it needs to be changed to a soluble lead compound. [1)
carbon monoxide [1/2m] & oxygen [1/2m]
{chemical formula not acceptable}
carbon dioxide [1m ] {chemical formula not acceptable}

B[1m]

C [1m)

a.

c.

e.

(a}

Exothermic [ 1m]
The reactants are at a higher energy level than the products. [1 m]

b.

a.

(b)
(c)
(d)

(a)

(b)

Paper 2
A1
(a)

A2

A3

A4

= 2.57

Na

4 1/16

41%

= 2.57

59%
59/23

(b)

A5

A6

i.

(ii)

(a)

(b)
(a)

(b

Th e empirical formula

= 78

= Nao

2NaOH + H20 2

Let the molecular formula be (NaO)n


Molecular mass

(23 + 16) n = 78
n= 2

sodium hydroxice

ii..

Na,02 + 2H20 ~

Formula of Q is Na20

iii.

No. of moles of Ce02 used" (18.2/1 000) X 0.102

2 mol Ce02 = 1 mol H20 2

1.86 X 10 _, mol Ce02=0.927 X 10_, mol H102

=1.86 X 10' mol

1 mol Na,02 " 1 mol H20 2


0.927 X 10_, mol Na20 2 0.927 X 10 _, mol H202

= (0.927 X 103 X molar mass of Q) = 0.0724 g

From equation:

Mass of Q

H20 2 is acting as a reducing agent.

CH3

II

[1 ]
[1]
[ 1]
[ 1]
[1]

/m!..:ag~ in 1M

{ ljfor draw;ing er circ;/t: iJTQ1.nd rhi.f


diagram

/ II

Reason: It reduces cerium (IV) ions to cerium (II) ions and itself is
oxidised to oxygen. [1]

A= copper(II) oxide or CuO


B " hydrochloric acid or HC/
t; copper(ll) chloride or CuC/2
O copper(ll) hydroxide or Cu(OH),
E = silver chloride or AgC/
dark blue [1I solution (1I

11

-C-0-

II

- 0 - CH2 - C - 0 - CH - C -

)(i)

{1]

(1]

[1]

[1]

A7

AS.

99

(C)

(d)

(a)

(b)

(i)

(i1)

Made into fibres which are woven into doth. [1]

polyester or terylene {1}

ester linkage [1]

fl]

(ii)

H- 0 -

Exoeriment A

C- 0 - H
II

(1)

Experiment B or C

CH 2 -

(ii)

Gas 1s produced at electrode 1. { 1)


Brormde ions are attracted to the anode. It loses electrons al the anode to
produce the brown bromine gas. (1)
Sodium hydrogencarbonate is an acid as it dissolves m water and get
dissociated to produce H ions which give rise to the acidic property.(1)

A layer of oxide around the aluminium prevents it from reacting with the
copper (II) sulphate

(c)

a.

It is a salt as it is formed when one hydrogen ion of carbonic acid is


replaced by a sodium ion. Thus, sodium hydrogencarbonate produced
consists of a positive metal ion and the anions from the acid. [1]

The concentrated hydrochloric acid reacts with the aluminium oxide to


remove It allowing the aluminium to react with the copper {II) sulfate 1n a
displacement re..clion
3

2Al (s) + 3Cu" (aq) - 2Al (aq) + 3Cu (s)


% of copper = (64 / (64+56+32)] X 0.5%
=0. 174%

CuFeSz is ground up-

(1]

c.
(e)

11.

The rate of reaction will increase.


Reason.
With the solid being ground up. the solid will have a larger surface area of

(1)

( 1)

(1)

[2)

B10

(b)

a.

b.

contact with the reactant particles. Thus, the no. of collision per unit time
between the reacting particles will increase and this give rise to a faster
rate of reaction.
(1]

Temperature raise.

The rate of reacbon w1ll 1ncrease.

Reason.
The partieles will gain k1net1c energy and moves faster.
The number of particles having the activation energy required for the
reaction will mcrase
[ 1J

Blue coloured water turns pale green.

(1J

{1]

Thus, the no of co!hsion per unit time between the reacting particles will
increase and this contributes to an increase in the rate of reaction.
( 1J
(i)

Pinkish brown solid is deposited around the iron metal.

(iii)

Q(1m)

Cu2 + fe '

[1)

[1]

(ii)
Copper (II) ions has been reduced to copper metal. As the oxidation state
of copper decreases from 2 in cu2 ions to 0 in Cu metal.
(1)

Iron metal is oxidised to iro1 (II) ions.


As the oxidation state of iron increases from O on Fe to +2 in Fe,. ions.

(i)

J?.(1m]

Since both oxidation end reduction occurs at the same time, the reaction
is considered to be redox.

(ii)

Na.so. ~. (1/2m for each formula)

Fe2 ' + Cu

(iii)

8 [112), because co1centrai1on of NO, - is the highest (1/2J

Caso. (not acceptable answer: because it is insoluble in water)


(iv)

Add anhydrous copper sulfate [1/2m). Water turns white


anhydrous copper(ll) sulfate blue [1/2m].

or N 1:1 i!n s::1~s:nl1i!I s:!s:ment in fert1hsers (1/2m] which can only be


found 1n location A

(v)

200 atm of pressure. 450

c temperature, iron catalyst


i.

811

or
811

(a)

(b)

(c)

a.

(2m for all 3 correct conditions; -112m for each incorrect condition)
potassium [1m)

phosphorus [1m)

ii.

= 52.2%

[2]

[2)

[11

[2]

[1]

= 2, because the percentage of C in C2 Hs0 = 24146 x 100%

2NH3 + H,SO.' (NH.nso. [1m]

iii.

(i)

(ii)

the isomers are ethan- 1-ol; and CH3 - 0 - CH.;


all covalent bonds must be shown

Cracking and addition/hydration

C3H6 + C4H10;

C rac1< heptane using catalyst at a high temperature;


C7H16 '
Addition reaction between propene and steam,
Phosphoric acid (catalyst), at JOO oC, 70 atmospheres:
C3H6 + H20 '

[2)

C3H70H;

Add silver nitrate solution follow by nitric acid,


HCI gives a white preapitate but not ethanotc acid.

As there is a higher concentration of QH ions than Cl ions present in


the electrolyte, OH- ions will get discharged at the anode to produ::e
oxygen gas. Since Cl' ions are not discharged at the anode, thus no
chlorine gas is produced.
[1]

A verv dilute solution of caesium chlonde has be used as the


electrolyte.

Gas X 1s oxygen

(1]

[1]

(1]

2H,O + 02 + 4e
40H.

Aqueous caesium chloride I concentrated caesium chloride solution


can be used as the electrolyte.

'

iii.

Chlonne gas 1s soluble 1n water

As there Is a higher concentration of er ions than OH- ions in the


electrolyte. c1- ions will get discharged at the anode to produce
chlorine gas
[ 1)

IV

CuSO. can be easily identified as 1t 1s the only blue solubon present [1]

Thus, some of the ch onne gas produced could have been dissolved
in the aqueous solution. This contributes to a lower volume of chlorine
gas obtained.
(1]

Place a portion of the other three colourless solutions into three separate
test tubes. Add the blue solution identified Into the three separate
portions_

m a blue solution. it shows that the

If a white D!OO!lltate is obser.ied


solution tested 1s 8aCI,.
BaCl2 + cuso.' 8aso. + cuc12

[11

If a green precipitate is observed in a colourless solution, it shows thal the


solution tested is Na2 CO,_
Na,SO. + CuSO, ~ Na,SO, + CuC03
[1]

(1]

If there 1s no visible reaction observed in the solution tested, HCI 1s


suspected to be the solution. Further test is required to confirm that 1t 1s
HCI
(1 ]
To a new portion of this solulion, add Na2C03 solution which has been
identified earlier.

C02.

If effervescence is observed and the gas evolved when passed into


limewater produces a white pregDttate. It shows that the gas is
This

Equation 2HCI + Na,co, ~ 2NaCI +co,+ H,O

confirms thot the unkno'Ml solubon tested j 5 HCI

Pasir Ris Secondary School


Chemistry
Paper 1

--

99

91

92

90

melting pomtlC

The melting points of four pure substances P. Q. R and S are given in the table below.

--

"-1

~---

I
!
s

I . : ---

''

B
substance Q

tl substance S

_ .. --"-;:::-=--~eleclron

I
I

!I

_ - o- _
' , __.--nucleus coniaining 7
/ ,...
' ,
___.A
protons and 8 neutrons
...X/
I

,,,.,/

r><:'/ \

u
-...._.,...-

.....

H. Which

An impure sample of a solid X melts over a temperature range of 96'C to 98C From the
information alone, we can infer that solid X is most likely to be
A substance P
C substance R

They all contain one neutron.


Tritium contains the most number of protons.
They all contain the same number of protons.
Tntium contains three neutrons and deuterium two neutrons.

Hydrogen exists as three isotopes. hydrogen H. deuterium ; Hand tritium


statement about these isotopes is correct?
A
B
C
D

I
I

'

.._

\
\

The diagram beiow shows the arrangement of eleclrons in a particle.

t
\
\

''
'

D Ne

What is the symbol for the particle?

A N"

o>-

C F.

10

11

12

13

ii C /2 and Br2
D CH. and NH,

Wl1ich molecules have the same number of protons?


A o, and N?
C CO, and SO,

Ethane has the structure shown below.

H-1-1-H

D 8

B 2

How many of the electrons in a molecule of ethane, C,H,, are not involved in bonding?

A 0

B conducts electricity
D low melting point

carbon aiom
o silioon atom

The diagram shows the structure of a compound of carbon and smcon, (SiC),,.

acts as a lubricant
insoluble in water

What will be a property of (SiC),,?


A
C

1 mol
3 mol

D 4mol

~2mol

A mixture of magnesium chlonde and magnesium sulfate 1s known to contain 3 rnoies of


magnesium ions and 4 moles of ctilonde ions. What is the amount of sulfate ions in this mncture?
A
C

What is the concentration of the acid?

B 0 1 mol/dm'
D 1.25 mo/cim'

2. 0 cm 3 of 1 0 mol/drn 3 potassium hydroxide just neutralises 25.0 cm' of a solution of nitric acid.
A 0 08 mol/dm3
C 1.0 mol/dm3

14

15

16

2.0 mol of graphite, C

B 1.5 mol of propane, C3He


D 0.5 mol of ethanol, C2H~OH

All of the follow1r.g substances produce carbon dioxide on complete combustion Which one will
produce 1.0 mol of carbon dioxide?
A

C 1.5 mol of ethene, C,H,


When sugar (M, = 342) 1s fermented using yeast, the following reaction takes place.

c,,H,,o,, + HiO --> 4C2HsOH + 4CO,

x 24 dm3
r. 1000 x 4 x 24 dm3
342

342x4

B 1000 x 24 dm3

What volume of carbon dioxide, at r.t.p.. would be produced by the complete fermentation of 1 kg
342 x 4

(1000 g) of sugar?
A

1000
C 342 x 24 dm 3
1000 x 4
The Cliagratl' shows the electrolysis of aqueous copper(ll) sulfate using copper electrodes.

copper catho<!e
_,,.- (negative)

_-:-:-:-_--:=::-1

ll-:-:::1Jl_copper11t)
sulfate
solu'11on

(positive}--

copper anode

time

lime

--:C

~mo

time

=:l::.

--:L

Whic~ grap~ shows how the mass of the cathode changes during electrolysis?

=:lL.

17

18

lamp

Two circuits are shown below

poly{etnene)
sugar.

The lamp w 'll light m both circuits if Xis

A
C

metal

dilute
sulfunc
acid

lamp

B sodium chlonde
D zinc.

--

__-_- _ ---- ----_-_-_-_-

Y-

X
a
-- = Y me~a
l

metal Z

Two cells were set up as shown in the diagram. The arrows show the direction of electron flow in
the external circuits

metal

_ _-_- _ _-_- ------------

metalX
silver
silver
coooer
zinc

metal Y
COOl'>f>r
zinc
zmc
COrv>Pr

metal Z
zinc
coooer
silver
silver

Which set of metals would give the electron flow in the d1rect1ons shown?
I

IA
. B

ID

shown below.

t;.H"' -54kJ/mol
H,O(l)

A The reaction is rapid.


B Heat 1s needed to start the reaction.
The OW ions have more energy than the H' ions.

P-->O+R

-40 kJ
+160 kJ

progress of reaction

The figure below represents the energy profile diagram for the following reaction

o The prodvcts contain less e11ergy than the reactants.

What can be deduced from the diagram?

energy

H'(aq) 01-T(aq)

,9, the energy level diagram for the reaction between sodium hydroxide and hydrochloric acid is

2.0

energy

100kJ

O +R

What is the enthalpy change for this reaction?

A -100 kJ
+40 kJ

21

22

23

24

Increasing the temperature and adding a catalyst would change the speed of a reaction. Whicl1
of the following is correct for the effect of these changes on the reaction?

activation energy increases

activation energy does not change

activation energy increases

collision frequency increases

collision frequency increases

activation energy decreases

activation energy does not change

activation energy increases

adding a catalyst

increasing the temperature

Which change will increase the speed of the reaction between 1 mot of eacl1 of the gases, X and

Y?

A a decrease in temperature
a deuease in surface area of the catalyst
a decrease in the volume of the reaction flask
an increase 1n the volume of the reaction flask

The equations of four redox reactions are given below.


2FeCf2 + Cf2 --> 2FeCf3
FeC/2 +Mg--> MgC/2 +Fe
MgBr2 + Cf2 --> MgCf2 + Br2
Mg + Br2 --> MgBr2

B
0

It is only slightly ionised in water.


It reacts only with very reactive metals.

B C/2
D Mg

Based on the four reactions, which of the following is the strongest oxidising agent?

FeC/2

A Br2
C

It is an organic acid.
It is a poor conductor of electricity.

Why is ethanoic acid described as a weak acid?


A
C

25

26

27

28

I
red

four solutions.

10

red

I
yellow

yellow

I 6

The diagram shows the colours of the 1nchcators, methyl orange and methyl red. at different pH
values
pH
colour of methyl
ora nae
colour of methyl
red
~Hof

I ~ I : I : I ~ I

The table shows the

l :ution

XandY

O Zonly

In which solutions will both indicators be yellow?

A WandX
Yand Z

An amphoteric oxide was added separately to a warm dilute solution of an acid and of an alkali.
Whict. entry 1n the table is correct?

salt formed

salt fonned

salt fonned

no reaction

alkali

salt formed

acid

no reaction
no reaction

c
no reaction

Which method of preparation of a pure salt solution reqwres the use of a ptpette and burette?

A BaC/2(aq) + H,so.(aq) -+ BaSO.(s) + 2HCl(aq)


B CuO(s) + 2HCl(aq)-+ CuCl,(aq) + H20(/)
C KOH(aq) + HC/(aq) -+ KC/(aq) + H,O(/)
0 MgC03(s) + HiSO.(aq) -+ MgSO.(aq) + H20(lj + C02(g)
On going down Group VII of the Penod1c Table, the

B II and Ill

I boiling points of elements increases.


II reactivity of the elements 1ncre<1ses
Ill number of shells in the atoms of the elements increases.

I and II

Which of the above statements are correct?


A

29

30

31

32

33

tand Ill

11

1. 11 and Ill

Francium will float on water


Francium will form an acidic oxide.
Francium has low melbng and bo1hng points
Francium reacts less explosively with cold water than caesium

Francium is at the bottom of Group I in the Penod1c Table. Which of the following properties w ill
francium likely to possess?

A
B
C

B zinc
0 silver

In a chemical factory a metal pipe has been used for many years to carry cold water to a storage
tank. Chemical engineers have warned people that the pipe would corrode quickly if 1t was used
to transport acids. Which one of the following metals could the ptpe be made of?

copper

A magnesium
C

X, calcium, nickel
calcium, X, nici<.el

D nickel, X. calcium

B calcium, rnci<el. X

The element X hberates hydrogen from dilute hydrochloric aad although it does not react With
cold water. When a piece of Xis placed into rnckel(ll) chloride solution, the green solution turns
colourless and a grey deposit is fonned. Which of the following sets gives the order of
decreasing reactivity of X, nickel and calcium?

A
C

,/

alum1n1um

,/

,/

COOOl'!r

,/

,/

,/

"

silver

"

,/

"

"

sodium

"

"
,/

"
"

Key
./ =can be obtained
" = cannot be obtained

Some metals can be obtained by the reduction of their oxides with hydrogen. Which line of the
table is correct?

A
B
0

Which reaction takes place in the blast furnace?

A 3Fe + 4H,O-+ Fe,o, + 4H2


B Si02 + Cao -+ CaSiO,
Si02 + 2Na0H -+ Na,5103 + H,O
FeCr20, + 4C -+ Fe + 2Cr + 4CO
0

34

35

36

37

2NO(g) + 0 2 (g)
colourless

12
Nitrogen dioxide, N02, is a dark brown gas that decomposes as shown by the equilibrium
equation.
2N02(g) ..:
dark brown

l~L_--------inlet for gas

The diagram shows a glass flask containing a mixture of the three gases. The mixture is pale
brown.

l~r-

B
D

It turns colourless.
It becomes a paler brown.

More oxygen is forced into the flask. What COiour change is seen in the mlXlure?

C It becomes darker brown

A There 1s no change.

Cl

In the past. CFC compounds were used as aerosol propellants. The structure of one CFC
compound is shown.

I I
F-C-C-H
I I
F

B chlorine
D hydrogen

Which element in this compound can cause a depletion of ozone?


A carbon
C fluorine

B
D

have the same empirical formula


are from the same homologous senes

A group of different organic compounds undergo similar chemical reactions. This statement is
true for compounds that
A undergo precipitation.
C burn to give carbon dioxide and water.

A marganne 1s described as 'high in polyunsaturates'. What does this type of margarine contain?

-C-N-

linkages

~ long chain alkane molecules


molecules containing many C=C bonds
\; many al kene molecules Joined by addition polymerisation

D polymer molecules with

H-

I
I

13

Br

Br

Br

C-

Br

I
I

I
I

I
CI
H

I
CI
H

I
CI
H

H
I
I

H-c-c-c-c-H

I
I

What is the formula of the hydrocarbon X?

c
H

I I I I
H - C - C = C -C -H
I
I

';! ';!

OH

;o
'

;o

t
I
I
H
H
H

H-c-c-c-c

H""

I
I

c=c-c-c-H

H/

""H

H
H
/H
H""
C=C-C=C

H/

H- C

'

'

H H

0-C-C-H
'
I
H H

H H H

0-C-C-C-H
I
I
I
H HH

;o

HC-C

The ester methyl propanoate has the molecular formula c.H60 ,. Which compound is not an
isomer of methyl propanoate?

I
I

38 A hydrocarbon X reacts in darkness with bromine to form the compound with the formula shown.

c
';!

H H

H-C-C-C
H H
I
I
'
I
I
H
H
0-C-C- H

40

14

and

Ho-C)-oH

Terylene (a polyester} is made by condensation polymerisation of the two monomers shown

H-o-c_ r - t -c- o-H


11.._____Fn
0

_(r--c=h-o-C>ol
lo
o
Jn

What is the repeat unit of the polymer?

frc::r-rc>-oJ

' f0-cJx-o-e:>ol
c

.fr-C}A-c>+

Paper 1

B
D

c
c
D
c

1
2
3
4
5

6
7

9
10
11
12
13
14
115
16

Pasir Ris Secondary School


Preliminary Exam 2011
Chemistry 5072
Secondary 4 Express

D
D

D
B

c
c
c
c

25 D
26 8
27
28
29
30 8
31
32 B

Marking Scheme

17
18
19
20
21

c
c
A
A

24

22 c
23 B
D
D
D

33
34
35
36
37
38
39
40

D
B

Pasir Ris Secondary School


Chemistry
Paper 2

4
(c) Draw the energy profile diagram for the combustion of methane in the space below. Label on
your diagram clearly, the activation energy, E.. and enthalpy change for the reaction, 6H.
energy

progress cf reaction

(3)
[Total 9)

:;

Petrol and diesel are liquid fuels used in the internal ccmbustion engines of motor cars and
lorries.

.. . " " ' " '

""''"

' " ' " ' " " ' " " ' "' ''" ' ' "' ' ' "' ' '' [1)

(a) Name the process by which petrol and diesel are obtained from petroleum.

'"" ""''"'

carbon
monoxide

59

29

oxides of
nitrogen

38

09

sulfur
dioxide

17

25

volatile organic compounds


e .g unburnt hydrocarbons

0.6

particulates

mass of pollutant/g

10

236

fuel

(b) The exhaust gas from an internal combustion engine contains pollutants. The table below
shows the mass of pollutants formed when one kilogram of each fuel is burnt.

petrol

18 6
diesel

(i) Which pollutant 1s a poisonous gas formed by the incomplete combustion of the fuels?

(1)

'""' [1 ]

(ii) Which fuel contributes the most towards acid ram for each kilogram burnt? Explain your
answer.

(i ii) Describe the condition in the internal combustion engine that allows oxides of nitrogen to
form.

"'' ""'" .. (1)

(iv) Many cars are fitted w ith catalytic converters. In the catalytic converter carbon monoxide
and nitrogen dioxide undergo a redox reaction to form two harmless gases. Construct the
chemical equation for this reaction and explain w hy it is a redox reaction.

[3!

[Total- 7]

"
Nz + 3H2 = 2NH3

(2]

. .. [2)

Ammonia is manufactured by the Haber process. The equation for the reaction is represented
below.

(a) What are the essential conditions for making ammonia from nitrogen and hydrogen?

(b) 3 tonnes of hydrogen when reacted produces only 2 tonnes of ammonia. Calculate the
percentage yield.

{c) A fertiliser factory uses ammonia to make nitrogenous fertilisers such as potassium nitrate,
KN0:1. and ammonium sulfate. {NH.),SO,.
(I) Why do plants need nitrogen?

........ ....... ... . ...... .... .. ....... ........ . [1]

............ (1)

(ii) Construct a chemical equation to show how ammonium sulfate may be prepared from
ammonia.

{Iii) Name the melhOd to prepare soluble salts such as ammonium sulfate.

. ...... ..... [1]

(d) (i) A water supply 1s suspected to be contaminated with ammonium sulfate Describe a
chemical test to determine the presence of sulfate ions 1n the water.

.......... [2]

.. .... [1]

... .... [2]

(ii) W nte the ionic equation w ith state symbols for the reaction descnbed in the chemical test
above.

(I) What is the purpose of adding calcium hydroxide to the soil?

(e) Calcium hydroxide is sometimes added to the soil.

(ii) Explain why the add1t1on of calcium hydroxide can cause a loss of nitrogen

....... [1]

[Total: 13)

'5

8
The hydrocarbon, W, has the structural formula as shown below.

/CH"-....
CH,
CH,

CH,--CH,

It is a member of the homologous series known as cycloalkanes.

(i) the hydrocarbon W ;

[1]

[1]

(a) Write the molecular formulae of:

(ii) the next member of this homologous series.


(b) A n isomer of W is an alkene. Draw the full structural formula of this compound, X.

[1]
(c) Both isomers Wand X react w ith aqueous bromine. The reaction of W is slow while that of X
is rapid.
(i) Describe what you would observe when X reacts with aqueous bromine.

. .... ....... [1]

Organic compound produced from the


reaction of X with aqueous bromine

(ii) For each reaction, draw the structural formula of one organic compound produced from
the reaction.
Organic compound produced from the
reaction of W w ith aqueous bromine

[2]

(d) The isomer X reacts with steam in a reaction known as hydration.

(i) What are the conditions for this reaction?

>-----~

::(ff:":":":l

HN03(aq)

Pt electrode

.. . [2]

[1]

[Total: 9]

(ii) Draw the full structural formula of the organic compound produced in this reaction.

____ ,_____ -

[:~:-:-~-:-:-:-:-:-:-:-:-:-:~=-=-;:

NaN03(aq) salt bridge

~------<

The diagram below shows the set-up of the zinc-hydrogen cell.

Zn electrode

--

Zn(N03)2(aq)

Zinc electrode is the negative electrode while platinum electrode is the positive electrode. The
sodium nitrate salt bridge completes the circuit by allowing the migration of ions. The reading on
the voltmeter shows 0. 76 V.

[1]

. ... [1]

.. ... ... ... ... ...... ... ... .. ....... . .. ...... ...... ..... [2]

(a) Write the ionic equations for the reactions at each electrode.
Negative electrode:
Positive electrode:

(b) Deduce what you would observe at the positive electrode.

(c) Draw on the diagram above, the direction of electron flow in the external circuit.

10
(d) The zinc electrode is replaced with a silver electrode. The silver electrode is dipped into a
solution of silver nitrate. Hydrogen gas is pumped in through the inlet labelled X The reading
on the voltmeter shows -0.80 V. Identify the negative and positive e lectrodes and write the
ionic equations for the reactions at each electrode.
Negative electrode:
tonic equation for reacbon at the negative electrode

Positive electrode:
tonic equation for reaction at the positive electrode:
......... [3]

(Total: 7]

11

Section B

Answer all three questions from this section.

The last question is in the form either/or and only one of the alternatives should be attempted.

1. Dl"ag cursor a r'ound plot area

l!_..a,

..

;.'.~ ~

~ -j} . .L :...

-'J

~-..:......-.~J...::.a-r,_ .1

il. 1~-Up'l

-_.
lJ.3 .~ j:c"'
SZ>1;1'o:i~~b.1 I

, '"':

WebElements

The figure below is a screenshot taken from the website "WebElements". It shows how the
atormc rad11 of the different elements in the Penodic Table vary The size of the atoms shown is
proportional to the actual size of the atoms.

; :,!

At omic radius.

:i

to t ho w Informat ion.
-~~:1;;~j
l-: 2 .. cUck on eltim ent within plot
0 a r e a to go to that element.

~Ol

. .
....n.,;::

~
- iO~Q~O'ct'.O'O
:o:o;o1 oti;;.;~-ol~t~ 1 ~! ol
':.J---'_~ - .J:!!!J--~----:-~-;.:;J~~~--'

~-~

~"~3.~;~
oi:::;~~J;~~gr~t6'. ~;

J ............ :....1 .. ~...-. .1-;.__,-..,'-~-........,,..., '."

(1)

_i1~:11lu.>.Jt.t,.___.
r::,,li:::J:~::1i~<1':;.
!
,............. ----~ .;_~.J.;..1...

~~~~~~=-~-~-~--'-"~

L 11u' t:::f;n,n: t

{i) down a group;

(1)

!ill Describe the trend in the atomic radii of the elements

{ii) across a period

(Ii) Explain why both sodium and chlorine belong to the same period.

(ll Write down their electronic configurations.

[3]

(1]

[1 ]

(b) Sodium and chlorine belong to the same period.

(c;) Suggest why the trend as descnbed in (a)(i i) 1s observed across a period

[1]

(d) On going down Group IV, the elements change from non-metallic to metallic character. Make
use of the data provided In the figure above to explain why metallic character increases on
going down the group.
[2]

extreme conditions to form xenon tetrafluoride, XeF..

(e } Group 0 elements are known to be unreactive However, xenon reacts with fluorine um~er

(I) Explain why Group 0 elements are unreachve.

(ii) Draw a 'dot-and-cross' diagram to show the bonding in xenon tetrafluoride You need to
draw only the valence electrons.
[2]

[Total: 12]

II
(.0

12

~
I

_J

20

II
0

The diagram below shows a section of the polymer chain of nylon.


H

I
II
0

- N - C - (CH ) - C - N - (CH ) - N - C - (CM )4

Nylon is a polyamide and is formed through the condensation polymerisation of its monomers, a
diamine and a dicarboxyhc acid.

EITHER

13

Two experiments were set up as shown below.

~'.~'.~'.

::~:~;~

...
::::-:-:-:-:::::::

zincmetatmJ
'"'~
aqueous copper(ll)
sulfate

Beaker A

Beaker B

-.-

dilute hydrochloric
acid

zincmetal

(ilf Describe what you would observe in each beaker after some time.

(1)

la) Wrth reference to the diagram above, deduce the structural formulae of the two monomers of
~on
~
th) Name the small mOlecule that is lost during o'ymerisation to form nylon.

{u~

(31

(2)

[Total: 8)

Food cans made of steel are prevented from rusting by a thin coating of tin. Explain how
the layer of tin protects the iron from rusting.
[ 1J

(c) Ii> Zinc is commonly used to galvanise iron to prevent it from rusting. Explain how the layer
of zinc protects the iron from rusting
(2)

beakers.

ib) With the help of appropnate equallon(s), explain why zinc has undergone 0X1dation in both

[1}

(cl F-rote1n is a naturally occurring polymer. It is formed through the condensation polymerisation
of its monomers, the ammo acids. The diagram below shows a secilon of the polymer chain
of protein

COOH

CH2

-N-CH-C-N-CH-C'
1

(Ctti)4

I
NH2
(i) Explain why the protein chain shown above is a polyamide.

(Ii) W ith reference to the diagram above, deduce t11e structL:ral formulae of the two amino
acids that make up the protein chair.
(2)

[Total: ~OJ

Id) lnsuhn is a protein used to treat people suffering from diabetes. In an experiment, insulin was
broken down into a mixture of amino acics. A chromatogram was set up with a sample of the
mixture and samples of the amino acids proline and lysine as references. Explain, with the
aid of a diagram, how chromatography could be used to show that proline and lysine are
present in the mixture.
[4}

OR

14

Small lumps of calcium carbonate, present in excess, were added to dilute hydrochlonc ac;1d in a
flask as shown in the diagram below.

flask

+--- - cotton wool

dilute hydrochl oric acid

' yr-~ ----. calcium carbonate


balance

(1)

The balance reading was noted as soon as the lumps were added and monitored for about five
minutes.
(a) What is the purpose of the cotton wool?

(b) Sketch a graph to show how the total mass of the flask and its contents vary with time until
the reaction is completed. Label this graph as (b).
[1]
(c} The experiment was repeated changing only the concentration of the hydrochlonc acid to
one of higher concentrabon. Sketch, on the same axes as in (b) above, another graph to
show how the total mass of the flask and its contents vary with time Label this graph as (c).
[1)

[3)

{dl Explain, using the collision theory, why powdered calcium carbonate would react faster than
lumps of calcium carbonate in this experimenl
(2)
(e) Give two types of reaction. other than that of an acid on a carbonate, which lead to the
formation of carbon dioxide. Write an equation for each of your chosen reactions.

[Total: 8)

15

Paper2
Section A
1
a
b

ci
cil

Pasir Ris Secondary School


Preliminary Exam 2011
Chemistry 5072
Secondary 4 Express
Marking Scheme

Total= 5
1
liquid state
2
Magnesium oxide is able to conduct electricity due to the presence of
mobile ions [1) while copper is able to conduct electricity due to the
presence of mobile electrons. [11
1
substance B
1
substance D
Total= 9
3
% of carbon in methane - 12 x 100%
16
= 75.0%
% of carbon in ethanol= 2 x 12 x 100%
46
= 52.2%
% of carbcn in octane= 8 x 12 x 100%
114
= 84.2%
Since ethanol contains the lowest percentage of carbon by mass, it would
produce the least carbon dioxide when burnt.
Or
CH, 202 ~ C02 + 2H,O
16 g of methane produces 1 mol of carbon dioxide
:. 1 g of methane produces 0.0625 mol of carbon dioxide

C,H5 0H + 302 ~ 2C02 3H20


46 g of ethanol produces 2 mol of carbon dioxide
:. 1 g of ethanol produces 0.0435 mol of carbon dioxide
2C8 H 18 + 2502 ~ 16C02 18H20
228 g of octane produces 16 mol of carbon dioxide
:. 1 g of octane produces 0.0702 mol of carbon dioxide
Thus, ethanol produces the least carbon dioxide when burnt
Method used - {1]
Answers (calcvlated valves are all correct) - [2]
Conclvsion (ethanol prodvces the least carbon dioxide) must be written
otherwise deduct - 111

bi
bii

E,

~~

k~L~

C02 2H20

progress of reaction

Methane gas is trapped under pressure in the ice structure. I A gas can
be compressed easily under hiqh oressure.
Amount of energy= 160 x 890 (1]
24000
[11
= 5.93 kJ
~

energy

CH. 20,
t.H = -8eo

temperature: 450C
pressure: 250 atm
catalyst: iron

% Yield of NH3 2 x 100% 11.8% [1 )


17
To make Proteins for arowth
2NH3 + HoSO. ~ I NH.i,so.

3 conditions correct - [2]


1-2 conditions correct - -11]
3 moles of H2 "' 2 moles of NH3
6 tonnes of H2 " 34 tonnes of N H3
:.Mass of NH 3 produced from 3 tonnes of H 2 = 3 x 34

=17 tonnes [1)

1
1

Total= 13
2

Eqvation for the reaction is correct - (1J


Energy profile mvst show exothermic reaction before awarding the next 2
marks
E, correctly shown and labelled - [1]
dH correctly shown and labelled (valve not insisted)- r11
Total = 7
3
1
fractional distillation
a
1
bl
carbon monoxide
1
bii Diesel, because it produces more oxides of nitrogen and sulfur dioxide
comoared to oetroL
1
bi ii hiah temperature
3
biv 4CO 2N02 ~ 4C02 + N2 [1)
Carbon monoxide is oxidised as it gains oxygen atoms to form carbon
dioxide. [1)
Nitrogen dioxide is reduced as it loses oxygen atoms to form nitrogen. [1)
As both oxidation and reduction occur concurrently, the reaction is a
redox reaction.
4

ci
cii

1
2

titration
To a sample of water, add dilute nitric acid (hydrochloric acid) fellowed by
aqueous barium nitrate (barium chloride). (1]
Presence of sulfate ions is indicated by the formation of a white
orecioitate. r11
Ba'(aq) + SO/-(aq) -> BaSO.(s)

1
1

ciii
di

dil
Balanced equation - {1]
Correct stats symbols - {1/
To neutralise the aciditv of the soil
Calcium hydroxide reacts with ammonium salts in the sOil producing

Thus nltroaen in the form of ammonia is lost

!!!l!!!!Q!!@ which escapes into the air.

C,H,o
r-H.
H

H-C-C-C-C=C-H

I
H H H
The orange/reddish-brown aqueous bromine turns colourless.

CHBr

Must mention oriqinal colour.


Orgarnc compound produced from the reaction of W with aqueous
bromine
/

' CH,
I

CHi--CH

Structural fonnula that shows any one or more than one hydrogen atom
correctly substituted is acceptable. [1]
Organic compound produced from the reaction of X w ith aqueous
bromine
H
H
H
H
H
I
I
I
I
I
H-c- c- c-c-c-H
I
I
I
I
H
H
Br
Br

Answer is marked based on the structural formula of the alkene drawn in


(/!)above (1)

Total= 9
1
1
1

ei
eii
5
ai
aii

Ci
ell

di

dii

6
a
b

c
d

temperature: 300C
pressure: 60 atm
catalyst: phosphoric acid

H H
H
I
I
I
c-c-c-H
I
I
I
0
H
H

H H
H
I
I
I
c-c-c-o--H
I
I
I
H
H
H

3 conditions correct - [2]


1-2 conditions correct-(1/
H
H
I
I
H-c- cI
I
H
H

or
H
H
I
I
H-c- cI
H

1
3

=12
1
1
1

1
1
3

Total= 7
2

H
Answer is marked based on the structural formula of the alkene drawn in
(b) above.

Negative electrode: Zn-+ Zn''+ 2e- [1)


Positive electrode: 2H' + 2e -+ H, [11
Bubbles of a gas
Direction is clockwise in the external circuit.
Negative electrode: Platinum/hydrogen electrode (Y,)
Ionic equation for reaction at the negative electrode: H2 -+ 2H + 2e [1]

Positive electrode: Silver electrode (Y,)


Ionic eauation for reaction at the oositive electrode: Aa + e -> Ag [1)

Section B

7
ai
aii
bi
bil

Total
The atomic radii of the elements increase down a arouo.
The atomic radii decrease across a oeriod.
sodium: 2,8, 1 (Y.)
chlorine: 2 8 7 IY.I
The have the same number of occuoied electron shells.
Moving across a period, the number of protons increases.
!hus, the force of attraction of the nudeus towards the electrons [1)
increases.
Although the number of electrons m the atom increases on going across a
penod, the number of OCCU!!!ed electron shell does not. (1) The additional
electrons occupy the same shell
The increased forces of attraction from the nudeus draws the electrons
doser to the nucleus there'2Y reduetng the radius of the atom. [1)

--=-

I On going down a group, the radiu s of the ato01il1Cfeases. The valence


electrons are thus further away from the nucleus [ 1)
This reduces the forces of attraction of the nucleus towards the
}
electrons. The valence electrons are thus easier to be lost which is [1]
demonstrated as metallic character_ _ __ __ __ _ __

Cl

(1)

x - electrons of fluorine

o - electrons of xenon

Legend:

I Their atoms have fully occupied electron shells

(1)

I
- N

Electrons mvo/ved in bonding- {1}

Other valence electrons - (1

H - 0 - C - (CH2J, - C - 0 - H
H
I

I
I
H - N - (CH,)6 - N - H

I water

It contains the amide link

(1)

(1)

Total= 10
2

ei
eii

8
a

b
ci

cli
H-N-CH-C-0-H

<TH2>,
II

NH2

H-N-CH-C-0-H

CH2
COOH

vstarthne

Diagram must show all 3


spots - (1]

-+- chromatography

paper

: mixture of amino acids as shown in the figure below.

T1) Prepare a chromatography paper spotted with proline, lysine and the

t t -f

prohne lyStne mncture

Diagram must show


correct set-up where
solvent level is below
start line - {1]

2) Place the chromatography paper in the set-up as shown below.


~ lid

paper

H--. chromatography

-!--+ solvent

3) Ensure that the solvent level os below the start line.


4) Allow the solvent to rise up the paper to separate the }
mixture of amino acids.
5) Remove the paper when the solvent has nsen close to
the top of the paper
{1]
6) Allow the paper to dry before spraying 11 with a locating
agent.
7) If the spots produced by the m ixture show similar positions as the
spots produced by both prohne and lysine, this shows that insu1in
contains both l'fOhne and lysine Qorrect conclusion drawn - (1)

9EITHER
a
Beaker A :
The colour of the solution turns lighter blue/colourless. (1]
Reddish-brown solid is formed at the bottom of the beaker. (1)

mass of
flask and rts
contents

(c)

(1)

b) (1)

time

Gradient of graph (c) must be steeper and its position must below graph

.JQl

Total = 8

Powdered ealcium carbonate provides a greater total surface area for the
particles of acid to collide into. [1]
This leads to more frequent collisions between the reactant particles and
consequently higher probabilit~ of effective coll isions occurring. [1]
Thus a faster speed of reaction results.
Combustion
Equabon: CH, + 202 -> co, + 2H20
For each reaction
Type of reaction - [Y,]
Equations - (1)

Beakers:
Effervescence is seen. [1I
In both beakers, zinc loses electrons to form Zn" ions. [1]
2
b
Zn -+ Zn2 + 2e 111
ci
Zinc, bemg more reactive than iron, corrodes in place of iron. [1] Zmc
2
takes on the role of a sacrificial metal. 111
1cil Tin forms an unreacllve coating around the iron to prevent air and water
from reach1no the iron to cause it to rust I 11
Total= 8
90R
to prevent acid spray
1
a
2
b I
c

Redox reaction
Equation. 2Cu0 + C -> 2Cu + C02
Fermentation
Equation. C 6H1,06 -> 2C,Hs0H + 2C02
Thermal decomposition
Equabon: CaC03 -> CaO + CO,

ReactlOfls 01ven must be of different tunes.

Name

I
Secondary Four Express

PRELIMINARY EXAMINATION 2011

PEI HWA SECONDARY SCHOOL

Index Number

L, l
CHEMISTRY

Optical Answer Sheet (OTAS)

Paper 1 Multiple Choice

Additional Materials :
READ THESE INSTRUCTIONS FIRST

5072/01
26 August 2011
1 hour

There are 40 questions in this paper. Answer ALL questions. For each question,
there are four possible answers A, B, C and D.
Choose the one you consider correct and record your choice in soft pencil on the
separate OTAS provided.

Write your name, class and index number in the spaces above and on the OTAS.
Shade your index nunber in the boxes provided.

This booklet consists of only questions for MCQs.

Do not open this booklet until you are told to do so.

1.

2.

3.

Each correct answer will score one mark. A marl< will not be dP.ducted for a wrong
answer. Any rough working should be done on this booklet
Hand in the OTAS separately from the question booklet.
A copy cf the Periodic Table is provided on page 20.

This question paper consists of 20 printed pages, inclusive of this cover page.

sodium chloride

iron(ll) iodide

copper(ll) chloride

calcium iodide

Substance X ~issolves in water to form a colourless sclut1on H.is solution reacts with
aqueous leaa(ll) nitrate in the presence of dilute nitric acid to give a yellow precipitate.
What is substance X?

boiling point I

- 78

- 198

- 186

nitrogen

- 183

argon

oxygen

carbon dioxide

gas

Tl>e boiling points of various gases found in the air are shown below.

I
A

nitrogen

argon

oxygen

carbon dioxide

If the air is cooled, the first substance to condense is water.


If \he temperature is lowered further, what is the next substance to condense?

Diagrams 1, 2 and 3 represent the three states of matter.

-~ l2
A

2 and 3

1and2

1 only

For which states can diffusion be demonstrated by using simple laboratory apparatus?

1, 2 and 3

The following measurements are made on a sample of pure water:


its boiling point, b.p.
its freezing point, f.p.
its pH
Sodium chloride is now dissolved in the water and the measurements repeated.
How do these values change?

increases

f.p.
decreases
increases

increases

pH

increases ,

unchanged

unchanged

b.p.
decreases

decreases ,

increases

increases

increases,-

c
D

Which statement about the substance formed when a given mass of an element burns in
excess oxygen is always correct?

soluble in water.

greater in mass than the element.

denser than the element.

The substance formed is

white in colour.

C
D

and

1
;

C. It was found that the relative atomic mass of carbon in the molecule is 12.2.

A giant molecule made up of a large amount of carbon, mainly consisting of isotopes

'8

proton
number

24

24

nucleon
number

12

12

number of
prot ons

12

10

12

The table shows the structure of different atoms and ions.


particle

12
9

19

19

10

number of
electrons
Mg

number of
neutrons
Mg2

12

10

10

10

10

10

= electron

key

F-

10
10

10

What are the values of W, X, Y and Z?

12

12

12
0

The electronic structures of atoms P and Q are shown.

P02

P and Q react to form an ionic compound.

P20

';c

P2~

What is the formula of this compound?

Ps0 2

C?

1 :4

A 4:1

1
:

3: 1

3: 4

What is the ratio of 1 ~ C to

10

11

12

The diagram below shows the arrangement of electrons in the outer shells of the atoms
in the compound XY 2.

13

The reaction between aqueous copper(ll) sulfate and aqueous sodium carbonate can be
represented by the equation

blue

colourless

green

X is likely to be a sulfur atom.

X is likely to be a calcium atom.

a green precipitate and a blue solution

a green precipitate and a colourless solution

a green precipitate only

a colourless solution only

ammonium sulfate

2 9 of hydrogen

56

colourless

CuSO.(aq) + Na2C03(aq) ___. CuC03(s) + Na2SO,(aq)

Y is likely to be a carbon atom.

In an experiment, 8.0 cm3 of 1.0 moVdm 3 aqueous copper(ll) sulfate was mixed with 6.0
3
3
cm of 1.0 mol/dm aqueous sodium carbonate in a beaker. What did the beaker contain
when the reaction was completed?

Which of the following statements is correct?

carbon dioxide

14 g of nitrogen

96

L
--1

--j

1 and 3 only

1and2 only

2 and 3 only

1, 2 and 3

~~H}, flOWdtlr

H;S0.<04)

The mixtures are stirred.


Which test-tubes then contain Cu 2-(aq) ions?

H,SO,{aql

~~wx.ler

H:-SO.(aq) -

~ r:uco, pa'M1ar
~

The diagrams show three experiments using dilute sulfuric acid. Three different powders are
added to the acid.

Y is likely to be a hydrogen atom.


14

sodium sulfate

ethyl ethanoate

Which compound has both ionic and covalent bonds?

44 g of carbon dioxide

32 g of oxygen

Which gas contains the same number of molecules as 9 g of water?

An element Z forms a chloride, ZC'3, which contains 65.5% of chlorine by mass. What
is the relative atomic mass of Z?

52

A 27

15

16

calcium
soluble

slightly soluble

hydroxide

insoluble

soluble

insoluble

carbonate

soluble

soluble

slightly soluble

sulfate

The table gives information about the solubilities of the hydroxides, carbonates and
sulfates of calcium, sodium and zinc.

sodium
insoluble

zinc

Shake aqueous zinc sulfate with aqueous sodium carbonate.


Shake aqueous zinc sulfate with solid calcium hydroxide and bubble in carbon
dioxide.
Shake solid zinc hydroxide with aqueous sodium hydroxide and bubble in carbon
dioxide.
Shake solid zinc sulfate and solid calcium carbonate with water.

What is the best way of making zinc carbonate?

A
B
C
D

aluminium sulfate

NaOH(aq)

colourless solution

excess
HN03(aq)

compound Y

The scheme shows some reactions of a compound Y.

calcium carbonate

white precipitate
insoluble in excess NaOH(aq)

6+

zinc carbonate

copper(ll) carbonate

What could the compound Y be?

17

1il

1'9

ZnO + 2HN03-. Zn(N03)i + H20

ZnC03 + 2HN03 --> Zn(N03)i + H20 + C02

Mg

Fe

ethanol
C2H50H

(H)

reaction of oxide of X
w ith carbon

sodium ethanoate
CH3C02Na

reaction of X with dilute


hydrochloric acid

no reaction

The following equations represent reactions of nitric acid. In which reaction does nitric
acid act as an oxidising agent?

3Cu + 8HN03 -

Cu(OHh + 2HN03 - Cu(N03)2 + 2H20

3Cu(N03)2 + 2NO + 4H20

XandY

Which of the reactions X, Y and Z involve oxidation?

ethyl ethanoate
CH3C02C2H5

Yonly

A X only

D YandZ

Na

The position of metal X in the reactivity series is shown.


K

hydrogen formed

no reaction

oxide reduced

Which statements about X and its oxide are correct?

no reaction

hydrogen formed

oxide reduced

no reaction

B
D

20

21

steel

copper

zinc

screws

magnesium

steel

steel

strips

wood

An old railway carriage 1s being restored. Metal stnps are secured on to the outside of
the wooden carriage by means of screws. After a few weeks of exposure to the wind and
rain, the screws are heavily corroded bt:t the metal strips are not.
metal
strip
metal
screw

B
zinc

Which two metals would give this result?

c
steel

Aluminium is extracted from purified bauxite (Al203) by electrolysis but iron is extracted
from haematite by reduction with coke

Reduction witt- coke gives a purer product than electrolysis.

Reduction with coke is cheaper than electrolysis.

Iron 1s less reactive than aluminium.

Haematite needs to be purified but bauxite does not.

Why is iron not extracted by electrolysis?

22

23

-+-

t=-=-.=.-:-:

IR

Qonly

P only

QandS

P and R

concentrated
hydrochloric acid

The following electrolysis circuit 1s set up, using inert electrodes P, Q , Rand S.

I.

;::~u)~
moJ::
A

At which of the electrodes is a Group VII element produced?

cartlOn

concentrated

_
negatr1e
.------- electrode

The diagram shows the electrolysis of concentrated aqueous sodium chloride.

positive

electrode ---..~...._

-:i

chlond& and
Universal lndic-ator

-~4-- aqutXlllS sodJurn

red

red

blue/purple

colQ!!r at 12ositive ~ ectrode

colourless

blue/purple

red

Colo11r at n~ative electrode

ateach electrode after five minutes?


B

colourless

What is the colour of the Universal Indicator

blue/purple

10

24

25

I\

I
-T
-

The diagram below shows the set-up of a simple cell.

copper electrode - -

The electrolyte fades in colour.

The electrolyte becomes increasingly acidic.

The copper rod decreases in size.

A reddish-brown solid is formed on the zinc rod .

zinc electrode

aqueous
copper(ll) sulfate
Which of the following is observed as the reaction progresses?

--

II

S has a lower melting point than P.

Q has more metallic character than P.

P is more reactive than S.

Group
Ill

IV

VI

--

VII

Part of the Periodic Table is shown. The letters are not the symbols of the elements.
Penod

T is more reactive than R.

Which statement 1s correct about the elements shown?

11

26

27

When pink crystals of cobalt(ll) chloride are heated, steam is given off and the colour of
the solid changes to blue.

CoCl2.6H20 . _ CoCl2 + 6H20

colour
changes to pink

increases

temperature
decreases

What happens when water is added to the blue solid?

changes to pink

decreases

increases

remains blue

remains blue

The table compares the strengths of the bonds for reactions of the type below.

weak

strong

strong

bonds in X2

weak

weak

strong

strong

oonds in Y2

weak

strong

weak

strong

bonds in XY

X2 + Y2- 2XY

weak

Which reaction is most exothermic?

12

28

zg

In experiment 1, an excess of finely powdered marble 1s added to 20 cm of dilute


hydochloric acid.
In e)(periment 2, carried out under the same cond itions of temperature and pressure,
3
an excess of marble chips is added to 20 cm of dilute hydrochloric acid of the same
concentration.

time

The total volumes of carbon dioxide given off are determined at intervals and plotted
against time.

total
volume

of C01

expenment 1

z
x

experiment 2

Which pair of curves is obtained in the two experiments?

A
y

x
x

c
y

B
D

2NH3(g)

Ammonia is manufactured by the Haber process using an iron catalyst:


N2(g) + 3H2(g)

3G

The diagrams show two experiments.

~~ ~

no change

hP..at

both pieces bleached

no change

experiment 2
both pieces bleached

sut~runc ;icld

ddu;~

fllnmol'!lu~1 Chl0r1da -~

<:qUOOIJS

clue""~

'-red

expenril"n! 2

!l!mU pllPt'r

ex::ier11nem I

'"""., ., Ll "'~1--..:

_t:.,:~

[--.~

blue"~ <f<.. ;oo


uhnus paper

aqueous

heat

G'

ftnwnoruum chlofida .... L-_-.

aqueous

SOdoum hydroxide

What happens to the pieces of litmus paper?


experiment 1
red
blue ~

blue~

blue ,

red ~

blue -

red

red ~

reaction 1:

2H2S01 + Ca3(P01)2 __, Ca(H2P01lz + 2CaS01

H2S04 + 2NH, __, (NH1hS04

Below are the overall equations for the manufacture of three fertilisers (underlined).

Nitrogen is reduced by hydrogen.

31

reaction 2:

3H2 S01 + Caa(PO,)i + 6NH3 -

2(NH,)3PQ, + 3CaS01

reaction 3:

M,

98

HaS01

(NH1)2S04
132

Ca(H2P01J2
234

NH,)3PO,
149

The relative molecular mass, M,, <:i sulfunc acid and of each fertiliser is shown in the
table.

It is not possible to obtain a 100% yield of ammonia.


A

Which of the following statements about the Haber process is n ot correct?

A high temperature will increase the yield of ammonia.

2
3

greatest mass

14

3
2
1
2

least mass

In each reaction, 98 tonnes of sulf1.ric acid was used. Which reaction gave the
greatest, and which the least, mass of fertiliser?

The iron catalyst is used to increase the speed of the reaction.

13

32

33

heal

A sample of clean, dry air is passed over hot copper until all the oxygen in the air
reacts with the copper.

The volume of air decreases by 30cm3 .

150 cm3

60cm3
D

B
300cm3

100 cm3

What was the starting volume of the sample of air?

- -i:_------;

_ _ petrol

------c _ .

-------c= -

'---x

------r

The diagram shows the separation of crude oil into fractions.

crude oil

~-brtumen

W hat could X, Y and Z represent?

lubricating oil

lubricating oil

diesel

diesel

paraffin

diesel

y
lubricating oil

lubricating oil

diesel

paraffin

paraffin

paraffin

15

34

The solvent ethanol is produced by the fermentation of sugar, using yeast.

1CO

100

Which graph correctly shows how the speed of fermentation changes with temperature?

speed of

temperature l"C

temperature /C

100
temperature iC

I~

temperature/ "C

100

O'--------.........

speedof
fermentation

0 - - - - - -..............

fennenlabon

~-----+-t~

speed of

fermentation

speed of

fermentation

16

35

36

Five structures are shown.

~-~-l-l=l
I

H-

H-

2
H

C-

ll

H-C- H

H-C- C= C
H

37

Sf!

How does the number of carbon, hydrogen and oxygen atoms m an esti::r differ from
the total number of carbon, hydrogen and oxygen atoms in the alcohol and carboxylic
acid from which the compound was derived?

less

hydrogen atoms

less

less

less

oxygen atoms

I
less

less

same

carbon atoms

less

same

same

same

same

A student investigated the reaction of different vegetable oils with hydrogen. 100 cm of
hydrogen was passed through 1 g samples of vegetable oils containing a suitable
catalyst.

Volume of hydro~en remaining

I I I I
C- C-=C- C-H
I
I

I
I
H-C-C=C
I
I
1-1

H- C- H

II

H-ll:-?-~
H

Vegetable oil

The volume of hydrogen remaining after each reaction was recorded.

1 and 3 only

Which structures represent identical molecules?


A

fem
100

2 and 3 only

C - C - 0 -H

1, 3 and 4 only

C-C -

1, 3 and 5 only

"-o- H

H-

63

87

Q , R and S only

Q and R only

P only

Which vegetable oils are unsaturated?

Sonly

18

The diagrams show two organic compounds.

H - c -c - C - C

Which statement about the compounds S and T is correct?

S and T react together to form the ester ethyl propanoate.


T can be changed rnto S using acidified potassium dichromate(VI).

Both S and T react with sodium carbonate.

They are in the same homologous series.

17

39

40

Alternative fuels are those that can be used in place of conventional fuels.

ethanol, methanol, biodiesel, water, carbon monoxide

hydrogen, vegetable oil, ethanol, natural gas, liquefied petroleum gas

methane biogas, biodiesel, hydrogen, ethanol, wood chips

hydrogen, carbon monoxide, ethanol, wood chips, biodiesel

Which of the following are all alternative fuels?

C-C-C

I
H

I
H

C -

C -

C -

I
H

The diagram shows part of a polymer.

CeH12

C2He

C2H

Which of the following compounds could be used as the monomer?

CeH14

C
D

End of Paper

19

ass
J

NarTlt'l

l
_

Secondary Four Express

PRELIMINARY EXAMINATION 2011

PEI HWA SECONDARY SCHOOL

Index Number -

CHEMISTRY

Paper 2 Theory

No add1t1onal matenals are required

3.

2.

1.

Candidates are to show all their working in a clear and orderly manner.

Section B: Free Response Questions


Answer ALL quesbons.

Section A: Structured Questions


Answer ALL questions

There are altogether 2 sections in this paper: Sections A and B.

Wnte you name, class and index number in the spaces above.

READ THESE INSTRUCTIONS FIRST


Do not open this booklet until you are told to do so.

5.

A copy of the Penodic Table is provided on page 17

l
5072/02
23 August 2011

120

1 hour 45 minutes

Section
Paper 1

Paper 2

Total

This question paper consists of 17 printed pages, inclusive of this cover page

A1

Section A

Answer all the questions in this section in the spaces provided.

The total marks for this section is 50.

Total

lithium, sodium and potassium are elements in Group I of the Periodic Table .
Francium, Fr, is another element m Group I.

{a) How many electrons are in there in the outer shell of a francium atom?

223

(b) Complete the following table about an atom of francium.


mass number

I atomic number
number of protons
number of electrons

I number of neutrons

(c) Predict two physical properties of francium

{d) A scientist predicts that francium reacts v 1clently with water.


Wrrte the equation for this reaction.

[1)

(2)

[2]

[1 ]

[61

A2

..

Oxides of nitrogen are atmospheric pollutants. Nitrogen monoxide, NO, is formed in an


in1ernal combustion engine when nitrogen and oxygen react together.
N2(g) + 02(g) - 2NO(g)

energy
change

The diagram shows the energy profile for this reaction.

energy

N2(g) + 02(9)

progress of reaction

(a) llfentify the energy changes X and Z.

x ..... ....
z............
(b) What type of energy change is involved in this reaction and how can you tel from
the diagram?

(:7)

[1]

(c) The exhaust system of a motor car is fitted with a catalytic converter. When nitrogen
monoxide passes through the converter ii reacts with carbon monoxide.

2NO(g) + 2CO(g) __, N2(g) + 2C02(g)

The catalyst increases the rate of this reaction.

(i) Explain how the catalyst in the converter increases the rate of this reaction.

(ii) l<>uring the course of a journey 2.4 dm3 of nitrogen monoxide was produced by the
engine.

Calculate the volume of nitrogen gas produced if all the nitrogen monoxide
reacted in the converter.

Total

(ffl) In reality, only 1.0 dm3 of nitrogen was produced after the gases had passed
through the catalytic converter. Calculate the percentage of nitrogen monoxide
that had reacted.

[1 J

[1]

[2]

[7]

A3

The distinctive smell of the seaside wos thought to be cau&ed by ozone, 0 3 .


Ozone is a fonn of the element oxygen.

302~203

(a) A mixture cf oxygen and ozone is formed by passing e lectric sparks through oxygen.

(i) g>uggest a technique that might separate this mixture.

(ii) Explain why this method separates the two forms of oxygen.
\

..... .... ...........

(b) Ozone is an oxidant (oxidising agent). It can oxidise an iodide to iodine.

21- + 03 + 2H ~ 12 + O, + H20
(I) What would you see when ozone is bubbled through aqueous acichfied potassium
iodide?

ozone is the oxidant in this reaction_

\ii) Using the information in the equation, explain, in tenns of transfer of electrons why

Total [7]

(c) lt is now known that the smell of the seaside 1s due to the chemical a1methyl sulfiot:.
(CH3)2S.
(i) Suggest the structural formula of dimethyl sulfide
Space for structural fonnula

(2)

[1 ]

[2]

(1)

A4

(ii) Hence. draw a diaQram that shows the arrangement of the valence electrons in
one molecule of this cQlla!eAH!ompound.

Use x to represent an electron from a carbon atom.


Use o to represent an electron from a hydrogen atom.
Use to represent an electron from a sulfur atom.
Space for diagram

Total

There is r:iuch international concern that an increase in the atrnosphenc concentrations


of methane and carbon dioxide can lead to global warming.

year

1.68 x 10-3

percentage, by volume, of methane


in the atmosphere

2003

1998

1993

1.79x10-3

1.78 x 10-3

1 73 x 10-3

1.71 x 10-3

3.85 x 10-2

3.75 x 10- 2

3.65 x 10- 2

3.55 x 10- 2

3.49x10-

percentage, by volume, of carbon


d ioxide in the atmosphere

The table shows the atmospheric :oncentration of methane and carbon dioxide over the
last 20 years.

1988

2008

[1 J

[7]

Methane is about 30 times more effective than carbon d ioxide as a greenhouse gas.
(a) Give one source of atmospheric methane.

(b) Describe two possible consequences of an increase in global warming.

(c) Use the information above to explain why scientists are as concerned about
methane in the atmosphere as carbon dioxide.

(d) Explain why both carbon dioxide and methane are gases at room temperature.
Use ideas about structure and bonding.

(e) Methane can be manufactured by reacting carbon dioxide with hydrogen. Water is
the only other product.
Construct the equation for this reaction.

[1)

(2)

(2)

(2)

[1)

o\.6

(f) Methane reacts with chlorine. Name the type of reaction that takes place and identify
one o rganic and o ne inorganic product of the reaction .

type of reaction .....


p roducts of reaction

Total

Butan-1-ol is used as a solvent for paints and varnishes. to make esters and as a fuel.
Butan-1-ol can be manufactured from but- 1-ene, which is made from petroleum.

Biobutanol is a fuel of the future. It can be made by the fermentation of almost any form
of biomass - grain, straw, leaves etc.

(a) But-1-ene can be obtained from alkanes such as nonane, CsH20. by cracking.

(i) Complete an equation for the cracking of nonane, C9Hoio, to g ive but-1-ene and
one other product.

C9H20 -+ ....................................~...................... .

(ii) Name the reagent that reacts with but-1-ene to form butan-1-ol and write the
chemical equation for the reaction.

Reagent ......... ............................. . ......... ..... .


Chemical equation ...

(b) The fermentation of biomass by bacteria produces a mixture of products w hich


include biobutanol , propanol. hydrogen and propanoic acid.

(i) Draw the structural formula of propanol and of propanoic acid. Show a ll the
bonds.
propanol

propanoic acid

[2]

[10]

(1)

[2)

(2)

A6

petroleum?

(ii) Why ls it importc111l lo develop these fuels. such as biobutanol, as alternatives to

Total

(iii) How could you show t'1at butanol made from petroleum and biobutanol are the
sarie chemical?

(a} thromium is a transition element.

..... .......... .....

(i} Predict one drfference in the physical properties of chromium and sodium.

...........

(i i) Predict one difference in the chemical properties of chromium and sodium.

+I

add more
chromium( III)
sutfate{aq)

object to
be plated

(b) CrromiurP is used to electroplate steel objects. The diagrarr. shows how this could
be done.

!ead anode

sulfate(aq)--'

chromium(m)

[1]

[1]

[7]

[1]

(1 I

(i) Give one reason whv steel objects are plated with chromium.

Iii) ihe formula of the chroniium(lll) ion is cr3 and of the sulfate ion is SO/-.
Give the formula of cf'romium(lll) sulfate.

p;J WritP the eauation for the reaction at the negative electrode (cathode).

Total

{i'' i During electrolysis, It is necessary to add more chromium(lll) sulfate but during
copper-plating using a copper anode, it is not necessary to add more copper(ll)
sulfate. Explain.

10

[1]

[1]

[1]

[2]

[7]

A7

experiment
number

50

100

50

50

volume of acid
/cm3

2.0

2.0

2.0

concentration
of acid in
mol/dm3
1.0

35

25

25

25

temperature of
acid/ 0C

33

65

65

132

reaction
time/seconds

Antacid tablets neutralise acids. A student investigated the time taken for an antacid
tablet to react completely with excess hydrochloric acid (the reaction time) under
different conditions.The table below shows the results.

I
(a) Which two experiments show that the volume of acid used does not affec: the rate
of reaction?
........ and ....... .
(b) From the data in the given table, deduce one factor which affects the rate of
reaction. Explain clearly how you used the data to arrive at your answer.

..... ....................................... ... ... ...... ...... .. .... . .. ............................. ..........

-+

C02(9) + H20(1) + CaCl2(aq)

{c) The antacid tablet contains calcium carbonate. T he equation for the reaction
between calcium carbonate and hydrochloric acid is:
CaC03(S) + 2HCl(aq)

(i) Calculate the number of moles of hvdrochleric acid used in experiment 2.

(1)

[2]

[3)

(3)

Total [6]

(ii) Calculate the mass of calcium carbonate that will react with this number of moles
of hydrochloric acid.

End o f Section A
11
End of Section A

Section B

Bromine is an important industrial chemical used for making flame retardants.


Bromine is extracted from sea water, which is naturally rich in bromide ions.

The total mark for this section is 30.

You are to start each question on a fresh page.

A nswer all three questions in this sectior. on the writing paper provided. The last question
is in the form of either/or and only one of the alternatives should be attempted.

1'18

Chlorine can be bubbled through sea water to convert bromide ions into bromine.

A studert carried out an experiment in a laboratory to investigate the reaction


between bromide ions and chlorine gas

She bubbled chlorine gas through dilute aqueous potassium bromide for 6 minutes.
She took samples of the reaction mixture every 30s and measured the colour
intensity of each sample using a colorimeter.

detector

absorbancc
reading

A colorimeter measures hght absorbed when the light passes through a coloured
solution.

The diagram shows how a colorimeter works .

/1'

-b light
souroe

sample being
tested

The darker the colour of the solution, the more is light absorbed and the higher the
absorbance reading on the colorimeter.

Aqueous bromine absorbs more light than aqueous chlorine or potassium bromide.
The student plotted her results on a graph.

12

[1 ]

Propanoic acid, C2H5C02H. is a weak acid.

EITHER

Bt'O
(a) Explain what is meant by the term weak acid.

[1)

(C2H5C02)2Mg + H2

(2)

:21

(b) Propanoic acid reacts with sodium carbonate. Write the equation for tr.is
reaction.

-+

(c) Magnesium reacts with propanoic acid to form magnesium propanoate and
hydrogen.
Mg + 2C2H5C02H
A student added 4.80 g of magnesium to 30.0 g of propanoic acid.

Explain your answer.

II

[1]

1-o-c---c-o-D-o-

0
I!

(I) Which one of these reactants, magnesium or propanoic acid, is in excess?

(ii) Calculate the volume of hydrogen formed at r.t.p.

II

(d) Terylene has the simplified structure shown.

II

(i) State the functional groups on the monomers used to make Terylene

[1)

- c- - -c-o-i

(II) State the type of polymerisation that occurs when Tery/ene is made.

[1)

[10]

[1]

(i ii) State one large scale use of Terylene.

Total

(e) Many problems are caused by the disposal of plastics. Describe one method of
disposal of a plastic and a problem caused by this method.

15

OR
81-0

Sorrel is a small green plant.

[2]

(a) The pigments in the sorrel leaf car be separated by chromatography.

(I) Describe how chromatography can be used to separate different pigments.

(1)

(10)

(1)

[1]

[2]

(1)

(1)

[1]

(II) Explain what is meant by Rrvalue.

(b) Sorrel plants contain a poisonous carboxylic acid X. What can be deduced about
X from each of the following three pieces of information?

(i) When X is warmed with acidified potassii;m manganate(Vll), the solution


changes from pink to colourless.

(ii) Aqueous bromine is not decolourised when added to a solu1ion of X.

(iii) A 0 .1 moVdm3 solution of X has a pH of 3 whereas a 0. 1 mol/dm solution of


hydrochloric acid has a pH of 1.

(c) Analysis of 10.0 g of carboxyhc aod X shows that it contains 2.67 g carbon,
0.220 g hydrogen and 7.11 g oxygen

(i) Deduce the empirical formula of X.

Total

(ii) The relative molecular mass of X is 90. Deduce the molecular formula of X.

(iii) Draw the possible structure for a molecule of the acid X.

End of Paper

16

PAPER 1

PEI HWA SECONDARY SCHOO L


2011 PRELIMINARY EXAMINATION CHEMISTRY 507211 and 507212
MARKING SCHEME

A
22

21

Key

1
B

23
D

Question Number

c
24

Key

3
D

25

Question Number

4
B

27
B

26

28

7
A

8
29

31

32

11
B

33

30

12
D

34

13

10

14

3S

36

15

16

PAPER 2

A1

ll lCU ""J

one

""l"1U

proton (atomic) number= 87


number of protons = 87
number of electrons = 87
number of neutrons= 136

[1 ]

All correct= 2 marks [2]


Any 3 correct = 1 mark
(c)

[1 ]

[2]

Any two of:


thermal conductor I
electrical conductor I
soft or cuts easily I
low melting point or low boiling point I
(relatively) low density
malleable/
ductile I
shiny or silvery-grey

Total

2Fr + 2H 20

(1 )

2FrOH + H2
(d )

X = activation energy

energy of product (2NQ) is greater that of the reactants (N2 and 02 /


[1]
energy of reactants (N2 and 0 2 ) is less that of product (2NO)

energy change is positive/enthalpy change is positive because:

--+

(a)

[1]

(b)

(a)

""l::Y 1 I V l'W

A2

Z =enthalpy change (of reaction)


(b)

(c)(i)

Volume of N2 produced

=2.4/2 = 1.2 dm,

[1]

[1]

(unit required)

The catalyst lowers the activation energy for the reaction /provides
surface for molecules to reacV so that the reaction takes place by a
[1]
faster alternative pathway.

(ii)

Method 1

(NOT: allows more frequent collisions]

(iii)

Hence, actual no. of mol of NO reacted (converted to N2)

37

38

= 2 x 0.0416 = 0.0833 mol

c
A

39

11
18

40

1.0dm3
= 0.04166 mol
24dm 3 I mo/

19

No. of mol of N2 produ:ed =

20

. 1

A3

(a)(i)
(ii)

(b)(i)

h u)

(c)(I)

=0.1 mol
actual
theoretical

x 100%

0.0833 mol x 100%


0. 1mol

=83.3%

Theoretical (maximum/expected no. of mol of NO reacted)


2 .4 dm'
24dm 3 Imo/

Percentage of NO reacted=

Method 2
Ratio of mol of gases =ratio of volumes.
actual volume of N2 produced =1.0 dm 3;
hence actual volume of NO reacted =2 x 1.0 dm3 = 2 dm 3
=2.4 dm3

x 100% =83.3o/o

actual
x 100%
theoretical

Theoretical volume of NO reacted

Percentage of NO reacted=
3

2 dm
2.4dm 3

(1]

[1)

[1)
Total

(1)

By diffusion [1)
ozone molecules and oxygen molecules have different Mr
I different densities or oxygen molecules,
hence oxygen molecules move/diffuse faster than ozone molecules [1)
OR (i) by fractional distillation [1)
(ii) because ozone and oxygen have different boiling points [1)

aqueous potassium iodide turns from from colourless to brown

c
H

-H

(1)

iodide ions (r) loses electrons (it is oxidised); [1)


electrons lost are accepted by ozone, hence ozone oxidises iodide ions
but is itself reduced. (1)

c - s

Structural tomilita of dimethyl sulfide

I
H -

17

A4

(ii )

I (a}

(b)

(c)

I Electronic structure of dimethyl sulfide (only valence electrons shown).

[1 )
Total

[1)
One of these sources:
- bacterial decay of organic matter, e.g. rubbish in landfills I animal
manure:
- from methane hydrate:
- from gas produced by cows
- from action of bacteria (methanogens) in marshes I swamps.
Any two of these three.
[1, 1)
- polar ice melting, leading to sea-level rising and flooding of low lying
areas;
- climate changes I (some) areas w ill have (severe) droughts and areas
may become deserts I decrease in crop yields;
- dissolved carbon dioxide being driven out into the atmosphere due to
increased evaporation of water from oceans. rivers, lakes, thus
increasing the greenhouse effect.

Methane percentage is increasing;


[1)
the same percentage of methane is 30 times more effective than the
same percentage of carbon dioxide, hence even th o ugh the
percentage of carbon dioxide Is about 20 times that o f methane, the
overall greenhouse effect of methane is still greater (by a factor of about
1.5) than that of carbon :lioxide.
[1)

Both carbon dioxide and methane have simple molecular structures I are
made up of small, (discrete)(covalent} molecules;
(1)
with
weak intennolecular forces I weak forces between molecules
hence little energy is needed to separate them;
(1)
hence their melting and boiling points are so low they are gases at r.t.p.

I
(d)

C02 + 4H2 ~ CH. + 2Hi0

[1 I
(e)

17

,___

AS

A6

(f)

Substitution
(1)

(1)

BOTH products to score (1)

O-H

equation that gives:


alkene + alkane I alkene + alkene + hydrogen, e.g.
C9H20 -+ C..He + CsH12
C9H20 -+ 2C4He + CH4
C9H20 4 C4He + CsH10 + Hz

.
H

Reagent : Water I steam


Equation: C4Hs + H20 -+ C4H90H
PropaneI

"'

-~p

I I I
C-C-C-0I I I
H

H- C-C-C

H-

Propanoic acid

~ofthese:

10

[1f -

[1)

[1)

[1)

[1]

(1)

Total
(1)
one of these:
chromium is harder or chromium is hard white sodium 1s a soft metal;
I has higher density or chromium has high density while sodium has low
density;
/has higher melting point and/or boi~ng point than sodium or chromium
has high melting and/or boiling point while sodium has low.

Petroleum is a finite and non-renewable resource; biofuels can help to


conse<Ve petroleum;
OR
blofuels are carbon-neutral, hence can reduce the greenhouse effect.
r11
They would have the same physical properties e.g. they would have the
(1)
sam e boiling eoint.

Total

(f)

(a)(i)

Any two from:


- HCI I hydrogen chloride
- CH3CI I chloromethane
- CH2Cl2 / dichloromethane
- CHCl3 / trichloromethane
- CCl4 / tetrachloromethane

I
(ii)

(b)(i)

(ii)

(iii)

(a)(i)

(ii)

A7

=~dm~
x 2.0 mol/dm 3 =0.1 mol
1000

oc.

(1 I
Total 6

(1)

{1 J

- sodium is more reactive than chromium;


- chromium has more than one oxidation state, sodium has a fixed
oxidation state (of +1);
- chromium forms coloured compounds while sodium compounds are
white;
- sodium reacts with cold water, chromium does not;
- chromium has catalvt1c orooerties, sodium does not.
(b)(i)
One of these:
[1]
- makes steel more shiny/more attractive;
- prevents steel from rLsting by forming a bamer;
- to provide a hard surface on the steel.
NOT becomes harder I stronger
(ii) Cr2(S04)3
[1]
(iii) Cr,.+ 3e- - Cr
[1)
(Iv) Aqueous chromium sulfate is gradually used up because chromium ions
from it are discharged at the cathode {steel plate) to fonn chromium
metal.
[1)
A copper anode would dissolve to give copper(ll) ions to replace those
copper(ll} ions discharged from the c;opper(ll) ulphate solution.
(1]
Total 7
(a)
Experiments 2 and 3.
(1]
Concentration.
(no marl< for this]
(b)
From expenments 1 and 2, when concentrabon of acid is doubled from
1.0 moVdm3 to 2.0 molldm3 , the time taken for complete reaction is
halved from 132 s to 65 s.
[1]
This shows that the rate of reaction increases with increase in
concentration of the acid/ is proportional to the concentration.
[1]
OR
Temperature.
[no marl< for this)
From experiments 3 and 4, when temperature increases from 2sc to
35C, the time taken for the reaction is halved from 65 s to 33 s.
[1)
This shows that when the temperature of the acid Increases by 1
the rate of reaction is doubled.
[1]
no. of mol of HCI used = volume x concentration
(c)(i)

(II)

No. of mol of CaC03 that reacts =Y, x 0.1 =D.OS mot


Mass of CaC03 =0.05 x M,
=0.05 x (40 + 12 + 16x3) g =5 g.

B9

correct equation [1]


state symbols [1]

(b)

(bromine is reddish-brow n; chlorine is yellowish-green} I (bromine is


a darker colour and absorbs more light.)
[1]

experiment 2
shorter time, faster rate
experiments 1 and 3

Moles of hydrogen (0.072 124) = 0.003


mass zinc= no. of mot x A,= 0.003 x 65 = 0.195 g
ALLOW: e.c.f.
Percentage of zinc in the sample
= 0 195/1.2 x 100% 16.3%

11 I
[1]

[1]

[1]

[1 ]
[1]

Both zinc and copper have metallic structures in which 12ositivelycharged ions are arranged in close-12acked. regular arrangement; [1)
These ions are surrounded by a "sea" of delocalised electrons which
[1)
can move to conduct electricity.
Powdered brass provides a larger surface area for reaction , hence.
[1]
reaction is faster.

[1 J
Total

when all the bromide ions have been used up/displaced, reaction
stops and there is no further change in absorbance.

as more bromide ions are displaced/as concentration of bromide ions


decreases, the reaction slows down and absorbance decreases; (1]

3 marks if data are used; 2 marks ff data not used.


at the start of the reaction , the amount of bromine displaced (fonned)
Is the greatest, hence the absorbance is the highest;
(1]

[1]

[1]

[1]

(c)(i)

(a)

(iii)

d(i)
(II)

(ii)

Absorbance is the greatest at the start of the experiment;


(e.g. in the first 2 min absorbance increases from Oto 0.5;)
then decreases w ith time;
(e.g. in the next 2 mins, absorbance increases from 0.5 to 0.7;)
and finally becomes constant at about 5.5 mi n;

SECTION 8 (30 marks)C/2(g) + 2Bqaq)--+ 2C /(aq) + Br2 (aq)


88
(a)

(b)(i)

(ii )

(iii)

(c)

(zinc gives white precipitate) (on addition of aqueous ammonia);


(white) Pill dissolves in excess ammonia giving a colourless
7

12

solution;
co1212er would give (hght} blue 1:mt (on addition of aqueous
ammonia); 1212t would dissolve in excess ammonia,
giving a dark blue solution with excess ammonia.

[1)

(1]

[1J

No blue 1212t will be obtained, showing that only zinc is present.

[1)

Total
Weak acid is one which is only slightly or partly ionised/partly
dissociated when dissolved in water I when in aqueous solution. (1)

2C2HsCOOH + Na2C03 - 2C2HsCOONa + C02 + H20

EITHER (a)
8 10

(b)

M, of pro12arioic acid= (3x12}+(6x1 )+(2x16)=74;

[1]

Method 1

8t of Mg= 24

[1)

= 0.405;

From the stoichiometry of the reaction,


1 mol of Mg requires 2 mol of propanoic acid;
OR
24g of magnesium requires 2 x 74 g of propanoic acid to react;
48
74
x (2x l g = 29.6g acid
24

Hence the acid (30g) 1s in excess


Method 2

= 4.8 = 0.2
mass
No. of mol of Mg used =
molar mass
24
0
No. of mol of mot acid used= ( x 12) + (:x1) 1 (Zx 16)
3

From the equation. mol ratio of Mg: propanoic acid = 1:2


0.405/2 moles = 0.203 moles acid com12ared with 0.2 moles Mg,
hence acid is in excess.

Since the limiting reagent is Mg,


0.2 mot of Mg reacted to give 0.2 mot of Hz;
( allow ecf from part (1) for wrong value of mol of Mg; )

Volume of H2 = no of mol x molar volume = 0.2 x 24 =4.8 dm3


(correct unit needed)

[1]

[1]

[1]

[1]

[1]

so 4.8g magnesium requires

(c)(i )

(ii)

(d)(i)

cart>oxyl groups from the carboxylic acid monomers;


hydroxyl grou12s from the alcohol monomers.

(both required)

OR
B10

(ii)
(iii)

(e)

(a)(i)

Condensation polymerisation.
To make (named) clothing /boat sails /conveyor or fan belts.

(1)
[1]

(1]
one of these:
- buried in landfill; causes land pollution as plastics are not
biodegradable and will take up space;
incineration/burning; harmful (toxic) fumes gases are produced.
e.g. carbon monoxide; hydrogen chloride.

different types of plastics/ polymers. )


Total

(Allow:
incineration produces more carbon dioxide which increases
global warming.
If not disposed off but when recycled - it is difficult to sort out

Either:
Description without diagram:
pencil line drawn about 2 cm from bottom of a strip of
chromatogra11hy 11a11er; s11ot of sorrel Qigment dissolved in
ethanol (or other suitable solvent} Qlaced on the ~ncil line; (1)
chromatograQhy 11a11er sus~nded vertically in solvent in a
beaker such that the 11encil line/sorrel s11ot is !!2Qve the solvent
level; (until coloured spots develop)
[1]

solvent (e.g. ethanol)

Pencil linm

SpotofP<Vmtit

B e:>Qr

Slripol ~

named solvent ; bottom of paper dipping into solvent;


spot of mixture put on pencil line on the paper,
original spoVpencil line above solvent level;
{optional: separated pigments vertically aligned on the paper}.

Or:
Description with diagram - equivalent marks if description s not
clear/complete but the diagram completely illustrates what the
process is about, i.e. diagram clearly labelled and shows:

10

f--

(ii)

II

0.220
1

II

c -

0
7. 11
16

=0 220 !.:.!! =0.4444


16

2.67
12

- c -

-H

Total

[1]

(1)

(1 I

[1]

[1]

[1]

[1]

Rr is the ratio of distance s11ot moves to distance of solvent front from


base {starting) line;
Or.
R _
distance moved by spot
1
- distance of sol1ent front from the base (starting) line

Element
Mass(g)

"'A;

M, of X n(12+1+ 2x16) 45n;


n = 2;
45n = 90;
Hence, the molecular formula of X 1s C 2H2 0,.

Let the molecular formula of X be {CH02)n:

Hence, the empirical formula of the acid 1s CHO,_

0.220

no. of mol

0.2225 : 0.220 : 0.4444 = 1.1:2


0.220 0.220 0.220 - -

=0.2225
ratio of
mol

.67
12

(b)(i)
X is a reducing agent as it reduces Mn ion in potassium
manganate(Vll) to Mn2 .
(ii) X does not contain a (C=Cl rtouhlF> bond;
I X 1s saturated;
(iii) X is a weak acid while hydrochloric acid is a strong acid;
IX is a weaker acid (than hydrochloric acid;
(c)(i)

(ii)

(iii)

10

10

A
B

carbon dioxide
chlorine
hydrogen
hydrogen chlonde

First

filter
dissolve
dissolve
dissolve

dissolve
evaporate
filter
evaporate

evaporate
crystallize
evaporate
filter

Last
crystallise
filter
crystallise
crystallize

Zinc nitrate crystals can be separated from calcium carbonate using the four
processes shown below. Which of the following shows U1e processes in the
correct order?

What is 1he gas?

The diagram shows a simple laboratory apparatus for the preparation


and collection of a dry gas.

QUEENSWAY SECONDARY SCHOOL

L.

ammon ia
carbon dioxide
chlorine
nitrogen

GasX
has no effect either on damp red litmus paper or on damp blue
litmus paper,
puts out both a glowing splint and a burning splint.
What is gas X?

A
B
C
0

.+

!)

to act as a catalyst
to oxidise the cblonde ion
to prevent precipitation of silver carbonate
to prevent the decomposition of any silver chloride formed

When testing for a chlonde ton using silver nitrate, the solutton must be
acidified with dilute nitric acid.
What is the purpose of the nitric acid'?

The table gives the properties of four substances.


Which substance is a sohd metal at room temperature?

1c

1465

1c

./

conductivity

./

,/

x = does not conduct

./ == conducts

key

electrical

808

890

./

boiling point

98

445

./

malting point

119

357

electrical
conductivity
when mol!en

--39

when solid

cotton wool soak&d In


concentrated
hydrochloric acid

--~D

Methylamine, CH3NH 2, and hydrogen chloride, HCI are both gases which are
soluble in water.
The gases react together to form a white solid, methylammonium chloride.
!n an experiment to demonstrate rates of J.iffusion the following
apparatus is set up.
Where will the white sohd form?

LJ~

cotton wool soaked in


concentrated methylamine
solution

ls

1()

more neutrons than protons.


more protons than neutrons.
more electrons than protons.
more protons than electrons.

Which statement about the numbers of particles in atoms is correct?


Apart from hydrogen, most atoms contain
A

c
D

loo

Lt
At3

o2

er
17

protons
3
13
8
8
20

4
14

neutrons

Q
S1

Cl

11

The diagram shows the structure of boron nitride which is sin1ilar to graphite.

1<11y
boron
n1tmg11n

It is a lubricant
It is transparent when pure.
It is very hard.
It has a low melting point

Which property is this compound likely to have?

A
B

2.8.2
2.8.3
2.8.6
2.8.8.1

<.: 3 only

D I, 2 aod3

Atoms ofY have more protons than atoms ofX.


Molecules ofY have more atoms than molecules ofX.
Y displaces X from aqueous soluuons of:X- ions.

B 2 only

They arc strong oxidizing agents.


They form oxides on reacting with wa1i:r.
Their melling points decrease on descending Group I.
Their reactivities decrease on descending Group I.

Which statement about the alkali metals is !rue?

A I only

Which statement~ arc correct?

I
2
3

Elements X and Y are in Group Vll of the Periodic Table.


X is a liquid at room temperature. Y is a solid at room temperature.

Pis below Qin tl1c same group of the Periodic Table. Ifthc proton number of
Q is 11, what is the electron arrangement of atom P?

C
:::>

12

A
B

II

Q/R ~ Q

V/R~Q/R~

13

JO
8
16

electrons

Which ion has a correct set of numbers for its structure?

c
D

111e forces of attraction between tile molecules.


The reactivity ofthc molecules.
The shape ofthc molecuh:s.
The strength of the covalent bonds in the molecules.

When a covalent hquid boils its molecules become more widely spaced.
Which property of the molecules has the most influence on the energy
required to boil a covalent liquid?
A
8

c
D

A molecule containing atoms of elements Q, Rand V has the following


structure. Which oft:1e following shows a possible combination oftbe
clements?

Cl

I'\

Ge

c
s

14

A
B
D

I~

16

17

Te02, Na2Te, Na2Te04


Te02, Na2Te, NaTe04
TcO;, NaTe, Na2Te04
TeO, Na2Te, NaTe04

Sulfur and tellurium (Te) arc both elements in Group VI of the Periodic Table.
Which of the following correctly list the fonnulae of some tellurium
compounds?
A

14 gofiron (II) sulfide


11 g of iron (rt) sulfide
11 g of iron (II) sulfide and 3 g of unchanged iron
11 g of iron (11) sulfide and 3 g of unchanged sulfur

When 7 g of iron reacts with 4 g of sulfur, 11 g of iron (II) sulfide is produced.


What will be produced if 7 g of iron is reacted with 7 g of sulfur?
A
B

C
D

tartin
line

P Q R X

developed with ethanol

It was suspected that an unkno\.vn substance X wa~ one of the three substances I',
Q, or R.
Spots of each substance were put on the starting lines of two pieces of
chromatography paper. One paper was developed with water and the other with
ethanol. The diagrams show the final cluomatograms

developed with water

P Q R X

It must be P
It must be R
ltmustbeQ

What is substance X?

It must be P or Q

A
B
D

18

!9

20

~l

Carbon monoxide and sulfur dioxide.


Nitrogen dioxide and sulfur dioxide.
Methane and ozone.
Nitrogen dioxide only.

Which pollutants can be removed from the chimney gases of factories and
power stations by treatment with calcium carbonate'!
A
B

c
D

methane
depletion of the
ozone layer
global wanning
photochemical
Smog
global warming

acid rain

Sulfur dioxide

global warming

Carbon dioxide

depletion of the
ozone layer
global warming

acid rain

acid rain

photochemical
smog
global warming

Methane, sulfur dioxide and carbon dioxide are gases which affect the
atmosphere and the environment
In what way do these gases affect the environment?

c
D

Sodium hydroxide solution


Potassium dichromate(Vl) solution
Siver nitrate solution
Anunonia solution

Which oompow1d, when added to aqueous iron (II) chloride, takes part in a
redox reaction ?
A
B

Phosphoric acid, H 3PO~ is found ut rust-removing products.

2 H,PO, + FezO; ? 2FcPO, + 3H20

an alkali
a base
a reducing agent
an oxidising agent

In the: process, the rust is acting as


A
B
C
D

22

"3

24

25

They react with copper to give hydrogen.


-:-hey react with magnesium oxide to give hydrogen.
They react with aqueous sodium carbonate tc give caibon dioxide.
They react with aqueous silver nil.rate to g'.ve a predpitate.

Which one of the following statements about the reactions cf dilute


hydrochloric acid, dilute nitric acid and dilute ~ulphuric acic is correct for a:1
three acids?
A
B

x_

x
ammonia
chlorine
carbon dioxi<!e
sulfar dicxide

potassium iodide
potassium nitrate
sodium hydroxide
iron (II) suifate

A gas Xis bubbled into a solution ofY as shQwn in the ciagram below. In
which cf the foll o\\~llg combinations would X reac: with Y?

A
~

:De volume of water increases.


';he Off ions neutrabze sorr. e o: 1t.e H ions.
Tl:e pH o!' the soh~:ion increases.
The reacticn Na'(aq) + c:(a-l) ~ NaCl(s) takes place.

Whell dilute sodium l1ydrox:Je sviution is addeC: to dihr.e h~.:rcchloric acid in


a beaker, whic.:1 one of:he :ollowing v:11i not occi;r in the beaker?
A
B

They have high densit:es.


Tncy have high melting points.
111ey are harJ.
They are able to conduct elcctrici1y.

Which one of the following statements is TRUE for all metals?

20

Ammonia is oJuced from hydrogen at1<l :.iitrogen in a process ca;k d Haber


precess.

v=.

llme

The gn;ih shows the yield o~ ammonia -'I 200C and Ja.m.
yie:ld of ammonia

400C

,,,,..,, -----// 200c

_____J00C

----- -400C

arr.mcr.ia

yield .::r

yieldofl

time

time

;)

time

200c

,',,,----A00C

--- ........... 400C

.
onia

----200c

Which graph shows a cmrect comparison of the yield of anunonia pmduced at


temperature cf 400C wi1h yield zt 200C, keeping 1he pressure at J 3tm?

y:e!d of
ammonia

yield 0 ; 4
amrr.onia

,,,.......
c

'27

28

~9

The graph below shows the total volume of oxygen evolved when
manganese{ IV) oxide which acts as a catalyst is added to hydrogen peroxide..
Total volume of
gas evolved

Time

The manganese( IV) oxide is used up.


Rate of production of oxygen increases with time.
The surface area of manganese( IV) oxide increases.
The hydrogen peroxide is being used up.

Which of the following statements explains the shape of the graph?


A
B
C
D

--

A gas is passed over heated copper (II) oxide as shown in the diagram below.
The products arc a pinkish brown powder and a colourless liquid that boils at
IOOC.
gas
_,.'1;o~) oxide

fi11eat

-==: ' z

ammonia
carbon monoxide
hydrogen
oxygen

Which oflhc following is the gas used?


A
R

c
D

tapwater
nail

t -r-----B

copper rod

zinc rod

ap water

carbon rod
D

t:=4:::::::-::::

r-:m-----t--

Which iron nail in the beakers shown below would undergo corrosion most
readily?

~iron
A

30

3~

32

clectrolyz.ing an aqueous solution fonncd from their ore.


electrolyzing the molten ore.
heating the ore wuh coke.
heating the ore with a more reactive metal.

Metals that arc situated near the top of the reachvity scncs arc usually
extracted from their ores by
A
B

is oxidised by a bl~t of hot air.


reacts with added lunestone.
is reduced by carbon monoxide.
decomposes at a high temperature.

Iron 1s fonned in the blast fumace when iron(lll ) oxice


A

B
C

reaction progress

>c

enthalpy

enthalpy

l:l

reaction progress

reaction progress

Wh ich graph represents the enthalpy change for an acid-base neutralization


reaction?

enthalpy

enthalpy

V\
reaction progress

:n

34

35

The heat re!lecting shields of some space rockets are gold-plated using electrolysis.
Which electrodes and electrolyte would be used to gold-i;late the heat shields?

heat shield

gold

carbon

negative electrode

carbon

heat shield

heat shield

positive electrode

gold compound

copper compound

copper compound

gold compound

electro! yte

c
gold

beat shield

Concentraied aqueous zinc chloride is electrolysed with platinwn electrodes. What


are the prodUCIS al the electrodes?
Cathode(-)
Anode(+)
A
Ollorine
7..inc
Hydrogen
Oxygen
B
Oxygen
Zinc
Zinc
Chlorine
D

tin

First cell
electrodes
Iron/iron
Tm/copper
Tm/Magncsmm
Tin/Zinc

iron

iron/Zinc
Magnesium /Zinc
Zinc/Copper
Magnesium/Copper

elec trc>des

Second cell

iron

COl)pel

ln each cell, only the underlined electrode dissolved To establish the order of
reactivity of the metals, it is necessary to set up two or more cells. Which of the
following pairs of cells are needed in addition to the four cells above?

36

l~

38

~9

40

have the same empirical formula.


contain the same number of bonds.
have the same number of carbon atoms.
undergo sinJ.ilar reactions.

All members of a homologous series


A

B
C
D

(I) only
(111) only
(II) and (111) only
(I), (11) and (Ill)

Which oftbe following can be the products when butane, C4H 10, undergoes
catalytic cracking?
(I)
C02 and H20
C3Hs and Cl-4
CJH6 and C~
(ll)
Oil)

A
B

c
D

C2H6
C4H6
CsH10
C1oH12

One mole of a hydrocarbon M reacted completely with two mole of hydrogen


gas in the presence of a heated catalyst . What could the formula of M be?
A
B

They belong to the same homologous series.


ll1ey are unsaturated hydrocarbons.
They are gases at room temperature and pressure.
They have the san1e physical properties.

Which one of the following statements concerning the compound5 C 2H4, C 3H6
and C4H10 is TRt.:K?
A
B
C
D

A polymer bas t.!le structure shown.

!H)

!H)

~H;

-CH2 -CH - CH2-Cll - Cll2 - CH -

C2H

End of Paper

Wbat is the molecular fonnula of the monomer?

C2H6
C1H6
C;Hs
C
D

Level:

QUEENSWAY SECONDARY SCHOOL

CHEMISTRY
5072 PAPER 2
Time: 1 hr 45 min
Total Marks: 80

PRELIMINARY EXAMINATION 2011

SEC. 4 Express

Name/Index No:
Class: 4~
INSTRUCTIONS TO CANDIDATES:
DO NOT OPEN THIS BOOKLET UNTIL YOU ARE TOLD TO DO SO.
Read these notes carefully.
Write your name, registration number and class in the spaces at the top of this page and
on all the wor1< you hand in.
Write in dark blue or black pen.
You may use a pencil for any diagrams, graphs or rough working.
Section A
Answer all questions in the spaces provided on the question paper.
Section B
Answer all three questions, the last question is in the form either/or.
Write your answers on the lined paper provided.
At the end of the examination, fasten all your wor1< securely together.

89
810
811

..,._ ........ ......

TOTAL
..._

'-

:.~

INFORMATION FOR CANDIDATES


The intended number of mar1<s is given in [ ] at the end of each question or part question.
You may use a calculator.
,__For Examiner's Use A copy of the Periodic Table is pnnted on page 15.
Section A

,,,

__
............

..

This Question Paper consists of 15 printed pages including the cover page. [Turn over]

<\. l

Sli:CTJON A
Answer all the questions in this section Ill the spaces provided
The total mark for this section is 50.
You should aim to spend no more than 65 minutes on this section.

75
50

81

Relative atomic mass

[l]

[I]

[I]

Suggest a name given to elements which have different relauvc abundances with
different atomic mass, such as clement X.

Suggest a name for the element X.

Using the diagram. calculate the relauvc atomic mass o f a sample of X.

79

fhe following diagram was obta111ed from an analys is of naturally occurring


element. X.
Relative
abundance
of X (%)

(a)

(b)

(c)

(d)

Aluminium reacts with a sample of element X to fonn a compound AIXJ which has
high boiling point. Draw a 'dot and cross' diagram to show the bonds in AIX3. Show
only the valence electron~

[I]

(c)

(a)

(b)

(c)

[2]

Element X reacts with hydrogen to fonp the compound HX, which has a low
boiling point. Explain, in tenns of bonding, why the boiling point ofHX is low.

silver

manganese

[2]

Silver occurs in the Earth' s crust as pure silver, but the other metals are
found as compounds.
Barium is the only one, of the four metals which reacts with cold water.
Nickel does not react with aqueous manganese nitrate.

nickel

Study eacli Oftlle statements below and then place the following metals in order of
their chemical reactivity, placing the most reactive first.
Barium

Most reactive

Least reactive

- --,.------

- - - - - - (2]

Explain why the statement that the zinc is oxidized in this reaction is correct.

Describe what you will observe before the reaction is started and after is
completed.

An excess of powder zinc is added to aqueous copper(Il) sulfate.


(i)

(ii)

_ _ _ _ _ _ __ _ _ _ _ _ _ _ _ _ (!]

Titanium white is a cheap chemical but pure titanium metal is very


expensive. Suggest an explanation.

A common white pigment in paints, paper, rubber and plastics called titanium
white is an oxide of the metal Titanium(Ti).
(i)

[I]

A3

(ii)

State one main advantage, other than economic advintage, of recycling


titanium.

(b)

(a)

The hydrocarbon X rapidly removed the orange colour from bromine.

Calculate the number of carbon atoms in one molecule ofX.

Calculate the volume of carbon dioxide formed at r.t.p. from one mole ofX.

Il l

(I ]

[I]

When 0.02 mole of a hydrocarbon X, was completely burnt in oxygen, 960 cm3 of carbon
dioxide was formed at room temperature and pressure.

(c)

[ l]

Write the structural formula of the product formed by hydrocarbon X and aqueous
bromine.
(I]

Give the molecular formula ofX.

(d)

Write the equation for the complete burning ofX in oxygen.

(2]

(e)

,\4

Tartaric acid 1s found in grapes. It has the compositicn by mass: 32% carbon, 4% hydrogen
and 64% oxygen. Its formula can be reprcsen1ed as H2X.
A standard solulion of tartaric acid was prepared by dissolving I .5g of pure tartaric acid in a

I '2X + 2Na0H

Na2X + 2 HiO

What is the empirical formula of tartaric acid?

How many moles of tartaric acid were presenl in the original l.Sg?

Calculate the relative molecular mass of tartaric acid.

[I I

[3]

[2]

250 cm 3 graduated flask. 10.0 cm 3 of O. lmol/dm 3 of sodium hydroxide was neutralised by


12.5 cm 3 of the acid solution, according to the equation:

(a)

(b)

(c)

AS

Potassium
Add water

Alkali A + Gas B

mix

Insoluble base E

Add dilute sulfur ic kid and warm

Salt solution F

Magnesium

Add dilute hydrochloric acid

Salt C +Gas D

Evaporate the solution

Epsom Salts

Identify the substances C and E and F.

~~~~~~~~~~~F

(I]

When red litmus paper 1s dipped into solution A, it rums blue. Name the ion that is
responsible for this observauon.

[3)

Srudy the reacllon scheme shown below which shows how crystals of a compound,
commercially sold as Epsom Sa lls, can be prepared and answer the queslions that follow.

(a)

(b)

(c)

[3)

Epsom Salts can be prepared in the laboratory by a more direct method. Suggest the
chemicals you would use and briefly describe the procedure for the preparation.

A6
Boil 3 litres of water and add 1 kg of sugar. Stir until all the sugar dissolves.
Add 0.5 kg of crushed blackcurrants and let the mixture cool to 25C.
Add lg of yeast.
Cover the container and leave it in a warm place for 5 days.
Filter the mixture into a glass jar which is fitted with an airlock.
Allow the mixture to ferment for 3 months before filtering and bottling.

In the laboratO!)\ a pupil uses the following procedure to make blackcurrant wine:
Step I.
Step 2.
Step 3.
Step 4.
Step 5.
Step 6.
Why was the mixture cooled before the yeast was added to it?

Suggest how the compound in (d){i) might have formed.

Full structural formula:

[l]

[ l]

[l]

[I]

[I]

Give the full strucrural formula of the compound which causes the sour taste
in blackcurrant wine.

The blackcurrant wine may have a sour taste after long storage.

Write an equation to show what happens during the fermentation in step 6.

Why was the glass jar fitted with an airlock?

(a)

lb)

(<:}

( d)
li)

(It)

A.1

nitrogen
xides

15
ratio of air/petrol by mass in mixture

20

Petrol vapour is mixed with air before being bumtin the engine of a motorcar.
The amount of air mixed with petrol vapour in an engine can be varied. Two pollutants in
the exhaust gases from a motorcar are carbon monoxide and nitrogen oxides. The graph
shows how the amounts of these pollutants in the exhaust gases depend on the composition
of the air/petrol mixture.

10

polluting gases. Nali\e these gases.

(J)

Carbon monoxide and nitrogen monoxide can react together to produce two less

Motorcars produce the maximum 4l!lOunt of energy when the air/petrol ratio is about
15. Suggest why the max imum amount of nitrogen oxides is produced under this
condition.

[l]

Suggest why the concentration of carbon monoxide changes as you described in (a)
above.

_ _ _ _ _ _ _ _ _ _ _ _ _ _ _ _ _ (2)

Describe how the concentration of carbon monoxide varies as the air/petrol ratio is
increased in the engine.

concentration
of
polluting gas

(a)

(b)

(c)

(d)

' l]

AS

beaker cont3:1ning

An expenment was carried out by passing a constant current of2.0 A (amperes) through
concentrated aqueous copper (!I) chloride, using carbon electrodes, for nearly three hours.
The diagram bc:low shows the setup of the experiment.

1i1

electrode L

lJ LJ
bromide

@ill---- aqueous sodium

(!)

Write an ionic equation for the reaction that occurred in the beaker

Name the gas that is produced at electrode L.

(2]

(I]

After 10 minutes, a gas was produced at electrode L and 11 turned the colourless
aqueous sodium bromide reddish brown

U-tube contammg
concentrated aqueous
copper (II} chloride

(a)

(1i)

Observations
Pink deposit at electrode K.
The solution in the beaker remained reddish brown.
The solution in the U-tube became much oaler.

The table below shows the observations for the experiment at different stages.
Sta2e of uotriment
Stage I
(after I hr)

Bubb! ing is observed at both electrodes.

(2)

All bubbling had ceased and the liquid in the U-n1be would
no lon~er conduct electricity.

cathode:

Wnte an ionic equation for the reaction that occurred in the anode during stage II.
[I]

anode:

Name the products responsible for the bubbling at the cathode and anode in
Stage 11.

Stage 11
(after 2 hrs}
Stage Ill
(after 3 hrs)
(b)

(c)

(d)

Explam the observations m stage II and Ill.


Stage II

Stage Ill

121

B9

SECTION B
Answer t hree questions from this section.
The last question is in the form Either/Or and only one alternative should be attempted.
The total mark for this section is 30.
You should aim to spend no more than 45 minutes on this section.

3000
2500
2000 I

MQO
,'
'

'

. .. '.Si02

~203

The graphs show the melting point of some elements and their oxides in Period 3 of the
Periodic Table.

~
Cl

1000

I
Na/

--

\_
~

'&.1500J-~~"'-~~~~~~~~~~,,_~~~~~~~~~~~~
c

""~
500

-500"--~~~~~~~~~~~~~~~~~~~~-

{e)

(d)

(~)

(bJ

(tt)

Draw a suitable diagram to illustrate the type of bonding in magnesium.

~~

Compare the melting points of the first three elements and their respective
oxides. Is th ere a trend and what possible suggestion you can make to explain the

Explain why you would expect the melting points for the first three oxides of the
elements from Period 3 to be much higher than that of sulfur and chlorine? (2)

Silicon has the highest melting point among the elements. Suggest a possible
reason to explain why this element has much higher melting point than any of the
Period 3 metals.
(2]

Describe the trend in the melting point of the elements across Period 3 of the
Periodic table.
[2]

Explain why aluminium is placed in Period 3 of the Periodic Table.

[I j

[IJ

<f)

JUO

Two experiments were carried out to measure the rate of reaction between excess powdered
zinc and dilute sulfuric acid.
In Experiment I, 50 cm 3 of2.5 mol/dm3 dilute sulfuric acid was used.
A further experiment using di lute sul l\Jric acid, Experiment 2, was carried out. The amount
of hydrogen ga~ collected is half the amount of Experiment I.
The results of Experiments 1 and 2 are shown on the graph.

Experiment 1

Experiment 2

Suggest the concentration and volume of acid used for Experiment 2.

tcm3

fa)

Using collision theory, explain why the reaction is much faster when the acid is
warmed before zinc is added to it.
[2]

hydrogen

volume of

(b)

When a few drops of coppcr(ll) sulfate solution is added to the mixture of zinc and
dilute sulfuric acid the rate of reaction increases significantly. Explain in tenns of
collision of particles, why the rate of reaction increases.
[2]

[2]

(c)

Draw the energy profile diagram for the reaction zinc and dilute sulfuric acid . Label
on the diagram the activation energy and the enthalpy change.
[2)

time/s

(ll)

~c)

Experiment 3 was carried out using powdered calcium instead of zinc and 50 em 3 of
1.5 mol/dm 3 sulfuric acid.
The initial rate of reaction for Experiment 3 was faster than for the other
experiments but the reaction stopped suddenly after only a small amount of gas has
been given off.
(i)

Explain why the imtial rate of reaction was faster tl1an for tl1e other
experiments.
[I]

Explain why the reaction stops suddenly.

[I]
(ii}

(a)

(b)
(c)
(d)

(e)

QUEENSWA Y SECONDARY SCHOOL

Preliminary Examina tion 20ll


Sec 4E C hemistry Mark Scheme

(I]
(I]
[I]

[Gr

HX 1s a covalent compound. Small amount of energy is required lo overcome the


[2]
weak intermolecular forces belWeen the HX molecules.

l0)~~- ,.J

80.0
bromine
Isotopes

PAPER 2
SECTION A (SO marks)

Al

A2

[2]
nickel silver
(a) Barium manganese
(i) The blue copper(II) sulfate solution will become colourless and
(b)
pinkish solid deposited .
l21
2
[I]
(ii) Zinc lost electrons to form Zn + ion
[I]
(i) It is very costly to extract titanium from titanium white.
[11
(ii) To conserve or to save the finite titanium resource.

(c)
3

[2)

A3

[1]
[I)
[I]

show working to obtain mole ratio ofC: H: 0 (2.67: 4: 4);


empirical fommla is C2H;Oi:

[I]
[I]

l I]
[2]

No. of moles of NaOH used= 0.00 I;


No. of moles ofH2X in 12.5cm1 = 0.0005:
No. of moles of H2X in 250cm1 = 0.0 I;

960/0.02 = 48000 cm
(a)
48000/24000= 2 mol Hence one molecule give 2 carbon atoms
(b)
(c)
C2H4.
(d) Structural formual of C2H4.
(e) C2H + 302 7 2C02 + 2H20

(b)

150

(a)

(c)

[I]
[I]

[IJ

(I]

[I]

(a)

A4

AS

C is magnesium chloride
I?. is magnesium hydroxide
F i~ magnesium sulfate

A6

A7

A8

(b)
(c)

II

hydroxide ion
[I]
Sulfuric acid and magnesium I magnesium oxide I magenesiurn carbonate
I magnesium hydroxide;
[I j
Procedure : Add excess solid to acid to ensure complete reaction: [J/2]
Filter excess solid;
[ 1/2]
Obtain magnesium sulfate by crystall isation.
[l/2]
[l /21
Filter and dry crystals:

P'4.rr-rM

Aca11C:tcd

C- C-0- H

(a)
(b)

11 -

II

Oxidation of ethanol by bacteria in the air;

(i)
' Structure:

(ii)

When the ratio is about! 5, the burning is the most complete, give out
most heat and the temperature is the highest, hence la rgest amount of
nitrogen will burn in oxygen to form nitrogen oxides.
[I]
Nitrogen and carbon dioxide
[I]

With the increase in the amount of air, the combustion will be more
compl~te wiU1 more oxygen. producing less carbon monoxide.
(I ]

As the ratio increases from I0 to 15, there is a significant decrea~c in the


[ l ].
concentration of carbon monoxide produced
The concentration remains constant when the ratio is greater than 15. (I ]

(I]

The enzyme in yeast would be denatured at high temperatures.


[I]
To allow the carbon dioxide formed to escape and prevent air from
[I]
entering the jar.
C~u06 7 2C2ll~OH + 2C02
LI I
[I]
(e)
(d)

(a)

(b)

(c)

(d)

(a)
(b)
(c)
{d)

(i) chlorine
[I]
(ii)
Cl2 + Wr 7 2cr + Br2
[2]
anode is oxygen cathode 1s hydrogen
[2]
40ff -4e 7 0i + 2H10
[I]
After 2 hours all the Cu2+ has finished discharged at the cathode hence
H+ ion is discharged and produced hydrogen gas at the cathode. At the
anode OH- ions are discharged producing oxygen gas after all er has been
discharged.
[ l]
After 3 hours nearly all the H and OH. ions are discharged left behind
almost pure water which can not conduct electricity.
[I]

SECTION B (30 marks)

[I]

(b)

The melting point increase from sodium to magnesium and remains the same
between magnesium and aluminium.
[l]
It increases again to silicon but decrease in phosphorus. It rises slightly to
sulfur and decrease again in chlorine.
[l]

Aluminium atom has 3 electron shells

(c)

Silicon exist as a giant covalent molecule with extensive covalent bonding


II)
within the structure.
A lot of energy is required to overcome these large number of strong covalent
[I]
bonding which account for its high melting point

B9 (a)

(d)

(e)

(f)

The melting points of oxides of metals are higher than the melting points of
metals.
ll is possible that the attractive forces between metallic ions an<l negative:: iuns
are stronger than the attractive forces between metallic ions and the
surrounding electrons.
[2]
Diagram of metallic bonding of magnesium.
[I]

Sodium oxide, magnesium oxide and aluminium oxide are ionic compounds
with giant crystal lauice. There exist very strong electrostatic forces of
attraction between the oppositely charged ions. A lot of energy is required to
overcome the strong electrostatic forces of attraction.
[I)
Sulfur dioxide and chlorine oxide exist as simple covalent molecules with
very weak intermolecular forces /van der Waals' forces of attraction between
them. I lcncc much less energy is needed to overcome the weaker
[I]
intermolecular forces of attraction.

BIO
(a)
(b)

(c)

volume is 50 cm 3 concentration is 1.25 mol/dm3


[2]
At higher temperature, particles gained heat energy and move with faster
speed hence there will be more successful collisions between the zinc and
[2]
acid particles per unit time.
The catalyst copper(ll) sulfate will help to lower down the activation
energy required for reaction between the zinc and acid particles. Hence
more successful collisions between the zinc and acid particles will occur
per unit time.
[2]

(d) Ea: activation energy

H2

t:.H : enthalpy change

Zn+ H,SO,

t:.H

ZnSO,

Progress of reacuon

[2]

The calcium sulfate salt formed on the surface of the calcium metal
[I]
is insoluble hence prevent any reaction occur.
Calcium is a more reactive metal than zinc.
[I)

l2J

Zinc will ionise out into the solution to form Zn 2+, leave behind
electrons in the metal , built up a negative potential. The copper
metal becomes positive potential Hence a potential difference is set
up .. The few C u 2' ions in the solution will receive the electrons
[2]
from the zinc metal through the wire.
[I]
From zinc to copper.
Bigger reading. Magnesium being more reactive than zinc set up a
[I]
higher potential difference.
Reading is zero. Ethanol is a covalent compound, is a non[I]
electrolyte
Advantages : pollution free, water being the only product [l/2]
Produces a contmuous supply of energy
[1/2)
[l/2j
Disadvantages : source of hydrogen is finite.
Careful storage ofH2 and Oi when mixed, they burnt with
[1/2]
explosion
2H2 + 0 2 7 2H20
II)

percentage yield is 90%

(ii)

(i)

(iv)

(ii)
(iii)

(i)

(ii)

(i)

energy

(e)

Either
Bil (a)

(b)

(c)

Or
Bl I

II

(a)

II

I
0 -C - H
H

[21

[I]

(i) 2CH,COOll + CaCOi -7 (CH1C00)2Ca + C02 + 1120


[2 J
(ii) A strong acid ionized completely in water to fonn W ions, while a
weak acid ionized partially in water to Conn few I t ions.
[I)

I
I
Br
H

H - C - C - C - 0 - CHi

I
Br
H

C = C-C - 0-H

[2]

H H H 0 [1)

H H

COOH

I
I
C-CI
I

H H1NH2C

H H H 0

H H

II

COOH

I
I
-C-C 1
I
H1NH2C

I
I

c=o

0
H

c=o

I
H-N-H

H-C-H H

c--c--

I
I

I I I II
I I I II
--N-C-C=C-C-N -C-c=c-c~1 I
I I

Polymerisation involving two different types of monomers and


small molecules li l.e water is being removed during the process.(1)

(i)

(ii)

(i)

H -

(b}

(c)

(ii}

Or
(i i)

I
I
--c-c
1
I

H-C-H H

I
H-N-H

"

SEC4 t
Total marks 40
JI

CHEMISTRY

PAPER I

MARKING SCHcMES

c
c

21
22
23

D
24
25

26 B
27 D
28 c
29 B
30 B

32

31

B
D
A
B

33

34
35

38

37

40

39

c
c

B
B

36

TIME: I H

QUEENSWAYSECONDARYSCHOOL
PRELU.HNARY EXAMINATION 2011

12

D
A

14

13

D
17

B
D
B

18
19
20

16

D
4

A
B
A
B

15

10

6
7
8
9

REGENT SECONDARY SCHOOL


SECONDARY 4 EXPRESS

PRELIMINARY EXAMINATION (SA2) 2011


PURE CHEMISTRY PAPER 1
(5072/1)

_.llG""~llOlll)lfl!O'ROl'flOf._,TQ~llCIUICf!llG.l -lilg.~l"IGAliftllGll!GnOnGtcIC1-1......
..'111C'_,.llGirtlOr,r~1~11G11Eflalll)l'll(tf----O--O~~

~---ilU !WWW. . . . . . . . . . .

Duration: 1 hour

Date: 19 August 2011

~~31............... . . . . . ..---~-- ................... . . .

Cl-...

ICJ"ll0'90'l01w...,. ....... ,...,_..,CllW~~Mil~llllOt~~QIC;"..., _ _ _..,.~~~C1 t'IO'llOf~

Name:
Class: Sec 4-8

INSTRUCTIONS TO CANDIDATES:
Do not open this booklet until you are told to do so.
Wnte your name, index number and class on the answer sheet in the spaces provided.
There are forty questions in this paper. Answer all questions. For each question, there are four possible
answers, A, B, C and D. Choose the one you consider correct and record your choice in soft pencil on
the OMR.
Read the instructions below very carefully.

INFORMATION TO CANDIDATES:
Each correct answer will score one mark. A mark will not be deducted for a wrong answer.
Any rough working should be done ii this booklet
A copy of the Data Sheet is pnnted on page 23.
A copy of the Periodic Table is printed on page 24.

This paper consists of~ printed pages including this cover page.

rT,,.....,,....._,.,.._.

1.

2.

Section A [40 marks]

Choose the correct answers and shade the corresponding oval in the OMR provided_

carbon

It is to remove carbon dioxide from the gas

It is to remove water vapour from the gas

water

It is to remove hydrogen chloride from the gas

._-_0..- -1

- -<> -

The diagram below shows the apparatus used to obtain the gas carbon monoxide.

dilute
hydrochloric
acid

lleat

calcium
carbonate

It is to prevent water in the trough from being sucked back on to the hot carbon

What is the main purpose of W?

Mercury

Iodine

Ammonium chlonde

Solid carbon dioxide

[Tum over

Which substance could not sublime when heated at room temperature and pressure?

3.

4.

;r

;r

""mofp,o, ,
and

concencrated
t-(2S0 (aq)

le
i
'"
IG

I - :xe

fT11,.n nuor

'le -0 "
:x+"
ie
;x
Beam of P
and

~x

The following mixture of gases, namely hydrogen, carbon dioxide, hydrogen chloride

NaOH(aq)

ooncenlrated

and sulfur dioxide. is passed through the arrangement shown below.

gases

mixture of ---+

water

;x is passed through two

Which one of the following gases will be collected at X?

Carbon dioxide

Hydrogen

Sulfur dioxide

Hydrogen chloride

A beam of proton (p), neutron (n), electron (e) and ion


charged plates.

;r

"
p

Which diagram shows correctly how the particles are affected by Che plates?

;r

Beam of p, n, e
and

Jx+

Beam of p. n, e
and

le
TG
le
TG

.,

5.

The diagram shows the fractional distillation apparatus which was used to separate

Heat

Water out

ethanol (boiling point 78C) and water (boiling point 100C).

Thermometer

30

to:mi)mturd"C

olum: of

distillate /cm3

,..., ,~u

dishlla1e/ cm:c

-'

IOt :

IO t: ::

temperature.fC

Which of the following graphs below best shows how the temperature varied against

t<;":mperaturef'C

JOO

50

lcmpt:telurcl'C

o lumeof
d1st1Ua1e /cm'

the volume of distillate collected?


A

c
100

30

Olumeof
d1shllatcl c.m,

fT11r"

lS

co cP
C{)

0
0 0

0
00
0
iii

co

C{)

Which of the following best describes the substances shown below?

m
ii

vi

~
~
Mixtures

rT1 1m nvP.r

K,O.Al203.6Si~.

~
v

Compounds
vi

c-

.6 <;)' ~
iv

Elements
i, iv, v

ii, ui

A
ii, iii
IV, V

iv

,__
B

ii, Ill, vi

i, v

c
ii, iii, vi

iv, v, vi

The old way of wntmg the formula of a mineral called orthoclase is

What are the values of x , y and z in this formula?

The modem way of writing this formula is K~l.Si,.O..

12
8

.
2

16
3

6
1

2
0

"

<:!.

a'

P"

Particles

28

27

27

Mass Number

12

13

12

13

Proton number

Consider the following data:

R"

29

p 3 and

p 3 and R3

s2

s,.

Which of the particles are isotopes?

1:1

R3 and

Q2 and R"

s2
0

SiC2

S iC

SiC

Empirical formula

burns giving a solid residue and a colourless gas

burns leaving no residue

burns giving a solid residue and a colourless gas

bums giving a solid residue only

when strongly heated in oxygen

" carbon atom

O " silicon atom

The diagram shows part of the structure of the compound, silicon carbide.

Silicon carbide

SIC4

Which set of information aoout silicon carbide is correct?

(Turnover

10.

11 .

12.

An element Z has the electronic configuration 2,8,4. The simplest compound formed

a colourless gas that dissolves in organic solvents to form a non-conducting

when Z combines with hydrogen is most likely to b e - - - - - - - - - -


A

a colourless gas that dissolves in water to form a solution that conducts an

solution
B
C
a solid that conducts an electric current when molten

a sohd non-electrolyte

electric current
D

Solid iodine
Atoms

I
I

Solid calcium iodide


Ions
Atoms

I
I

What particles are present in solid iodine and sohd calcium iodide?

Ions
Ions

-Molecules
Atoms

c
Molecules

-- 0

points. Which of the following can be easily overcome that resulted in this property?

Discrete molecules like hydrogen, oxygen and carbon dioxide often have low boiling

1'3.

14.

8 g of X2 0,, an oxide of element X, contains 5.6 g of X.

16

8 3
-x-

- x16 3

2.4 3
-x16 2

-x16 3

2.4

How many moles of X does 5.6 g of the element contain?

c
D

A mixture containing 1 mole of ethene ano 4 moles of hydrogen were reacted

120 cm3

96 cm,

72 cm3

2'- cm3

temperature and pressure, of the gas(es) at lhe end of the reaction?

completely in the presence of a catalyst. What is the total volume. at room

Weak attractive forces between atoms inside each molecule

What is compound X?

Compound X reacts with powdered zinc carbonate to give an acidic gas.

Potassium hydroxide, KOH

Calci1..m hydroxide, Ca(OH),

One mole of compound X gives three moles of ions in aqueous solution.

Weak electrostatic forces between molecules

Weak covalent bonds between atoms inside each molecule

15.

Weak van der Waals forces between molecules

c
D

&;lfuric acid. H, so.

Propanoic acid, CH, CH2COOH

ffi.

17.

plotted as follows:

A substance X is added to barium hydroxide solution. The change of conductivity is

:J

-~

1S

~ l:
c tt
!~

w8
Amount of substance X added

Sodium chloride solution

Potassium nitrate solution

Dilute nitric acid

Dilute sulfuric acid

What could substance X be?

Magnesium oxide

Copper

Ammonium sulfate

Solid

Gas evolved relights glowing splint

Solid dissolved with no effervescence

Gas evolved ignited with a 'pop' sound

Alkaline gas produced

Observation

(Tum over

Different solids were added to separate portions of warm dilute sulfuric acid. Which of

Zinc carbonate

the following observation is correctly matched to the solid?

18.

Aluminium oxide

Calcium oxide

Zinc oxide

Substance Y

Hydrochloric acid

Hydrochloric acid

Propano1c acid

Propanoic acid

Substance Z

(Turn over

Substance Y was added bit by bit, with stirring to aqueous solution Z. The changes 1n

13

pH of the mixture are shown in graph below.

pH

amount of substance Y added

7 14'-------- ------ - :.;.::. - .

31
0

Sodium oxide

What could Y and Z be?

10

19.

20

The table below shows the results for the tests carried out on solid Z .

Heat a small sample of solid Z


gently in a test-tube
Solid Z was soluble in water. A green
precipitate was observed upon adding
aqueous ammonia.

Water droplets were observed in the


test-tube.

Observations

2
Dissolve solid Z in water and add
few drops of aqueous ammonia to
the solution.

A white precipitate was observed upon


adding silver nitrate solution.

Tests

Add few drops of silver nitrate


solution to a solution of solid Z.

Anhydrous copper (II) sulfate

It

~st redturnslitmus
~~per
blue

Based on the information above. what could solid Z be?

Hydrated iron ('I) sulfate

Hydrated iron ('I) chloride

Hydrated copper (II) sulfate

Blue
precipitate

add excess aqueous


sodium hydroxide
solution !

A solution of fertiliser was tested as shown.

fertiliser
solution

21.

The diagram shows the apparatus used in an experiment to reduce substance Q with
the gas generated in the conical flask. Which one of the following pairs could be P and
Q?

dilute hydrochloric acid

Copper

Lead

Magnesium oxide

Lead (II) oxide

Copper (II) oxide

Zinc

Iron (Ill) oxide

heat

Magnesium

"" D
'-

X, Y and z are metals which form cations, X3, v2 and z respectively.

no visible change

Given the following information.

Z. Y, X

Z.X.Y

Y,X,Z

X, Y,Z

,n_

v2 + z 7

zz + Y 7

yz + 2Z

3y2 + 2X 7 2X,. + 3Y

NH; and N03

NH; and Cu2

-0

The order of increasing reactMty of the metals is - - - - - - - - -

Cu2 and So/-

wannthe
mixture

Fe3 and NQ3-

Which ions must be present in the fertiliser?

23.

24.

25.

Which statement about the production of iron 'rom iron (Ill) oxide in the Blast Furnace

Tl-.e reaction between calcium oxide and silicon dioxide is a neutralization

is not correct?
A

Molten iron floats on the slag at the bottom of the furnace.

Air and coke are added to produce carbon monoxide.

reaction.

B
Carbon monoxide is the main reactant used to reduce 1rcn (Ill) oxide to iron

C
0

The following reactions can occur in the industrial conversion of ammonia into nitr:c

4 NH3 + 502 7
4NO + 6H 20

5N2 + 6H2 0

2N02

+ 6NO 7

The oxidation state of chromium is decreased from +6 to +3.

The solution changes from orange to green.

rn.:rn over

acid. Which one of these reactions shows the greatest change :n oxidation state of

A
3N02 + H20

nitrogen?

4NHl

2NO + 0 2 7

-7 2HN0 3 + NO

Whicr. of the following statements about the reaction below is Incorrect?

The oxidation state of sulfl..r is increase<! from +4 to +6.

3Na2SO, + 2crl + 4H20

Na2SO, is acting as an oxidising agent.

---+

3Na2SOl + Cr20,2 + 81-r

1~

2.

'l.?.

Energy 1s released during the complete combustion of compounds. The table below

C2H,;

Formula

58

30

M,

-4194

.. 2877

.. 1560

!!> H in kJlmol

Compound

C,H,o

86

.. 551 2

Butane

Ethane

c.1-1 ..

114

shows the value of this energy when some compounds are used as fuels.

Hexane

C.Hu

I
Octane

Butane

Ethane

'l'lost energy?

1 g of each fuel is completely burnt. When completely burnt, which fuel produces the

Octane

Hexane

U, V and W as ele ctrodes, immersed in dilute sulfuric acid of the same concentration.

Three electrochemical cells are set up using c~er metal and three unknown metals,

Cu.

+1.11

-0.45

Voltage I V

Cu

Cu

Positive electrode

Cu. V

+2.71

Metals used

Cu, W

Electrochemical cell

The potential d ifferences between the metals are given in the table below.

v
v

w
u
Cu
Cu

w
u

(Turn over

From these results, deduce which of the following correctly list the metals in the order

v
v

of increasing ease of oxidation.

Cu

Cu

u
w
D

14

28.

P'Q ;
/\H = negative

6H =positive

proceeded by way of compound Q , which could be isolated. The steps involved were:

:n \he conveJsion cf compound P into compound R, it was found that the reaction

Q ~ R;

t
energy

progress of readioo

rr....... "'''""'"'

progress Of reaction

-J \

energy

Which one of t~e following energy profile diagrams agrees with these data?

energy

progress Of readion

..Jyl Ji
Q
progress of reaction

"'

29

30.

aqueous sodium th1osulfate.

(I) + S02 (9) +

s (s)

Two experiments were carried out in which dilute hydrochloric acid was added to

~o

100 cm 3 of 4.0 molidm 3 aqueous sodium thiosulfate

100 cm3 of 4.0 mol/dm3 dilute hydrochloric acid

400 cm 3 of 1.0 mol/dm 3 aqueous sodium thiosulfate

400 cm 3 of 1.0 mol/dm 3 dilute hydrochloric acid

Na2S203 (aq) + 2HC/ (aq) -+ 2NaC/ (aq) +


Experiment I

Experiment II

The initial rate of formation of sulfur and the total mass cf sulfur were measured ir

faster i;i I

slower in I

Rate of formation of sulfur

the same in I and II

less :n l

the same i, I and II

Total mass of sulfur formed

Experiment II?

each experiment. How do the results in Experiment I compare with those in

the same in I and II

more in I

the same in I and ti

'----

In the Haber process for the manutact..:re of ammonia, which of the fellowing
statements are correct?

He hydrogen needed can bEs obtained from the cracking of petroleum.

The unchanged reactants are re-circulated.

The nitrogen needed can be obtained from fractional distillation of liquid air.

The catatyst needed 1s vanadium (V) oxide.

II

I, II and IV only

HI
IV

I, Ill 3;'1d IV cnly

I, II and Ill only

I, II, Ill and IV

<O

.:1.

; .:.

A I
y

II

z
z

Ill
.I

lime

T i:ree different solutions of ~ydrogen peroxide were decomposed 1dh tl1e s&me mass

of catalys'. ihe results are shown in the diag -am below.

Volume

cf 0 2

50 cm of 0.5 moVdm3 ~,~ \ac)


3
50 c.m' of 1.::> molldm l-202 (aS}--

25 cM3 of ~ .0 moll d"Tl3 H20, 1aq) ),

ne three solutions useo we~e

'I I

x
x

Which liat matches the grephs correctly with t.'le solution~?

c
y

z
I

The/ dsplace astatine from aqueous sodium astatide.

They ee oxiC'si11g agt.i-ts.

!1'111 n nvAr

Which o'. the follow;ng statements is not true of chlorine, l:romine and icoine?

-h~>

Thy form gaseo:.is diatomic mo1ecules.


forrr ior.ic ccmpcJnds will GroJr 'I elemer.ts.

17

~'S.

'A.

relative atomic mass :s 370. w 1 J t roper.y is bistatine 1:1..ely tc have?

A newiy d;scovered e lement. l.1statine s piaced in Groui; '/11 ot tie Periodic Table. Its

tis a aarl< ~:rF.en gas at room emrcrarure .

II fcrrrs a black compound with sodium.

II is d isplaced by fiuorine fmm potassium b'statide.

II reacts wit, aqueous potassiuir 'od1de.

TJree electrolysis cells ai e set up as show1. oelcw. In all the cells, o:lly caroon

e-ec;--ode!. ae used and the electJolY?es a.-e aqueous s:itutions <:' silVEor nitrate,

Alumin:um nitrate
solutio11

+ .,..::..-----

Copper (11) nitrate


soiuliol'

--

copper (H 11;1..ate and aluminium r.itrate r-.ispecliVl>ly.

.---

Siive1 nitrate
solution

' 08

54

54

Mass of Ag I g

1c

64

16

32

Mass of Cu lg

4.5

27

13.5

Mass of ~Ilg

'

Turn over

'v\'l,.ch or the foilowing correctly gives the masses o' metal deposited at the catho<le of

each cell if O.b nwle of electrons flew u.rcugn ! 1c circui'.s?

54

I
IA I

ol

18

$5.

36.

Methane, carbon dioxide. nitrogen dioxide and sulfur dioxide, are gases which affect

Sulfur dioxide

Nitrogen dioxide

Carbon dioxide

Methane

Gas

Volcanic eruptions

Lightning activities

Bacterial decay

Internal combustion engines

Source

Depletion of ozone layer

Acid rain

Global warming

Photochemical smog

Effect

Which of the following gives the correct source and effect of the gases?

the atmosphere and the environment.

lactic acid is shown below.

Lactic acid occurs naturally, for example in sou.- milk. The structural formula of

"-0-H

c-c-c,,.
I #0

H-'
I HI

Ucan be formed by reacting ethanol with methanoic acid.

It decolourises aqueous bromine rapidly.

It is insoluble in water.

Which statement is true regarding lactic acid?

fT111'rt1"""""

Two molecules of lactic acid react with each other in the presence of a strong

D
aCJd.

10

31.

38

=>old cataly$t. h<at,

HH

C2HsCOOH

C~.,OH

C2HsOH

H H

HH

H-C-C-C-0-C-C-H
I
I
I
I

H H 0

The following reaction scheme shows the reactions of three substances X, Y and Z.

X KMnO,JH', heat

C3H.,OH

CH3COOH

CH3COOH

What are the molecular formulae of substances X, Y and Z?

C2HsOH

C3H.,OH

C2HsOH

~He

C3H.,OH

C3Hs

H
I

//0

Its aqueous solution reacts with sodium hydroxide.

II decolourises aqueous bromine.

' c - C- H

..y

[' 0- H

I
[ ..._ 0

'C~

It turns acidified potassium dichromate from orange to green.

f"T"........ " ' ' ' " '

Aspirin is a drug which is used as a general painkiller. T he full structural formula of

11
[

aspirin is shown below.

' c/ ~c/
H

It is formed from an alcohol and an organic acid.

Which statement about aspmn is Incorrect?

""'

.39.
structure is:

CH3

II
C

CH3

= C -

CHa

c
0

I
= C -

CH3

c -

c -

I
c

II

0- C

II

IT11rn nvP..r

Poly(methyl-methyl-acrylate) is used to make exterior lights of automobile. The

CH3

1
I

H O=C

-C-C-C-CO=C

I
0

0
CH3

CH3

HJC -

H3C -

0 -

CH3

c -

H3C -

What is the monomer unit used to make the polymer shown above?

O=C

I
0

I
CH,
?1

40

The diagram below represents the partial structure of nylon:

- CONH2

-CONH-

-COO-

-0 -

c ,I
4f , ~
: o C(C

(Turn over

.....
x--cJ
-Gm-- X-cJ--- X-Brn-

Which of the following linkages does X represent?

END-OF-PAPER

22

A2.

;
REGENT SECONDARY SCHOOL
SECONDARY 4 EXPRESS

PRELIMINARY EXAMINATION (SA2) 2011

(507211)

PURE CHEMISTRY PAPER 2

A new type of electroplating Is known as 'brush electroplating'.


It Is used to electroplate zinc onto very large iron supports to be used in buildings
The Iron supports are too big to be plated in a normal electrolysis tank.

battery

tl1il coaling ol zinc


forms on surface

The surface of the iron acts as a cathode.


Zinc Ions from the solution form zinc on the surface of the iron
Wnto an ionic half-equation, with state symbols, for this reaction.

Two different designs of metal brush are available.


One type of brush is made from zinc, one type is made from platinum

(1)

[2)

During the process, a metal brush spreads a layer of aqueous zinc sulfate over the Iron
sur1ace. A battery gives the brush a positive charge and gives the Iron support a
negative charge. A layer of zinc forms on the surface of the iron support

(a)

(b)

Whal will happen to the concentration of the zinc ions dunng the
electrolyslS 1f the brush is made from platinum?

(1)

concentration of zinc ions in the solution.

As the electrolysis takes place, each brush has a different effect on the

(i)

What will happen to the concentration of the zinc K>OS dunng the
electrolyslS 1f the brush is made from zinc?

JT,, .._ ,..,,......

(i)

.,

(c)

(d)

(ii~

Platinum brushes are much more expensive than zinc brushes.


However, zinc brushes need replacing regularly but platinum brushes
[2]
do not. Explain why.

Dunng the process, a wori<er needs to hold the brush

Graphite

Iron

Poly(ethene}

[2]

Which of the following matenats would be a good choice for the handle of the

Copper

brush?
Grve a reason for your answer
Chromium

Material
Reason

Explain why Iron supports coated with zinc do not rust, even if the zinc coating
is damaged.
[2]

l'To '"'"' ,..,,..., ...

ll.3
'Lean burn' engines are a type of car engine.

Higher

Operating
temperature

Higher

Concentration
of CO In
exhaust
gases

Amount of air
m ixed w ith
petrol

Less air
Lower

I
Lower

Higher

Concentration
of nitrogen
oxid es In
exhaust gases

This table shows some infonnation about lean burn engines compared to nonnal car
engines.
Type of
engine

Nonnal
Lowe r

How is carbon monoxide formed in a car engine?

More air

(i)

Suggest why lean burn engines produce smaller amounts of carbon


(1)
monoxide.

(1 )

(ii)

Explain how nitrogen oxides are formed in a car engine.

IT11mov..r

(2)

(i)

Suggest why lean burn engines produce smaller amounts of nitrogen


oxides.
(1)

Lean burn
(a)

(b)

(ii)

"

(c)

Another way of reducing the amounts of carbon monoxide and nitrogen oxides
from cars 1s to use catalytic converters.

2C02 + N2

This is the equation for one reaction that happens in a catalytic converter.

2CO + 2NO

[Turn over

Use oxidation states to show that carbon is ox1d1sed and nitrogen is reduced in
this reaction.
(2)

Al"
(1]

[2]

[1]

Ammonium phosphate contains ammonium ions, NH;, and phosphate


ions, PO.lo.
What is the formula for ammonium phosphate?

Ammonium nitrate has the fonnula NH4 N03


Calculate the percentage by mass of nitrogen in ammonium nitrate.

The waste water also contains nitrate ions. Explain why nitrate ions cannot be
removed by precipitation reactions.
(1]

W rite an equation, with state symbols, to show the reaction between calcium
ions and phosphate ions.
[2]

The calcium ions react with the phosphate ions to form a precipitate.

One way of treating the water to remove these phosphate ions is by adding
calcium ions.

Phosphate ions, P04lo, are present in the waste water from the factory.

(ii)

(i)

Ammonia is used to make ammonium phosphate and ammonium nitrate.

What are the essential conditions for making ammonia from nitrogen and
hydrogen?

A fertiliser factory uses ammonia to make fertilisers.


(a)

(b)

(c}

(d}

(e)

The bags of ammonium nitrate feruliser have the following warning printed on
them.

Do not add fertiliser to soil that has been recently


t reated with any lime-containing product.

Why is calcium hydroxide added to soils?

The main lime-containing product used on farms is calcium hydroxide.


(i)

Why is it important not to add ammonium compounds to soils that have


been treated with calcium hydroxide?
[1]

(11

(ii)

A5
~ C2H~(g)

<\H = -124 kJmor1

C- H
348

C- C

436

H-H

time/s

Bond
412

This table shows some bond energy values.

Energy/kJ

Draw the energy level diagram for this reaction.

C2 H.(g) + H,(g)

Ore mole of ethene gas reacts with hydrogen to release 124 kJ of energy.

(a)

(b)

Bond energy (kJmor1)

(21

Using the bond energies given, calculate the bond energy of the C"'C bond. [31

rTtemnuat'

Mass of

Forms

12

Forms an acidic oxide with oxygen

its sulfate solution.

--,

a compound with oxygen.

Information about the particle


and Its compounds

The following table g ives some information about five particles.


Total number of

24

Particle

Forms a compound with the


formula

10

39

electrons

18

81

12

protons

35

64

It Is formed when zinc reacts with

29

(21

(2)

29

[Turn over

Draw the 'dot-and-oross' diagram to show the bonding in t~e acidic oxide of Y.
Show only the valence electrons.
(2)

v
w

Identify the particles V. W, X and Z.

What are the values of wand x?

w,o.
x

It displaces iodine from potassium


iodide solution.

Particle

(a)

(b)

(c)

10

Brand

1.40 g

1-20 g

Mass per tablet

unknown

0.60g

Mass of magnesium
hydroxide per tablet

$0.80

$0.50

Price per tablet

Two brands of anti-acids, X and Y, both contain magnesium hydroxide, as the active
ingredient. The following table lists some information about them.

Mass of magnesium hydroxide in a tablet of anti-acid Y can be determined by the


following method:

(b)

(a)

Calculate the mass of magnesium hydroxide per tablet of Y.

Write a chemical equation for this reaction.

What could be observed at the end-point of the titration?

(3)

(1)

(1)

A tablet of anti-acid Y was crushed to a powder. All the powder was then added to a
conical flask containing 10 cm3 distilled water and a few drops of methyl orange
indicator. The content of the flask was titrated against 1 oa.moVdm3 hydrochloric acid
and it was found that 7.!?_cm3 of the acid was required for the complete reaction.

(c)

(d)

rT,, ...,...,...,,,.. ...

Show by calculation, which of the two brands of anti-acid tablets has a lower price per
[2]
gram of magnesium hydroxide.

..,

se

Section S
Answer th ree questions from this section.
The l ast question is in the form of eitherfor and only one of the alternatives should be
attempted.
Write your answers on the separate writing paper provided.
The total mark for this section is 30.

"'"''iv.OH

filtrate:
colourtess
solution

aqueous
AgN03

grey
residue

IT11rn over

Colourless gas s:
burns with 'pop'
sound

DifuteNH3

white precipitate.
soluble in excess
ammonia solution

~olution

colourless
solution
R

shake with excess water;


filter

substance

Substance X is a mixture of compound A and element B. Compound A is soluble


in water but element B is not soluble. The reaction scheme shows the results of
some experiments on substance X.

dilute HN03 and


aqueous Pb(NO:J2
yellow
precipitate
p
white precipitate Q;
insoluble in excess
sodium hydroxide
solution

white
preicipitate

1'l

(a)

Name the
yellow precipitate P,
white precipitate Q,
colourless gas S.
white precipitate T.

[4]

Name the cation present in the colourless solution R.

(i)
(ii)
(iii}
(iv)

(b)

Write an ionic equation for the reaction of the grey residue from the filtration with warm
hydrochloric acid.
[1]

[1]

(c)

Silver chloride is an insoluble salt.


Outline the preparation of pure. dry silver chloride. starting from solid silver nitrate. [4]

[Turnover

(Total:10]

(d}

14

89

- - --t

The NASA space shuttle uses fuel cells to generate electricity.


The diagram below shows a hydrogen-oxygen fuel cell.

hydrogen In

negative eledrode

water

40H-(aq)

At the positive electrode, oxygen reacts with water as shown.


0 2(g) + 2H20(I) + 4e-

2H20(I} + 2e

electrolyte
positive electrode

At the negative electrode. hydrogen reacts with hydroxide ions as shown.


H2(g) + 20H-(aq) '

G!ve one source for hydrogen and one source for oxygen for use in a fuel cell.

The overall reaction in the fuel cell is the reaction between hydrogen and oxygen to
make water.
\ a)

What is the name of the electrolyte used in the fuel cell?

f\1t0t'

11 J

[2)
{b}

What type of reaction take place, reduction or oxidation, at the positive


electrode? Explain your answer.
[1)

rT1un

(' i

1 t;

(e)

(OJ

Describe one advantage and one disadvantage of usirg a fuel cell to generate
electricity.
[2)

A fuel cell uses 240 dm 3 of hydrogen. Calculate the volume of oxygen needed
and the mass of water formed. All gas volumes are measured at room
temperature and pressure.
[3)

Construct an overall equation for the above reaction.

rT1 11"n n H or

{Total: 10)

[1)

\f'

11':

Nylon and terylene are synthetic polymers.

EITHER
610

0
-

0
11

The&e diagrams show the structures of nylon and terylene.

N-

nylon

II
-

II

C 4 H8

C - N - C6 H 12
I
H

0
II

0
II

II
C - C 4 H8

-cTerylene

0
II

[2)

[6]

II
C - N - C6 H 12 - N I
I

c-~~-c -o-~~-o- c-~~-c-o-~~-o-

Construct a table to show tne following infonnation about nylon and terylene.

Both nylon and terylene are condensation polymers.


Explain what the terms condensation and polymer mean.

(a)

(b)

the formula and name of the type of linkages present m each type of
polymer
the structures of the monomers that react to form each polymer
the formula of the molecule that is eliminated when the polymer fonns

(c)

[2]

fTum over

[Total: 10)

Both nylon and terylene are used to make tents for camping .
One reason that they are suitable for this purpose is that they are nonbiodegradable.
Explain why being non-biodegradable is both an advantage and a
disadvantage.

17

. I

OR

810

Butane is an alkane and butene is an alkene.


(a)

Compare the structure and properties of butane and butene.


Your answer should include a discussion of the similarities and differences
in their

structural formulae,
combustion reactions.
reaction with aqueous bromine.

Your answer should include equations for any reactions you discuss.

(b)

[7]

[Tum over

[Total: 10)

Propane undergoes a photochemical substitution reaction with chlorine gas.


Explain, with the help of an equation, why this reaction 1s a substitution reaction
and suggest why it is described as photochemical.
[3]

18


REGENT SECONDARY SCHOOL
PRELIMINARY EXAMINATION 2011
SECONDARY 4 EXPRESS
CHEMISTRY
ANSWER SCHEME

Potassium manganate (VII)

Iodine

A luminium hydroxide

Mercury

Air

PAPER 2
SECTION A : STRUCTURED QUESTIONS
A1

Barium

11
D

c
23

22

21

35

34

33

32

31

iii

ii

The concentration will remain unchanged

The concentration will decrease


b

Correct formula and balancing - 1 mark


Correct state symbols - 1 mark

1
f

B
12

A
24
D

36

Zinc brushes being an active electrode,


will dissolve into the solution and decrease in size,
hence it needs regular replacing.
Platinum brushes being an inert electrode does not take part in the
reaction, hence it does not need regular replacing .
c

Poly(ethene)
It is a non conductor of electricity hence it would be safe to use as a
handle

!'...;>%;%

Zinc, being more reactive than iron,


will corrode in place of iron

~--'

y,
y,

,.,

1
1

'ff"';'~-:;~

Carbon monoxide is formed from incomplete combustion of petrol in the


engine.

- -+
i

Lean burn engine uses more air for burning, resulting in complete
combustion and hence will produce smaller amounts of carbon
monoxide

1
1

-~~~"'-"'" ~

ii

Temperature in the car engine is very high,


hence nitrogen will burn with oxygen to produce oxides of nitrogen.

*.ff.>M<

Lean burn engine has a lower operating temperature

~#><-~"'""' ~f~"W>'U~

1
D
13

c
25

37

~-~

A
14
D
26
B

38

0
15

A
27

39

zn2 (aq) + 2e- -7 Zn (s)

4
D
16

c
28

40

~~j$4fu:l<t~~~~:f~~~.!J0~'7~~~~

c
17
A
29

SECTION A : MULTIPLE CHOICE QUESTIONS


~....,.....W..o.o;

'!.~'!.

D
18
B
30

7
A
19

......

8
B
20

A2

10

~"'"'

A3

ii

~n-;,.,u..'.;~~-~;

A4

d
e

~~::;:~

A5

..

..

~<:~_"'t'._.: :~--:~~t'W~::l~'.H/tf.?'
,_,_

The oxidation state of carbon increases from +2 in CO to +4 in C02,


hence carbon is oxidized.
The oxidation state of nitrogen increases from +2 in NO too in N2,
hence nitrogen is oxidized.
1

(NH4)Jf'04

450C, 200 atm and finely divided catalyst

i
1

(All correct - 1 mark)

ii

M, ofNH.N03
= 14 + 4(1) + 14 + 3(16)
= 80
Percentage by mass of Nitrogen
= 2(14) / 80. 100%
= 35.0 %
3Ca2 (aq) + 2P0.3- (aq) 7 Ca3(P04)i (s)

Nitrate ions are very soluble in water

Correct formula and balancing - 1 marl<


Correct stats symbols - 1 mark

Calcium hydroxide is added to soils to neutralize the acidity in the soil

Reaction pathway

1 mark for correct diagram


1 mark for correct labelling

:io#Z:;~:~~::-'ni:r~~N ;..;/-::.. ~~; -

Ya

..........

ii

'

Ya

"

......'I'...

~- --,.._:>-...:;.~_,._,_,;,,.,.

The ammonium compound will react with the calcium hydroxide in the
soil to form ammonia gas which will escape into the atmosphere.
Hence resulting in a loss of nitrogen content in the soil.

~::.:.-::;:,. ,~.

Energy I
kJ
C2H (a\ + Hi (! 1)

C2~ {g)

AH = -124 kJ/mol

..

' '"'

-A6

A7

---

Let the bond energy of C=C be x kJ/mol

---

-----

" ' -~

_.,.

~.

_________

Legend
X - electron of Y
0 - electron of 0

-~~--:--=

1
1

----

...__ ~_

Energy change for bond breaking


= 4(412) + x + 436
= +(2084 + x) kJ

---

-~ - ~,,_.

'! _

....

Energy change for bond forming


= 348 + 6(412)

=-2820 kJ

+2084 + x - 2820 -124


x= 612

Hence the bond energy of C=C bond 1s 612 kJ/mol

Mg(OH)2 + 2HC/ 7 MgCl2 + 2H20

Red

1 mark for correct number of shared electrons


1 mark for correct number of unb-Onded electrons

(Every correct answer- Y. mark)

: Magnesium
: Potassium
: Bromine
: Copper

w: 19
x :35

V
W
X
Z

a
b

Correct formula and balancing - 1 mark

t~~:;j't.,.~ ~,)

Number of moles of HCI

=27.5/1000. 1
=0.0275 mo!

From equation,
Number of moles of HC/

---..---------------------- - -Number of moles of Mg(OH)2

Number of moles of Mg(OH)2


= 0.0275/ 2
= 0.01375 mol
M, of Mg(OH)2
=24+2(16+1)
= 58
Mass of Mg(OHh

= 58 0.01375

=0.7975 g

= 0.798 g (3 s.f.)

Cost of tablet per gram of Mg(OH)i in Tablet X


$0.50 / 1.2 0.6

= $0.25
Cost of tablet per gram of Mg(OH)2 in Tablet Y

= $0.80 I 1.4 * 0. 7975


= $0.46

~~!~i:L~'"'t;.::n~~'Y"'-- ~_.,.,. -.......:.~

.-.-...,.... .~-".": :~~- -:--f'!:::*'-~:--/:~y; ..

y.

y.

y.

y.

y.

y.

"'.'.._;_:"'." :'l~- ,",

P : Lead (II) iodide

.-.,

i
Q : Calcium hydroxide

+ (~~~p-..:.::;;~:--

Hence Brand X has a lower pnce per gram of magnesium hydroxide.

SECTION C : FREE RESPONSE QUESTIONS


88
ii

S : Hydrogen gas

Zinc ions
Zn 2 + H2

iii

Zn + 2H '

iv T : Silver chloride

., ,

-"'

'.i"..,-:.~ ,::;:~~:J..,,,\_, _,.0

Y>

-~..:..~;~~:':.;,

:~: ...... , . .- ~ ;;::;:;~=-~~ -

Dissolve the solid silver nitrate into water to obtain silver nitrate
solution.
Mix aqueous solutions of silver nitrate and sodium chloride in a beaker
and stir well.
A white precipitate of silver chloride is formed in a colourless solution
of sodium nitrate
Filter the mixture.
Silver chloride is the residue and sodium nitrate is the filtrate
Wash the residue with cold distilled water to remove any water-soluble
impurities.
Dry between pieces of filter paper to obtain pure dry sliver chloride.

Sodium hydroxide solutiol1

Hydrogen comes from the fractional distillation of petroleum


Oxygen comes from the fractional distillation of liquid air
b

~ii:E~-~;

89

-- (1)
--- (2)

Reduction
The oxidation state of oxygen decreases from 0 in 0 2 to -2 in OH-.
Hence reduction has taken place.

'

4H20 + 4e-

40H' + 4Hi0 + 4e'

(3)

0 2 + 2H20 + 4e
40H'
H2 + 20H- -7 2H20 + 2e
2H2 + 40H" '

'

(2)x 2

(3)+(1) 0 2 + 2H20 + ~ + 2H2 + ~


02 + 2H2 ' 2H20

y.

y.

y.
y.
y.

y.

~~.

1
1

y.
y.

:;..-:::-.:.:.=

.,_.,

.,

2
______ __ __ - ---

From equation,
Volume of H2
Volume of 02

y.

Ya

y.

Volume of 02

= 240 / 2
=~

Number of moles of HiQ =10 mol

I Ya

y.

.I --

I y.

------------- -Number of moles of H20


2

From equation,
Number of moles of H2

= 10mol

= 240/24

Number of moles of H2

M, of H20
= 2(1) + 16
=18
Mass of H20

=18. 10

=180 a

Advantage:
In the fuel cell, the chemical energy from the reaction is converted
directly into electricity.

~-~::;:;....._"'-":l:t-';.,,.-~~"S;~-----;:!~:.;;:~;.:a.1

Disadvantage:
Liquid hydrogen must be stored at very low temperature in an
insulated tank or under very high pressure in a thick and heavy steel
tank
~--~- ' '"':..;..-t:;:~~

810

Nylon
Amide linkage

0
II
-C-NI
H

0
II
H - 0 - C - C.H8

[1/2)

Terylene

Ester linkage

0
II
-C-0-

0 - H

(1/2]

[1)

(1)

H - 0 - C2H4

Both monomers correct


-1 mark

HiO

[1/2]

0
0
II
II
H-O-C-C,.H.-C-0-H

[1/2)

0
II
C- 0 - H

H - N - CsH12 - N - H
I
I
H
H

Both monomers correct


-1 mark

H20

(1 I

[1]

Polymer is the product formed from the joining together of a large number of
small molecules together.
[1)

Condensation means small molecules such as water is eliminated during the


formation of nylon and terylene.
(1]

Formula
of
molecule
eliminated

Structure
of
monomer

Type of
linkage

Polymer

EITHER
(a)

(b)

(c)

Advantage : as it is durable and weather-proof. Hence it is long lasting.


Disadvantage : as it cannot be buried in landfills for decomposition when
discarded alter use.

610

OR
(a)

(b)

Structural
formula

Both butane and butene are


hydrocarbons with 4 carbon
atoms
[1]

Sim ilarities
Butane contains carbon-carbon
single bond while butene
contains carbon-carbon double
bond
[1]

Differences

C.Ha + Br2 7 CHaBr2

I
(1]

[1/2]

Butene reacts with aqueous


bromine in a reaction called
addition reaction.
Red brown bromine is
decolourised.
[112)

CH10 + Br2 7 C.HgBr + HBr


[1/2]

Butane reacts with aqueous


bromine in the presence of
light. This process is called
(1/2)
substitution.

1 mole of butane requires 6.5


mole of oxygen while 1 mole of
butene requires 6 mole of oxygen
(112)

Combustion
reactions

C3H1CI + HC/

Both butane and butene can


react with aqueous bromine
(1]

Both butane and butene can


2C.H10 + 1302 ~ 8C02 + 10H,0
undergo complete
C.H1 + 602 ~ 4C02 + 4H,O
combustion to produce
(1/2)
carbon dioxide and water [1]

Reaction
with
aqueous
bromine

C3Ha + Cl-Cl 7

It is a substitution reaction because 1 or more C - H bonds are broken to form


C - Cl bond. The hydrogen atom is lost from propane and the chlorine atom
takes the place of the h'ydrogen atom, hence hydrogen is substituted by
chlorine.
[1)
Photochemical means that it is a chemical reaction which takes place in the
presence of light. In this reaction, uv light is required as the catalyst for the
reaction to happen.
[1)

Class:_ _ __

1 hour

Marks:_ _ _/40

Preliminary Examination 2011

"BETIER SELF FOR BETIER TOMORROW"

SPRINGFIELD SECONDARY SCHOOL

Name:_ _ __ _ _ _ __

1 1NCI\\"

~-

......

CHEMISTRY 5072/01
PAPER 1
MULTIPLE CHOICE
Secondary 4 Express
15 September 201 1
Additional Materials:
Optical Answer Sheet
Wnting Paper

READ THESE INSTRUCTIONS FIRST


Wnte your name, registration number and class in the spaces at the top of this page
and on all the work you hand in.
You may use a soft pencil for any diagrams, graphs, tables or rough working.
Do not use paper clips, highlighters, glue or correction fluid.
There are forty questions in this paper. Answer all questions. For each questi on, there
are four possible answers, A, B, C and D.
Choose the one you consider correct and record your choice in soft pencil on the
separate Optical Answer Sheet provided.

INFORMATION FOR CANDIDATES


Each correct answer will score one mark. A mark will not be deducted for a wrong
answer.
Any rough working should be done in this booklet.
Hand in the Optical Answer Sheet separately from the Question Booklet
A copy of the Data Sheet is printed on page 23.
A copy of the Periodic Table 1s printed on page 24.

Do not turn over this question paper until you are told to do so.
This question paper consists of 24 printed pages.

The table gives data about four substances.

-80

Which substance has particles in a disorderly arrangement at room temperature?

- 114

445

boiling point I C

120

1407

750

2230

melting point I

1610

-- 1

,_.

T he following diagram shows four balloons W, X, Y and Z which were inflated to


the same size with four different gases: nitrogen, helium, carbon dioxide and
hydrogen gas.

8 8
w

z.x.w. y

W , X, Y.Z

Y,W,X,Z

Z,Y,X,W

After several hours. all four balloons became smaller. Which of the following
shows the correct decreasing order of balloon size?

50 s

40s

30s

20 s

50 cm3 of ethene gas diffused through a porous pot in 30 seconds at room


temperatu re. How long will 50 cm3 of nitrogen take to diffuse through the pot at
60 C?

Urermo.-netor

'

dlSllllate

The following apparatus was used to separate a mixture of three miscible liquids
P (boiling point 78C), Q (boiling point 55C) and R (boiling point 123C).

heat

concentration of R

temperature

concentration of R

temperature

concentration of R

temperature

concentration of R

Which of the following graphs indicates the concentration of liquid R in the round
bottomed flask over time as the mixture is heated from room temperature to 75C?

temperature

()

corn

beans

tomatoes

P,

Corn

Beans

Apricots

P9

artificial additives

._____..__..,

P2

Samples of tinned apricots, beans, corn and tomatoes were tested for additives
using chromatography. The chromatograms were compared with those of three
artificial additives, P1, P2 and P3. The results were as follows.

apricots

Tomatoes

Which tinned food does not contain any artificial additives?

An atom of an element contains 17 protons, 18 neutrons and 17 electrons.

18

17

17 '

18

18

18

20

neutrons

17

17

16

17

electrons

What is the atomic structure of another isotope of this element?

20

protons

Compo und 3

Compound 3

Which of the following correctly identifies the following mixtures and compounds?

Mixture 2

Mixture 2
Water

Compo und 1
Sea water

Com~ound

CD~

Carbon dioxide

8~

Sulfur dioxide

Water

Hydrogen
Carbon dioxide

A ir

Silicon dioxide
Hydrochloric acid

c
Diamond

Silicon dioxide
_,

Poly(ethene)

Methane

Ethane

Ethanol

Which compound contains three elements?

Tl(V03)z

TIN03

'D'

T IV03

Th(VOa)z

The formula of thallium sulfate is Tl2S04 and that of ammonium vanadate is


NHN03 . Which of the following represents the formula of thallium vanadate?

10

11

(i) only

(ii)

N Ne

Which of the following electronic structures is/are correct?

(i) and (ii) only

(i)

(i) and (iii) only

~ '

(i), (ii) and (iii)

.~

c
......

ror

(iii)~
c ~ 2+

X, Y and Z are different elements. The electronic diagram (only va lence electrons
shown) of the compound formed by X, Y and Z is shown below:

l
C

1
2
3

1, 2 and 3

2 and 3

1 and 3

1and2

There is one e lectron in the valence shell of an atom of X. ~


There are five valence electrons in the valence shell of an atom of Y.
There are eight electrons in the valence shell of an atom of Z.

Which of the following statements are correct?

12

13

14

hydiogen molecules in 24 dm3 of hydrogen gas art r.t.p.

hydrogen ions in 1 dm3 of 1 mol /dm3 of aqueous sulfuric acid

neon atc:>ms in 12 dm3 of neon gas at r.t.p.

chlorine molecules in 35.5 g of chlorine gas

Which of the foll001ing contains 1 mole of the stated particles?

40

X)
1ooom' 0-

When 0 .999 g of hydrated aluminium sulfate [Al2(S0.)3nH2 0] was heated,


0.513 g of anhydrous aluminium sulfate was produced.

7
13

Calculate the value of n in the hydrated salt.


A

so

12

?.O

The diagram shows an experiment.

'I

excess of hydrochloric acid

r<-;--------'

~ ~,

\
(_I______cp _-j-me1al

5 g of zinc

5 g of magnesium

5gof iron

5 g of copper

Which metal would fill the syringe with 100 cm3 of gas in the shortest time?

15

Three electrolysis cells are set up as shown below. In all the cells, only carbon
electrodes are used and the electrolytes are aqueous solutions of silver nitrate,
copper (II) sulfate and aluminium nitrate respectively.

aq copper(ll) sulfate

aq aluminium nitrate

,------- ----11 - - - - - - - - - - - .

aq. silver nitrate

54

54

54

54

Mass of Ag/ g

16

64

16

32

Mass of Cu I g

4.5

40.5

13.5

Mass of Al I g

Which of the following correctly gives the masses of metals deposited at the
cathode of each cell if 0.5 mole of electrons flows through the circuit?

16

17

m~tal
Y
meta.I Z

Two cells were set up as shown in the diagram. The arrow shows the direction of
electron flow in the external circuit.

metal Xo = e t a l Y
dilute
sulfuric
acid
Which set of metals would give the electron flow in the directions shown?

Cu

Ag

Ag

Zn

Zn

Cu

metal Y

Ag

Ag

Cu

Zn

metal Z

meta!X

c
Cu

copper

magnes1u

magnesium

Dilute HzSO,

SLogar solut100

copper

copper

Zn

An ammeter can be used to detect electron flow in a circuit.

Distilled water

copper

Which circuit would produce the highest reading on the ammeter?


A

iron

Dilute H2SO,

18

19

20

I 432 I 349

I Cl- Cl I H - Cl I C - C

410

C-H

330

C - CI

243

436

H- H

The values of bond energy for some bonds are given in the table below.
bond
bond energy (kJ/mol)

- 432 kJlmol

+ 432 kJlmol

- 327 kJ/mol

+327 kJ/mol

CH4 + 3Cl2 -> CHCI) + 3HCI

Calculate the heat change (L'.H) for the reaction shown below.

hydrogen and chlorine has more energy than the product.

hydrogen chloride has more energy than the reactant.

the temperature falls during the reaction.

heat is absorbed in the reaction.

The formation of hydrogen chloride from hydrogen and chlorine is strongly


exothermic. From this, it can be deduced that

Which of the following processes is/are exothermic?


Process 1: '2 (g) -> 21 (g)

Iii

1and2 only

3 only

1 only

Oi (g)

-> C02 (g)

Process 2: CuCOa (s} -> CuO (s) + C02 (g)

2 and 3 only

Process 3. C (s) +

10

21

22

1ron(lll) hydroxide is formed.

iron is formed .

Fe2 is oxidized to Fe3 by hydrogen peroxide.

hydrogen peroxide is acting as a reducing agent./

When hydrogen peroxide is added to iron{ll) sulfate solution, the pale green
solution turns brown. This is because

1.0 mol/dm' of aqueous sodium chloride solution. The mixture was then filtered

10.0 cm of 1.0 mol/dm of aqueous silver nitrate was mixed with 20.0 cm of
and the filtrate
collected.
~ -

Silver ions and chloride ions only

Sodium ions and nitrate ions only ,

Which one of the following is true of the ions present in the filtrate?

B
Sodium ions, chloride ions and nitrate ions
Silver ions, sodium ions, chloride ions and nitrate ions

c
D

11

23

24

--pH

The table below gives information about three indicators.


indicator

4.5

blue

yellow

red

6.5

pink

colour in strongly
alkaline solution

yellow

9.0

at which
colour In strongly ~
acidic solution
colour changes

bromothymol blue

colourless

methyl orange

phenolphthalein

Red

Yellow

Orange

Green

If equal amounts of the indicators were added to distilled water, what would be
the resulting colour of the mixture?

A sample of air is slowly passed through aqueous sodium hydroxide and then
over heated copper.

Nitrogen and water vapour

Nitrogen and oxygen

Carbon dioxide and oxygen

Carbon dioxide and water vapour

Which gases are removed by this process?

12

25

26

,, '

,,
/

,/

2H202(aq)

llme

,"""

----------- U
I

2H:10(I) + O,(g)

Curve I is obtained by observing the decomposition of 100 cm3 of 1.0 mol/dm3


aqueous hydrogen peroxide, catalyzed by manganese (IV) oxide.

volume of
oxygen
fomied

'/'

A
Adding some 0.1 mol/dm3 aqueous hydrogen peroxide

Using less manganese (IV) oxide

Which alteration to the conditions will produce curve II?

la
Using a better catalyst
Lowering the temperature

c
0

27

Calcium reacts rapidly with cold water. The element J liberates hydrogen from
dilute hydrochloric acid although it does not react with cold water. When a piece
of J is placed into nickel (II) chloride solution, the green solution turns colourless
and a grey deposit is formed.

Nickel, J, calcium

Calcium, J, nickel

Calcium, nickel, J

J , calcium, nickel

Which of the following gives the correct order of decreasing reactivity of J, nickel
and calcium?

moist Universal Indicator paper

The apparatus was set up as shown in the diagram. When the test tube was
heated, which set of results is likely to be observed?

hydroxide

5
Il

~~~
sodiu~
0

Sulfuric acid

Calcium carbonate

Ammonium sulfate

Aluminium oxide

red

red

blue

blue

28

SolidZ -
heat

Haematite, FeiOJ

Colour of Universal Indicator paper

Malachite, Cu2(C03)(0H)2

Solid Z

Chalcopyrite, CuFeS2

A chemist sets out to identify the mineral present in a particular ore. Treatment of
the m ineral with d ilute sulfuric acid resulted in the formation of a blue solution
and the evolution of a colourless gas which gave a white precipitate with
limewater.
Which of the following is most likely to be the mineral?

Siderite, FeC0J

14

13

29

30

A new e lement Z, thought to be a halogen, has been discovered. Its relative


atomic mass is found to be 370.

It is a white solid which reacts with acid to produce hydrogen gas.

It is a black liquid wh ich boils to give a brown gas.

It is a black solid which has a high melting point.

It is a dark green gas which is soluble in water.

Which of the following statements is a likely property of Z?

iron nail

water

zinc

iron nail

water

water
magnesium

Three test- tubes X, Y and Z were set up as shown in the d iagrams below.
Each iron nail was coiled with a different metal.

copper

iron nail

A
X and Z only

X only

Y and Z only

Y only

In which test-tube(s) will the iron rust?

15

31

Magnesium

Lead (II) oxide

Calcium oxide

coiour!ass liquid

burning in air

oxcess llydrngen

Using the apparatus shown below, dry hydrogen gas is passed over a powdered
solid and then through a cooled U-tube before the excess hydrogen is burned in
air. A colourless liquid collects in the U-tube.

powdered solid

Copper

What could the powdered solid be?

16

32

I
l

,--1

\ Jf' !
L---.J......._.

ta

<

Jl:

The diagram compares the amount of carbon in two steels, P and Q.

amount of

ca't-on

we:igtn

s~rength

"ro"'"

s:rengln

r-1

rJ

L-----

oit

-1
; .
~

[OJ

LJ l_a

LJ:..L.. . . _,___

r;l

t;r, rtls".ess

t:r!ttleness

brttt:oness

bfi:llor:ess

11
.

1 I
-~

EL
L..-~ -

Cl

[J
I

le]

Fl

IL.LJ
1oi n
1ai

--1:J

Which two diagrams correctly compare the strength and brittleness of P and Q?

17

33

34

35

Carbon d i oxide

Global warming

Unburnt hydrocarbo1

Excessive amounts of oxides of nitrogen, carbon dioxide and unburnt


hydrocarbons released from cars can affect the atmosphere and the
environment. In what ways do these gases affect the environment?

Oxides of nitrogen

Global warming ,

Global warming

carbon monoxide

ozone

Pollutant

vehicle engines

detergent

Source

fertilizer

aerosol propellant

methane

nitrogen oxide

--

Photochemical smog

Acid rain

Depletion of ozone layer

Depletion of ozone lay

Acid rain
Acid rain

'

Photochemical smog
Global warming

Depletion of ozone layer

Which of the following pairs of pollutants and their source is correct?

t:.H = -92 kJ

In the Haber process, nitrogen and hydrogen react to form ammonia.

N2(g) + 3H2(g) ~ 2NH3(g)

Increasing the temperature

Increasing the pressure

Decreasing the temperature

Addition of a catalyst

Which factor increases both the speed of reaction and the amount of ammonia
produced?

18

36

37

CH~OH

C02H

C02H

Which compound is unsaturated and forms a neutral solution in water?

CH20H

CH

II

CH
C02H

I
CH2
I
CH2
I

C02H
D

I
CH
II
CH
I

CH20H

I
CH2
I
CH2
I

CH~H
B

Bitumen is a substance obtained from the fractional distillation of petroleum.

low

low

high

high

boiling points

small

large

small

large

sizes of molecules

What are the boiling points and the sizes of the molecules in bitumen?

19

38

1 12Hs1

12Hs 1

The structure of an addition polymer is represented below.

1 2Hs

CH3 H

CH3 H

CH3

-y-1-1-1-1-1I

HFHHH

Which one of the following is the monomer?


A

H H H

c=c-c- c-c-H

H H

H H F H H
I
I
I
I
I
H- c-c=c-c-c- H

H H H H H

H H H

H F H H H

c=c-c-c-c-H
I
I
I
I

H- C-C=C-C- C-H
I
I
I
H
H H

20

39

0-H

I I J I
H-c-c-c-c-H
I I I I

Compound Q has the structure shown.

0-H

I
I
H-C-HH-C-H
I
I

H-C--C-H

Which of the structures below is an isomer of Q?

0-H

I ti I

H-C-C-C-H

H-C-H
H

H H

D
H

c
H

I I I I
H-o-c-c-c-c-H
I I I I

I I I I
H-C-C-C-C-H
I I I I

21

40

/ H
H

:H

~N-H

Below is a diagram of the amino acid, Arginine.

' N

I
H

H- C - C - C - C-N- C
I I I
'N -H

I
he

o 7 ' o-H

It readily decolourises acidified potassium manganate (VII).

It forms a polymer with the same linkage as Tery/ene.

It forms an addition polymer with other amino acids.

It reacts with magnesium to form hydrogen gas.

Which of the following statements about Arginine is true?

End of Paper

22

Class:_ _ _ _ __

Preliminary Examination 2011

Marks:_ _ _/80

1 hour 45 minutes

"BETIER SELF FOR BETTER TOMORROW"

SPRINGFIELD SECONDARY SCHOOL

Name:._~~~~~~~~~~

..

~-

' ''wCl'~"
CHEMISTRY 5072/02
PAPER 2
Secondary 4 Express

25 August 2011
Additional Materials:
Optical Answer Sheet
Writ.Ilg Paper

READ THESE INSTRUCTIONS FIRST


Wnte your name, index number and class in the spaces at the top of this page and on all the work
you 'land in. You may use a pencil for any diagrams, graphs, tables or rough working.
You may use a calculator.
Do not use paper clips, highlighters, glue or correction fluid.
Section A (50 marks)
Answer all questions.
Write your answers in the spaces provided on the question paper.
Section B (30 marks)
Answer all three questions, the last question is in the form either/or.
Write your answers on the separate writing paper provided.
At the end of the examination, fasten any separate answer paper securely to lhe question paper.
INFORMATION FOR CANDIDATES
The number of marks is given in brackets [ J at the end of each question or part question.
A copy of the Data Sheet is printed on page 18.
A copy of the Periodic Table is printed on page 19.

Do not turn over this question paper until you are told to do so.
This question paper consists of 19 printed pages.

A1

Section A

The total mark for this section is 50.

Answer all the questions in this section in the spaces provided.

isotope

325
345

protons

number of

--

neutrons

Define the term relative atomic mass.

S and 12.0% by

32

(2)

. ........ ....... .. ....[1I

electrons

Calculate the relative atomic mass of the volcanic sulfur.

[2)

Complete the table below to show the atomic structure of each isotope of sulfur.

A sample of sulfur from a volcano contained 88.0% by mass of


mass of 34 S.

Sulfur and sulfur compounds are common in the environment.


(a)

(i)

(ii)

(iii)

(b)

(c)

Rotten eggs smell of hydrogen sulfide which is a poisonous gas.


Draw a 'dot and cross' diagram, showing the outer electrons only, of a hydrogen
sulfide molecule.

(2)

(2)
[Total 9)

Every year, between 20 to 50 million tonnes of sulfur are released into the atmosphere
from the oceans in the form of OMS, a compound of carbon, hydrogen and sulfur.
OMS causes the bracing feeling by the sea.
OMS has the percentage composition by mass of C: 38.6%; H: 9.7%;5 : 51%
Calculate the empirical formula of OMS.

A2

The table gives some information about the melting and boiling points of substances, A , B, C,
D and E.

Yes

Liquid

Electrical conductivity

oc

Solid

Yes

B oiling point I

Yes

oc

3570

Melting point I

1083

----A

No

Yes

No

881

No

98

No

716

2230

- 161

-182

1610

Yes
D

Explain, in terms of its structure, how substance A conducts

electric~y .

. ........... ...... ....... (2)

Which substance could be magnesium chloride?


Use the structure of magnesium chloride to explain its electrical conductivity when
solid.

No

1412

Substance

(a)

(b)

......................................................... ..... ..... .. .. . .. .................(2)

(c)

(d)

Explain, using ideas about structures, why substance D and substance E have
different melting points.

.................. .............. ......... ...(3)

[Total: 9]

.. ................ .(2]

Which substance is most likely to be found in Group I of the Periodic Table? Explain
your answer.

A3

(a)

(b)

[2]

Which element has been oxidized in this reaction? Deduce its change in its
oxidation number.

... ... ... ... ... ... . .. ... ... ... . .. .. . .. . . .. ... ... . .. . .. . . . . ...... .. ... ... ... .... .. ... ... ... ... ... ... ... .[1]

Explain in terms of electrons, what is meant by oxidation.

Ba(s) + 2H20 --> Ba(OH)2(aq) + H2(9)

Barium reacts with water in a redox reaction.

(i)

(ii)

element ........

oxidation number changes from .......... to ......... .

Write dolN!l a chemical equation with state symbols for the reaction.

Explain why the electrical conductivity decreases.

[Total:

71

.. .... .......[2]

.. .....[2]

Sulfuric acid was added to aqueous barium hydroxide until the solution was just
neutralized, form ng insoluble barium sulfate and water.
The electrical conductivity of the solution steadily decreased as sulfuric acid was
added.
(i)

(ii)

A4

-->

Pb'2 + 2KN03

An experiment was carried out to study the reaction between aqueous lead(ll) nitrate and
aqueous potassium iodide solutions. This reaction proceeds as follows:
Pb(N03)i + 2KI

4+-----------------..,..,,---,,,,,----,,.,..-

-e 4.s
~ 3.5+-------------~,,,,.,,.--INil---...1@4-----tfil!1-

3
4

5
6

10

.2 3 +------------~~---tn:l~-mltll----ltltil---l!M
H 2.5 +---------~~-4J!tl---4ftl
a.
2 +---------,.,,,..-IW\-.W---1
u
1.5 +-- - - -.........--rza.-m1--mM-- 1
~

0.

o.J-.E=..,.....=~

"o 1 1 - - --.--mn-----im--m:1--rz::a---11::::i
l: o.s --~wa-

"'

volume of lead(ll) nitrate added (cm')

Name the yellow precipitate formed.

. . ....... ... ... .. ..(1)

State the volume of lead(ll) nitrate solution required to react completely with 1O cm3 of
the potassium iodide solution.

.......... ...............[1)

10 cm3 volume of 0.4 molldm3 potassium iodide was placed in each of 10 different boiling
tubes. Different volumes of lead(ll) nitrate solution of unknown concentration were added to
each tube. The mixtures were then shaken and left to stand. In each case, a yellow
precipitate was formed and the height of the precipitate in the test t ube was measured.
The following graph shows the relationship between the height of the precipitate and the
volume of lead(II) nit rate that had been added.

(a)

(b)

(c)

.(1 )

Explain why the height of precipitate formed does not increase after the volume of
lead(ll) nitrate solution stated in {b) has been added.

AS

{d)

II

11

II

II

yH2

CH 3

. ..... ... .. ......[1)

[Total: 4)

[1)

Using your answer from tb) above, calculate the concentration of the aqueous lead(ll)
nitrate solution in mol/dm .

. .. .. ... .. . .

[1)

. . ... ......... .. ........... ....(1)

Suggest another polymer which contains the same linkage that is stated in (c).

Name the linkage found in proteins.

Draw the full structural formula of one of the amino acids.

How many types of amino acids are combined in this part of the chain?

SH

-N-C-C-N-C-C-N-C-C-N-C-C-

HHOHHOHHOHHO

Part of a protein molecule is drawn below.

{a)

(b)

(c)

{d)

... ............... .................................................(1)


[Total: 4)

AS

(a)

methanal
HCHO

Fonnula of
aldehyde
HCOOH

Formula of compound
formed when aldehyde is
oxidised

Name of aldehyde

1
CH3CHO

ethanal

C4H9CHO

C3H,CHO
C.HsCOOH

C3H1COOH

CH3COOH

2
C2HsCHO

pentanal

propanal

5
table to answer the following questions.

[2)

Number of
carbon (n)

The table below shows some information about the first five members of the aldehyde
homologous series.

-in the

Complete the missing information in the table above.

Use the information


(I)

Predict the general fonmula of aldehydes.

oxidized.

[1]

Draw the full structural formula of the compound formed when propanal is

......... ...... (21

Suggest a reagent that can be used to oxidize aldehydes. State any


observation(s) that will be seen.

...... ... ... .. ' ............ ,,. ............. ......................(1 )

(ii)

(iii)

(iv)

{b)

Yoghurt is made by fermenting fresh milk. Enzymes help to convert lactose in the milk
to lactic acid.

The structural formula of lactic acid is shown below:

I I /
H-C-C-C
I OH
I "-oH
H

What would you observe if sodium carbonate is added to lactic acid?

A colourless solution was obtained after the reaction between lactic acid and
sodium carbonate. Upon evaporation. a white crystalline solid was collected.
What type of bond{s) is/are found in this new compound?

(i)

{Ii)

Sugar can be added after fermentabon has taken place to sweeten the yoghurt.
Suggest why the sugar is added after the fermentation stage and not before.

............... ...... ...... ... ... ... ...... ...... .................. ... ... ...... ............. .. ... ... ....(1)

(iii)

[Total: 9)

.. ........................ ' ... ............... ..... '' ..... ' ........... ' ..... .... " ... ...................[1)

10

A7

gases

exhaust

hot
converter

catalytic

to atmosphere

Some vehicles are fitted with a catalytic converter. which changes the exhaust gases into a
more acceptable form of emission.

engine

2CO, (g)

N2 (g) + 0, (g)

-+

Each of the catalysts, which only works at fairly high temperature, bnngs about one of the
following reactions.

-+

2CO (g) + 0, (g)


2NO (g)

gases
_____.. ~talyst~s;r,;o:--__..

Which of the gases is removed (converted) by catalyst B?


...................(1)

...................(2)

Why is it necessary to change nitrogen monoxide into nitrogen and oxygen before
emission?

catalytic converter

air ---+-~~ to
atmosphere

extra

exhau7~~

One design of a catalytic converter is shown below.

(a)

(b)

11

(C)

(d)

(e)

............ ..........(2)

If the volume of the nitrogen monoxide (NO) was much greater than the volume
of carbon monoxide (CO), one feature of the catalytic converter could be removed.

Which part of the converter could be removed? Explain why.

own.

One catalyst 1s platinum, which is coated on aluminium beads. State two


reasons for coating platinum on aluminium be<1ds r<1ther th<1n using platinum on its

.........................(2)

If a large excess of air were drawn into the engine with the fuel, any carbon
monoxide which was formed in lhe hot engine would burn away to form carbon
dioxide. This would avoid the problem of carbon monoxide. Suggest why this is
not done. (Hint Many engine parts in contact with the very hot gases are made of
iron.)

[Total: 8)

. ... ..... ................................................ .......................................................(1)

End of Section A

12

Section B
Answer all three questions from this section.

The total mark for this section is 30.

The last question is in the form of an either/or and only one of the alternatives should be attempted.

BS
Three experiments were carried out to investigate the rate of reaction between zinc and dilute
hydrochloric acid.
The chemical equation is:
Zn (s) + 2 HCI (aq) ~ ZnCl2 (aq) + H, (g)

234

117

Experiment 1

5.0

Calculate the percentage purity of powdered zinc used in Experiment 1.

4.0

-~~~~~-Ex
;..:..
perimem2

---~~~~~~_;::-:_ -=~nment3

3.0

Time/min

2.0

All three experiments were carried out at room temperature (25 C), using excess
hydrochloric acid. The results are shown on the graph below.

(,)

"e
c

..
ei

-g_

.c

'ls
~
0

>

1.0

(a)

Suggest how the conditions might have been altered to produce the results for
[2)
Experiment 2.

In Experiment 1, 0.78 g of powdered zinc was added.

(b)

In Experiment 3, some copper (II) chloride solution was added.


Suggest reasons for the results obtained for th is experiment

[3)

(c)

If Experiment 1 was repeated using powdered magnesium instead of powdered zinc,


how would the rate of reaction change? Give a reason for your answer.
[2]

(3)

(d )

13

69

(a)

(b)

--- Hl

hotoobrut{ll)o.,te.olloo

i' /------
/di
/ syringe
-:::~1~~)tl~~ ..
- graphite rod

---~/-1111 -. ...-..

Cobalt(ll) nitrate is pale red in colour. A dilute solution of hot cobalt(ll) nitrate was
electrolysed in the apparatus shown below. During electrolysis, a grey solid formed on
electrode X and bubbles of gas appeared around the graphite rod. The gas was
collected in the syringe. When the syringe was fu ll of gas, the gas was bubbled
through limewater. White solid formed in the limewater.

!""' " -

~k
=={:t

L
electrode X

Name the grey solid formed at electrode X.


Write an equation for the reaction that occurs at this electrode.

(i)

Name the gas collected in the syringe. Explain the presence of this gas. Include
equations for any reactions that occur.
[3)

<.A_ _ _

~l-,_,:7- (ii)

Describe another observation, not mentioned above, that would be seen as the
electrolysis proceeds.
[11

[2]

(iii)

Lead{ll) iodide was placed in a crucible. The circuit shown below was set up and the
power switc/1ed one The bulb did not light up. The lead(ll) iodide was then heated and
after a few minutes, the bulb lit up. Purple fumes could be seen coming from the
anode while silver coloured droplets around the cathode. Heating was stopped and the
contents of the crucible solidified. The bulb continued to glow.

~------1 111--1

crucible

,.;,,,,,'"'"'

1_____L~
- - _J
1

heat

14

(iii)

{ii)

(I)

Explain why the bulb remained lighted after the contents of the crucible
solidified.

Identify the purple fumes and silvery coloured droplets.

Explain why the bulb lit up only when the lead{ll) iodide was heated.

[1)

[2)

[1)

Melting point/ c

Atomic radi us/ nm

Element

1342

181

0.16

Li

883

98

0.19

Na

760

63

0.24

686

39

0.25

Rb

The table below shows some information regarding some Group I elements.

EITHER
810

Boiling point/ c

The electronic configuration of sodium is 2.8.1.


Write down the electronic configuration for magnesium and aluminium.

The first ionization energy is therefore the energy required to remove the first electron
from the atom; the second ionization energy is the energy required to remove the
second subsequent electron, and so on.

(I)

[Total: 10)

[1)
(ii)

From their electronic configuration, suggest why there is a large increase from
1
the 1 to the 2nd ionization energy for sodium, but a similar increase is only
observed from the 2nd to the 3rd ionization energies for magnesium.
[3)

OR
B1 O Cyclobutene is a hydrocarbon and has the following structural formula.

H H
[1)

[1]

C-H

C=C

C-C

Bond

440

410

600

350

Bond energy
(kJ/moll

16

H-H

The table below gives the bond energies of some chemical bonds.

Cyclobutene reacts with hydrogen to form cyclobutane.

H H

H-C-C
I
I

11

Explain why the atomic radius increases down the group.

[1)

Draw the full structure of cyclobutane.

(a)

Group I elements are all metals with metallic bonding. Explain what is meant by
'metallic bonding'.

(2)

(i)

H-C-C

(b)

In the context of metallic bonding, suggest why the melting point of the elements
decreases down the group.

[2]

(a)

(c)

Group I elements are good reducing agents. Explain why.

(b)

(d)

Al

[2)

578

By means of their structural formulae, write an equation to represent the


reaction between cyclobutene and hydrogen.

Mg

(ii)

738

The table below shows the ionization energies of sodium, magnesium and aluminium.

Na

(e)

496

1817

[2)

Element

1451

2745

State the cond itions required for the above reaction .

First Ionization energy/ kJ

4562

7733

(iii)

Second Ionization energy/ kJ

6910

The ionization energy is the energy required to remove one electron from each atom to
form a positive ion.

Third Ionizati on energy/ kJ

15

(Iv)

(i)

Calculate the enthalpy change. ~H. of the reaction between cyclobutene and
hydrogen.
[2)

[Total: 10)

Using the enthalpy change you have calculated, draw an energy profile diagram
of the reaction between cyclobutene and hydrogen. On your diagram. show
clearly the activation energy and enthalpy change.
[3)

End of Paper

17

DATA SHEET

iron(lll) hydroxide

iron(ll) hydroxide

copper(II) hydroxide

calcium hydroxide

aluminium hydroxide

white

red-brown

green

light blue

white

white

Colours of some common metal hydroxides

lead{ll) hydroxide

white

zinc hydroxide

18

~
17
18
19
20
21
22

c
23
24

D
D

A
A

9
10
11
12
13
14
B
D

Answer

number of

c
B
D
B
B
A

c
B

33
34
35
36
37
38

B
B

B
B
A
39
40

25
26
27
28
29
30
B
A

neutrons

16

16

1
1

Marks

31
32

SPRINGFIELD SECONDARY SCHOOL


BETTER SELF FOR BETTER TOMORROW'
CHEMISTRY 5072
Secondary 4 Express
MID-YEAR EXAMINATION 2011
MARKING SCHEME

A
B
B

15
16

Paoor 1 !40 marks]

A
D

1
2
3
4
5
6

7
8
Paper 2 (80 marks]

'iSolcPe
protons
16

electrons

16

18

16

-..-..n

nunbord

--

1Mlorcarroct

ll\lllng cl

1Mfor~

--

Relative atomic mass is the mass of one atom of


an element compared with 1/12 of the mass of one
atoms of carbon-12.

: 51 .7/32

1
1

GEB
Mole ratio:

H
9 .7/1

(881100 x 32) + (121100 x 34)


= 32.2

I"S
l"'S

Section A: Short-Structured Questions 150 marks)


Questi
on No.
A1
(a)(1)

(a)(1i)

(a)(111)

(b)

(c)

c
38.6/ 12 :

2
6
1
therefore empirical formula = C2 H6 S

A
B
B
A
D

c
A

Total
Marks
9

(b)

Substance A has a giant metallic structure


consisting of regularly arranged and closely
packed 12osit:ve metal ions 1n a sea of mobile
electrons
The mobile electrons can move to tal'.!Y electrical
charges to conduct electricity.

A2
(a)

(c)

Substance D has a sim12le molecular structure.


Little heat enem:r: is ~uired to overcome the
weak Van der Waals forces of attraction between
the molecules
Hence, the melting and botlng point is low_
Structure E has a giant covalent structure.
A large amount of heat energy is required to break
the strong CQvalent bonds between the atoms.
Hence the melting and boiling point is low_

Sodium chloride has a giant ionic crystal lattice_


When solid, the ions (Na and Cr} are held in fixed
positions and cannot move.
There are no mobile ions to conduct electricity.

(d)

Substance B 1s most likely to be found in Group I.


It condu!<t~ ele!<trici!,y both when solid and l!guid, so
it is a metal but it has low melting 12oint and boiling

122i.!J..!,

y,

'h
'h

'h

'h

Y..

y,

Yz

'h

y.
y.

'h

y.

y.
y.; y.

Oxidation is t he loss of electrons.

A3
(a)(i)

~;%

O to+2
1;1

Barium

Ba(OH)2(aq) + H2SO.(aq)-+ BaS04(S) + 2H20 (I)

(a)(ii)

(b}{i)

(b)(ii)

lead(ll) iodide

Concentration of ions 111 the solution decreases as


!!JSoJ\,Jble barium sulfate is formed when the barium
hydroxide has completely reacted leaving only
covalent water molecules.

(a)

8 cm3

A4
(b)
(c)

Limiting reactant is potassium iodide which


determines the yield of lead(ll) iodide formed

(0. 002x1000)18
0.25mol/dm3

Three

OR

OR

(d)

AS
(a)

H H 0
I
I II
H-N- c - C-OH

Amide linkage

(b)
I

H H

SH

CH2

I I
II
H-N-c-c-OH

HHO
I I 11
H- N-C-C-OH

(c)

Nylon

CH 3

(d)

._ .. :

A6
(a)(i)
(a)(1i)
(a)(iii)

(a)(iv)

(b)(i)
(b)(ii)

C2HsCOOH/ Butanal
C,..,H~1CHO

Acidified potassium dichromate Nil turns from


orange to green OR
A cidified aqueous potassium manganate (VII) turns
from purple to colourless.

~ ~

H H 0
I
I
ii
H-C-C-C-0-H

Effervescence

Ionic and covalent bonds.

(c)

(b)

(a)

The need for extra air could be removed.


If the volume of NO was much greater than CO, a
larger volume of CO would be produced and can
react with the smaller amount of CO to produce
C02.

Carbon monoxide

NO will .combine further wrth oxvqen in the arr to


form NO~ that will dissolve In rain water to form acid
rain.
Note: can mention about respiratory problems

It is to prevent sugar from being broken down by


yeast to alcohol.

(d)

T he surface area of the platinum would be


increased.
By coating the platinum on the bead, ii also ensures
maximum contact with the gases.
(Altemattve ans platinum is expensive}

A7

(e)

The engine parts which are mainly made of iron will


rusVcorrode easily because carbon dioxide is an
acidic gas.
(Alternative ans: iron may corrode/rust when
eXQ()Sed to oxygen and moisture m excess air)

}S; 1'2

Rate of reaction would be faster.

A displacement reaction takes place when copper (ti)


chloride solution is added.
Zinc reacts with COPl"/er (ti) chloride to form coQper
and zinc chloride.
The mass of zinc th_<?_t react with dilute hydrochloric
acid is less. than that used in Experiment 1, so the
final volume of hJldrogen gas produced in
Experiment 3 is lower than that in experiment 1.
The concentration of dilute hydrochloric acid
reacting with zinc is also lowered, so the initial rate
of reaction is slightly lower than that in Experiment l.

The final volume of hydrogen produced is halved, so


the mass of zinc used
is halved (0.39 g of zinc has been used)

The initial rate of reaction is lower, so lumps of zinc


could have been used
(instead of powdered zinc).
OR A lower concentration of dilute h)'.drochloric acid
could have been used.
OR Temperature of acid used was lower.

= 0.663375/0.78 x 100 =81.3 %

=mass of pure zinc! mass of impure z inc x 100 %

% purity

mass of zinc used = 0.0975 x 65 = 0.63375 g

No. of moles of zinc = 0.00975

No. of moles of H2 = 0.234 I 24 =0.00975 mol

Section B: Free-Response Questi ons (30 marks)

BB
(a)

(b)

(c)

(d)

Magnesium is more reactive than zinc, so it would


react more readily with dilute hydrochloric acid.

1
1

1
1

10

(a)(i)

Cobalt
Co2 (aq)+ 2e ~Co (s)

B9

(a)(ii)

Carbon diox de
Hydroxide ions are discharged at the anode to
produce oxygen;
Oxygen reacts with the graphite anode to form
carbon dioxide;
40H. (aq) ~ 2H20 (I) +02 (g) + 4e
C (s) + 02 (g) ~ C02 (g)

The cobalt(ll)nitrate solution will turn from pale red


to colourless.

'h

y,

y,
'h

!DJ~-~.ll_Q!i9__~_@.t~Jg_~9_{1J})QQ)Q~_Y!!!.~Jl..Q[ able !Q
conduct electriciiy as its ions were he!d in fixed
oositions .
W hen heated lead! ll!iodide becomes a llguid and
their ioos are able to move about to conduct
electricity.

'h
'h

(a)(iii)

(b)(i)

Purple fumes: iodine


Silver droplets: lead

(b)(ii)

(b)(iii)

Lead(ll) ions that were discharged at the cathode


formed lead metal which is able to conduct
electricity.

1
(a)

610

The number of electron shells increases down


the gro up, therefore the atomic radius increases.

(b)

Group I elements have only one valence electron in


its outermost shell. Hence
it loses/donates this valence electron readily, and
are thus good reducing agents.

Going down the group, the atomic radius/size


increases . As the atomic size increases, t he
valence electron Is f urther awall from the
positive nuc leus.

Metallic bonding is the (electrostatic) force of


attraction between the delocalized electrons and the
positive metal ions.
(c)

(d)

(e)(i)

10

10

::>

e(ii)

810
(a)

(b)

Mg : 2.8.2 Al: 2.8.3

-H

IJ

ti-..{;--{-- ~
I

f1-...c-c-H

1-f

T'~~<l~;.<y
\
.

I
- -
-?!~111(.tk)o

i'JOQr\lfll.I

-~

Sodium has only 1 electron in its valence shell.


The second ionization energy for sodium involves
removing an electron from a full electron shell. This
results in a large increase in the ionization energy.
Simila~ since magnesium has 2 valence electrons,
their 3 ionization energies involve removing an
electron from a full electron shell. Therefore large
increases in ionization energies are observed then.

H-{:-{:..
H H

H ....H.-

H-C..-G- H

H.

'1' 'i'
-?--n
i.--v-r
H
t:.)'--~'!'A"">.;i~

200 C, nickel catalyst

c,...,..AM...~

(c)

=-130kJ

AH= 600 + 440- (350 + 410 x 2)

I Allow ecf
(;>~:~ei~:l"'rnu (

lly.;ro9~r-

........,..,. !

- -1-

/',1;i~~~t!~~

(d)

(e)
.1
l.:- K>l(I}'

- - ...

,d'~.:.-. L.-~!~:':!"u
--------

End of paper

Y.

'h; y,

I
Y.
1

1M correct
formula of
reactants
1M for

correct
tormulaof
products

1 ;1

1Mforl!.H

1Mlo<Ea

l>Qher than
products and

1M fof ractatn

piollle doa\;l'am.

ar.oe ot .-.rgy

10

CF3CHFOCHF2

(CF3)2CHOCH2F

CHF20CHCICF3

CFaCHCIBr

formula of the anesthetic

24

59

49

50

boiling point in C

1-

-0

-+---
j

start line

, 3.6cm

solvent
front

The results of a paper chromatography experiment shown below were obtained.

Which of the following anesthetics does not require warming in the operating theatre to turn
it into a gas?

The temperature in an operating theatre is usually above 30 C.

Many new anesthetics are based on alkanes in which some of the hydrogen atoms are
replaced by fluorine (F), chlorine (Cl) and bromine (Br).

Anesthetics are used to put patients to sleep before an operation.

St. Anthonys Canossian Secondary School


Chemistry
Paper 1
2

dyeQ
dye P
!2.4cm

ink sample

---e;:::: +~--------..t.......
.....................

porous tube
containing
nitrogen and
oxygen

beaker

hydrogen-

Two expenmental set-ups used to demonstrate diffusion of gases are shown in the
diagram below. What changes, if any, to the water levels at P and Q would you expect
to see in both experiments?

carbon dioxide beaker

porous tube
containing
nitrogen and
oxygen

experiment 2

Experiment 2

P and Q remain the same

Experiment 1

experiment 1

a remain the same

Q is higher than P

a is higher than p
a is higher than p

P and Q remain the same

P is higher than Q

P and

a is higher than p

B
D

Sodium nitrate crystals can be separated from sand using the four processes shown
below. Which of the following shows the processes in the correct order?

evaporate
crystallize

filter

crystallise

crystallise
evaporate

crystallise

evaporate

evaporate

last

dissolve

filter
filter

first

dissolve

Given that the R1 value of dye P is 0-40, determine the Rt value of dye Q.

0.50
0.60

filter

dissolve
dissolve

.a

A
0.70

(Turn over

0.80

SACSS Prehminary Exammbon II 2011

CHS072/01

CHS072/01

SACSS Preliminary Examination II 2011

C
D

The tubes below contain a dilute solution of a solid G dissolved in a liquid H. Which
set up is most appropriate for finding the boiling point of liquid H?

The electronic structure of two atoms P and Q are shown below.

AtomQ

~
~ @
Atom P

10

NCh

C02

C2H4

Which of the following molecules contains six bonding electrons?

SFs

B
D

Which ion has the same number of protons as the hydroxide ion?

F
Mg2

0 2

Na

A
C

B
D

An atom of element X has a proton number of 6 and nucleon number 16. Which of
the statements about X is correct?

X is likely to form four covalent bonds and a giant covalent structure.

An oxide of X is likely to be basic.

The formula of a molecule of X is likely to be X2.

The formula of the compound formed by X and hydrogen is XH2.

13
D

ionic bonding

type of bond;ng

102

70

43

mass of one mole of compound I g

C.HaNa

C3H6Ns

C2H4N4

CH2N2

Melamine is a plastic. It has an empirical formula of CH2N2. If its relative molecular


mass is 126, what is its molecular formula?

covalent bonding

113

11

A
ionic bonding

When these two elements combine chemically, what will be the type of bonding and
mass of one mole of the compound formed?

B
covalent bonding

CH5072/01

[Turn over

One mole of each of the following compounds is burnt in excess oxygen. Which
compound w ill produce two moles of carton dioxide and two moles of steam only?

C3Ha ~
C2HsOH

C2H

C2HsCOOH

A
C

B
D

SACSS Preliminary Examination II 2011

12

>----c

MgO
Na20
P4010
Si02

CH5072/01

Fibre glass can be considered to be a mixture of ionic oxides and giant covalent
oxides. Which of the following is not a constituent of fibre glass?
A
B
C

SACSS Preliminary Examination 112011

13

14

15

16

48 g
96 g

6g
12 g

6
What is the mass of magnesium which completely reacts with 500 cm 3 of
1.0 mol/dm3 sulfuric acid?

c
D

2 H202 7

02 + 2 H20

68 g of hydrogen peroxide decomposes in the presence of manganese(IV) oxide to


give 1.2 dm 3 of oxygen gas as follows.

15 0%

100%

5.0o/o

2.5%

What is the percentage purity of the hydrogen perox de?

silver nitrate

salt

add excess silver to warm dilute nitric acid

method of preparation

Which of the following salts 1s incorrectly matched with its method of preparation?

titrate aqueous ammonia with dilute sulfuric acid


mix aqueous lead(ll) nitrate and dilute hydrochloric
acid
add excess iron(lll) oxide to warm dilute sulfuric acid

ammonium sulfate
- -lead(il) chloride
_
lron(lll) sulfate

c__
D

A
B

iron (Ill} hydroxide


lead (II) hydroxide

calcium oxide
copper (II) hydroxide

CH5072/01

Which of the following dissolves in water to form an alkaline solution?

C
D

SACSS Preliminary Examination II 2011

17

18

19

20

21

aqueous silver nitrate and aqueous zinc chloride


aqueous zinc sulfate and aqueous aluminium nitrate

aqueous barium hydroxide and hydrochloric acid


aqueous magnesium nitrate and aqueous copper (11) sulfate

Which of the following solubons, when added together, would produce a white
precipitate?

c
D

ZXY

vzx

YXZ

XYZ

X, Y and Z are elements in the same period of the Periodic Table. The oxide of X is
amphoteric, the oxide of Y is basic and the oxide of Z is acidic.
What is the order of increasing atom1c(proton) number for these elements?

Group I of the Periodic Table_ Which statement is

Caesium cannot displace rubidium from aqueous rubidium chloride.

in

Caesium carbonate decomposes readily to form carbon dioxide.


Caesium is extracted by the electrolysis of aqueous caesium chloride.
Caesium reacts more vigorously with water than rubidium.

Caesium is below rubidium


most likely to be correct?

B
C
D

pressure
1 atm
1 atm
200 atm
200 atm

temperature

vanadium(V) oxide
iron

catalyst

vanadiumM oxide

iron

4so c
1000c
4so0 c
450c

In the Haber process for the manufacture of ammonia, what are the usual operating
conditions?

A
B

r---c
D

adding potassium to water.


reacting magnesium with steam.
warming aqueous sodium hydroxide with aqueous potassium nitrate_

All the following reactions produce at least a gas except

electrolysing concentrated aqueous sodium nitrate using graphite electrodes.

[Turn over

A
B
C

CH5072/01

SACSS Preliminary Examination 11 2011

22

23

24

25

The lime and ammonium sulfate react together to form insoluble calcium
sulfate.

Nitrogen in ammonia gas is lost from a reaction of lime w ith ammonium sulfate.

The lime makes the ammonium sulfate insoluble in water.

Which is the best explanation why farmers should not add lime (calcium oxide) and
ammonium sulfate to the soil at the same time?

The lime makes the soil alkaline so that plants can no longer absorb
ammonium ions.

Refer to the various graphs of volume of hydrogen gas given off against time from
the reaction between magnesium and hydrochloric acid under different conditions.
Magnesium is the limiting reactant in the reaction and curve X is obtained under
normal circumstances.

A ,,,._;;;~-

,,;

, /'

curveX

C.....;-------

,.,, ,.,.,,...

---/ -~---------------,,"'

/:/s

Which of the graphs would show that hydrochloric acid was diluted to half its
strength but ii was still in excess?

Volume
o1
hygrogen
gas

,.1 /
/,

,,r,
,,,

~/

~;.. / /

~~~~~~~~~~~~~~~~--tTime

Dilute sulfuric acid reacts with copper (II) oxide to form copper (II) sulfate and water.
W hich factor would not alter the rate of this reaction?

The pressure at which the reaction takes place.

T he concentration of the sulf uric acid.


The size of the particles of copper (II) oxide.

The temperature of the react ing mixture.

B
D

Cu(N03)2 + 2H20 + 2N02

CuS04 + 2H20 + S02

MgC/2 + 2H20

MgC/2 + Hz

Cu + 4HN03 7

CH5072/01

Cu + 2H2S04 7

Mg(OH)i + 2HC/ 7

Mg + 2HC/ 7

Which one of the following is not an oxidation/reduction reaction?

A
B
C
D

SA.CSS Preliminary Examination II 20 11

26

27

Hi02

-2Ag

+8

+6

+4

+2

HzO

What is the role of hydrogen peroxide as shown in the reaction?

a dehydrating agent

Ag20

A
B

a reducing agent

an oxidising agent

a catalyst

What is the oxidation state of uranium in KzUFe?

initial mass of nickel fg

solutions of metals P, Q , Rand S.


solution

02

mass of nickel after


15 minutes lg
5.0
5.5
4 .5
0.0

28 The table shows the results of adding weighed pieces of n ickel metal in salt

6 .0
6.0
6.0
6.0

Q
R

Metal R can displace the metal in solution S.

W hich one of the statements A, B , C or D below is correct?


A

The metal P is higher than metal Q in the reactivity series. ,

S could be nickel(ll) chloride solution.

The metal R is higher than nickel in the reactivity series.

CH5072/01

[Turn over

SACSS Preliminary Examination 11 2011

29

30

31

10

- 2L O
LC/2 +
Hi
L
+ H20

An element L reacts in the following ways.


2L
+ 02
L
+ 2HC/
LO
+ H2

magnesium

iron

copper .

calcium

Which of the following could be element L ?

The position of a metal M in the reactivity series is as shown.


K. Na, Al, Zn, M, Fe, Pb, Cu, Ag

reduction of its oxide by heating with coke

reduction of its oxide by heating with copper

electrolysis of its molten oxide

electrolysis of its aqueous sulfate

Which method will be used to extract M from its ore?

The electrolysis shown in the diagram is set up.

aqueous

potassium

:x:incentrated

solution turns brown

bubbles of colourless gas

bubbles of colourless gas

at positive electrode

silvery droplets

bubbles of colourless gas

solution turns brown

bubbles of colourless gas

at negative electrode

IOdde

solution turns brown

What is observed after some time?

CH5072/01

SACSS Preliminary Examtnallon II 2011

32

33

11

. - -----11--- ~ 1 -- - .

The apparatus shown below was set up.

carbon
electrode

ethanol

bulb

carbon
electrode

water

[Turn over

Which substance when added in small amount to the water will make the bulb glow
brighter?
A

sodium

lead
sugar

B
D

Cl

CH5072/01

Which of the following changes is exothermic?

A
8

2H + 07 H20
2H
H2 7
H20(1) 7
H20(g)
H

HCI

7
D

SACSS Preliminary Exam111ation II 2011

34

35

12
The diagram shows substances and processes in the lime cycle.

\lcin...

Caldum~ide

CaCOl ~Ci

Calcium carlrnuate

Cob.,..tiM

Call'lum bydroxkle

~o~~
Slaking
The words calcining, slaking and carbonation are used in the lime industry to name
the processes involved.

A
carbonation

calcining

Which of the following parts of the lime cycle contributes to global warming?

slaking
slaking and carbonation

To reduce atmospheric pollution, the waste gases from a coal-burning power station
are passed through powdered calcium carbonate.

carbon monoxide, CO

Which waste gas will not be removed by the powdered calcium carbonate?

A
nitrogen dioxide, N02
sulfur dioxide, S02

CH5072/01

phosphorus (V) oxide, P205

Preliminary Examination II 2011

SA~SS

36

37

C=C /

Ol,

'H

+ J,

13

I I
CH -C-C-CH
I I
l

Iodine (12) reacts w ith unsaturated hydrocarbons as shown in the equation.

'

CH3

H/

The iodine number is the number of grams of iodine that will react with 100 g of the
unsaturated hydrocarbon. The higher the iodine number, the more unsaturated the
hydrocarbon is.

3 g of iodine are added to 1 g of vegetable oil. When the reaction is finished, 1.2 g
of iodine are left unreacted.

120
180
240
300

What is the iodine number of the 011?

@
0

The melting points of alkanes vary inversely with increasing molecular masses.

All alkanes are gases at r.t.p.

Which of the following statements correctly describe the physical property of


alkanes?

CH5072/01

[Turn over

The percentage mass compositions of carbon in alkanes are 50%.


The viscosities of alkanes increase with increasing molecular masses.

SACSS Preliminary Examination II 2011

Br

Br

Br

Br-C-C - H

llandHI

I and 11
I and IV

Ill

H-C -C-H

A
B
all of them

A
B
HOOC - CsH - COOH

H2N - (CH2)s - NH2

CH3(CH2)sNH2

HO~OH

14

The structure of a polymer 1s shown below_

CsHsCOOH

Br

H-C-C -

Br

Br

II

I
H

H - C - C - Br

H
IV

Which of the following monomers reacts with the monomer below to form a synthetic
fibre?

38 Which of these molecules are isomers?

39

40

CH3

CH3

CH a

-CH2-CH-CH2-CH-CH2-CH-

CsH10

CaHe
C.H5

C2H4

CH5072/01

Which is the molecular formula of the monomer?

B
C

SACSS Preliminary Examination II 2011

15

DATA SHEET

white

white

[Turn over

Colours of Some Common Metal Hydroxides

aluminium hydroxide

calcium hydroxide

copper(ll) hydroxide

red-brown

green

white

hght blue

iron(lll) hydroxide

white

iron(ll) hydroxide

zinc hydroxide

CH5072/01

lead(ll) hydroxide

SACSS Preliminary Examination II 2011

St. Anthonys Canossian Secondary School


Chemistry
Paper 2

heat

Gas R

Black solid Q

Ca(OH);(a

Gas R

[41

Ca(OH)2 (aq)

White
precipitate S

Write the ionic equation, including state symbols, for the reaction of P
with HCI (aq).

S:

T: ..... .... ... ......... ... ... ..... ....... ...

Q : ... ..... . ................................ .

P: .. ....... ............ ......... ............ .

Identify P, Q, T and S.

Blue
solution T

Compound P is a pale green powder. The scheme below shows some reactions
of a compound P.

(a)

(b)

CH5072/02/A

...............................................................................................(21

_,

SACSS Preliminary Exam II 2011

I_

For

Use

Examiners

2-

.... ......... ... ... [2]

(3]

Calculate the total volume of gas(es) left when 6 dm of carbonyl sulfide


3
and 6 dm of oxygen are burnt. (All volumes of reacting gases and
products measured at room conditions.)

Write a chemical equation, including state symbols, for the combustion


of carbonyl sulfide in oxygen.

The gas carbonyl sulfide (COS) burns in air (oxygen) forming carbon dioxide
and sulfur dioxide only.
(a)

(b)

(c)

-+ 4H

+ SO

+ 2Cl-

When sulfur dioxide is passed into chlorine water. the following reaction
takes place,

so2 + Cl2 + 2H20

C H5072/02/A

[Turn over

................ .... ..... .......................... .......... (21

Which substance is oxidised in the reaction? Explain in terms of oxidation


state of a named element concerned.

SACSS Preliminary Exam II 2011

For
Examiner's
Use

Ihm
layers

6
Steel is dipped into molten zinc to prevent it from rusting.
The zinc combines with the iron in the steel to form layers of zinc-iron alloys.

,~

air

"'. . .

. . ..

Pure 1.tnc. 70 rN

+--94% 1foc,6irun, 179HV

The diagram shows a cross-section through the layers.


HV is a unit of hardness. The higher the HV number, the harder the metal.
surface

f ..

1 . -. '. : .' ; -: . .' ; -

I..

CH5072/02/A

_244HV
75%zinc,25%iron,25QHV

Explain why steel is harder than pure iron.

... ........................... ................... .......................................... ......(2)

Steel car bodies are coated with zinc before painting. Explain why
the car bodies do not rust, even if the paint is damaged.

......... ... .... ,, ...... ................................................... ..[2)

Use information from the diagram to deduce a relationship between


the hardness of the alloy and the composition of the alloy.

i="i-99.?'-tt.>a.O.l >"... 15?IIV

I ~-"'"'"'' ....

-{

(a)

(b)

(c)

SACSS Preliminary Exam II 2011

For

Use

l:xammat's

For
Examtnet's
Use

[Turn over

.. ............................................ ................................ ..................(2)

CH5072/02/A

Solution P is an aqueous solution of sodium thiosulfate.

SACSS Preliminary Exam 11 2011

For

i:xamt!HYS
Use

Solution Q is dilute hydrochloric acid.

-+

2 NaC/ (aq) + S02 (g) + S (s) + H20 (/)

When a solution of sodium thiosulfate is mixed with hydrochloric acid a reaction


takes place and a fine suspension of sulfur is formed in the solution.
Na2S203 (aq) + 2 HC/ (aq)

volume of PI
cm3

volume of
water I cm3

5.0
5.0
5.0
5.0
5.0
5.0

cm3
8
15
25
41
64

time/s

volume of QI

Experiment

0.0
10.0
20.0
25.0
30.0
35.0

89

50.0
40.0
30.0
25.0
20.0
15.0

Explain why the total volume of solution used in each experiment is


kept constant

......................... .. ......... (2)

Explain, in terms of colliding particles. why the rate of reaction


decreased.

.................. (2)

Use the information in the table to show how the rate of reaction
changes when the volume of P decreased.

1
2
3
4
5
6

In an experiment, a beaker containing the reaction mixture is placed over a


piece of paper marked with an x. The sulfur as it forms slowly hides the x
from view. The time taken for the "X" to disappear from view is tabulated below:

(a)

(b)

(c)

CH5072/02/A

........................ .........................................................(1)
A student set up a cell as shown, to make electrical energy.

SACSS Preliminary Exam 11 2011

For
Examiner's
Use

zinc rod

::~:~=~=~:~=~=~=~=~=~=~:
:~~~~~~~~~~~~~~~~~~~~~~:

----~------------~----

copper rod

aqueous copper (II) nitrate

Label on the diagram the cathode and the flow of electrons in the
external wire.

::::::::::::::::::::::::::::::::::::::::::::::::::1
(a)

How would you expect the read ing on the voltmeter to change if the
zinc rod is replaced by an iron rod? Explain your answer.

Mendeleev devised the modem Periodic Table.

CH5072102/A

[2]

[Turn over

. ............... ... ..................... . ........................... .. ......... .............. ....(2)

(b)

(a)

SACSS Preliminary Exam 112011

For

Use

Examiners

Solution Q is dilute hydrochloric acid.

-+

2 NaC/ (aq) + S02 (g) + S (s) + H20 (/)

When a solution of sodium thiosulfate is mixed with hydrochloric acid a reaction


takes place and a fine suspension of sulfur is formed in the solution.
Na2S203 (aq) + 2 HC/ (aq)

volume of PI
cm3

volume of
water I cm3

5.0
5.0
5.0
5.0
5.0
5.0

cm3
8
15
25
41
64

time/s

volume of QI

Experiment

0.0
10.0
20.0
25.0
30.0
35.0

89

50.0
40.0
30.0
25.0
20.0
15.0

Explain why the total volume of solution used in each experiment is


kept constant

......................... .. ......... (2)

Explain, in terms of colliding particles. why the rate of reaction


decreased.

.................. (2)

Use the information in the table to show how the rate of reaction
changes when the volume of P decreased.

1
2
3
4
5
6

In an experiment, a beaker containing the reaction mixture is placed over a


piece of paper marked with an x. The sulfur as it forms slowly hides the x
from view. The time taken for the "X" to disappear from view is tabulated below:

(a)

(b)

(c)

CH5072/02/A

........................ .........................................................(1)
A student set up a cell as shown, to make electrical energy.

SACSS Preliminary Exam 11 2011

For
Examiner's
Use

zinc rod

::~:~=~=~:~=~=~=~=~=~=~:
:~~~~~~~~~~~~~~~~~~~~~~:

----~------------~----

copper rod

aqueous copper (II) nitrate

Label on the diagram the cathode and the flow of electrons in the
external wire.

::::::::::::::::::::::::::::::::::::::::::::::::::1
(a)

How would you expect the read ing on the voltmeter to change if the
zinc rod is replaced by an iron rod? Explain your answer.

Mendeleev devised the modem Periodic Table.

CH5072102/A

[2]

[Turn over

. ............... ... ..................... . ........................... .. ......... .............. ....(2)

(b)

(a)

SACSS Preliminary Exam 112011

For

Use

Examiners

(b)

10

lithium

lithium oxide

oxides fonned
Li,O

formula

sodium oxide
sodium peroxide

N~2Q._
Na202
K02
Rb02

element

He put elements in the same Group if their properties were similar.


One of the properties he used was the formula of the oxides of the elements.
When Group I metals tarnish in air. different types of oxide are formed.

sodium
potassium superoxide
rubidium superoxide

potassium
rubidium
Mendeleev put all of these elements in Group I.
(I) Do the fonnulae of the oxides given in the table show clearly that all of
the elements belong m the same Group? Explain your answer

[1]

[1]

............[1]

(ii) Do the formulae of the oxides given in the table show a trend down the
Group? Explain your answer.

{i li)What are the formulae of the peroxide ions?


peroxide ion .................. .. ... ............. ........... .
Sea water contains potassium bromide.

Write the balanced chemical equation for the reaction in (i)

reason..................... ___ ... ... ... ... ... ... ...... ... ... ... ... ... ... ... .. .... . .. [2)

name: ................................. ... ........... ......... ...... ................. .

Give a reason for your choice.

(i ) Bromine can be produced from sea water"by displacement.


Name an element that can displace bromine.

(Ii)

CH5072/02/A

............... ............ . .. ..........................................................(1)

SACSS Preliminary Exam II 201 1

11

8.0

car idling

4.0

car
acceleratin

4.0

car
cruisin

. .. ............................................................... ... ... . .......................(2)

Explain how the oxides of nitrogen are produced in the car and give
one harmful effect of this pollutant.

-----121

Explain why the percentage of carbon monoxide is highest when the


car is idling.

% volume of
carbon monoxide

Carbon monoxide and oxides of nitrogen are air pollutants that could be found in
the exhaust of a car. The following table shows the percentage by volume of
carbon monoxide in exhaust gases from a car.

(a)

(b)

(c)

A catalytic converter is fitted in the car to remove harmful pollutants


before the exhaust gases are discharged. Name the two less
polluting gases when the oxides of nitrogen and carbon monoxide react
together.

CH5072/02/A

(Turn over

.. ............................................................................ ... .. ...........[1]

SACSS Preliminary Exam II 2011

For

Use

Examiner's

(a)

12
Section B
Answer all three questions from this section.

A pupil carried out two separate experiments using different electrodes 1n the
laboratory.

Write your answers on any lined pages provided and/or separate answer paper.

The total mark for this section is 30.

The last question is in the form either/or and only one of the alternatives should be attempted.

81 O

In each experiment, he electrolysed 2.00 dm of aqueous silver nitrate


containing 2.68 g of silver ions. The same amount of current was passed in
both experiments and the increase in mass of the cathode was weighed
every 5 minutes for 50 minutes.

"'"'""~'

Experiment 1

The diagram below shows the results of the two experiments

m3SSof
silver
deposrtedlg

,., ~

[2]

Tome/min

Describe how the mass of silver deposited at the cathode changes


with time in each experiment.

(1)

50

{I)

Write the half equation for the formation of silver at the cathode for
both experiments

(2)

40

{ii)

One possible reason for the different results obtained in the above
experiments is the use of different materials as the anode. Suggest
suitable substances that can be used to make the anodes of
Experiment 1 and 2?

(iii)

Using your answer in {iii), explain the shape of the graph in


Experiment 2.

[2]
(iv)

State the observation at the anode for Experiment 2 and give the half (2)
equation for the reaction.

CH5072/02/B

(v)

SACSS Preliminary Examination II 2011

811

{b)

13

A circuit was connected as shown in the diagram below and a current


passed through it for a period of time.

AH,

2C02 + 2H20

(Total: 12)

Given that 12.8 g of copper and 14.0 g of cerium were deposited at


electrodes 8 and D respectively, calculate the charge on a cerium ion.

Ethene, C2H burns in oxygen.

~i

C2H + 302 ?

bond energy, in kJ/mol

Use the following data to answer the following questions that follow.
bond

347
6 12
803
412
496
464

(3)

Calculate the heat of reaction, 6H for the combustion of ethene.

C-C
C=C
C=O
C-H
O=O
0-H
{a)

Using the ideas of bond breaking and bond forming, explain why the reaction
is exothermic or endothermic.
(2)

[2]

(b)

Draw the energy profile diagram for this reaction.


Label on the diagram the activation energy and the enthalpy change.

(Total: 8)

(2]

[2]

(c)

[Turn over

Using ideas about structure and bonding, explain why ethene has a low
boiling point.

CH5072/02/B

(d)

SACSS Preliminary Examination II 2011

formula of
alcohol

C HOH
2 5

C HOH
3 7

c.HpH
CH OH

14

number of
carbon
atoms
2
3

2 5

CH QC H
3

CH 0CH
3
3
CHOC H

formula of
ether

methoxypropane

methoxyethane

methoxymethane

!--

name of ether

The table shows some information about 2 homologous series: alcohol and ether.

Either
8 12
name of
alcohol
ethanol
propanol
butanol

[2]

pentanol

Deduce the name and formula of the ether that contains 5 carbon atoms

(1]

11

(a)
Deduce the general formula for the family of ethers.

(b)

(d)

Propanol reacts with ethanoyl chloride (CH3COCI) in the following reaction .

Ethanol has a higher boiling point than methoxymethane. If all the


compounds in the above table undergo fractional distillation, which
compound would be collected last?

[1]

(c) A student made a statement during his test that alcohols and ethers are [2]
isomers. Using the information given in the table, explain whether you agree
or disagree with his statement.

(e)

HCI

[1 ]

CH3COCI + C3H10H -7

What is the name of compound X?

[1)

CH3COOCsH1 +
compound X

(I)

Ethanoic acid also reacts with propanol.


Write an equation for the reaction of ethanoic acid with propanol.

(Total: 10)

(2)

(Ii)

(i ii )

CH5072/02/B

Give one similarity and one difference between the reaction of


ethanoyl chloride w ith propanol and the reaction of ethanoic acid with
propanol.

SACSS Pre liminary Examination II 2011

Or
812

Study the reaction scheme below.

(C~HJ

Substance P

concentrated

OXlda!Kxl

col~S llQUKl

Bromine decOlourtses and


compound Q is fonned.

15

r 1

[3]

colour1ess llquid

Name P, Q and R.

(1)

aod

(i)

Draw the structural formula of T.

l~~~1
(a)

(i i)

(4)

Propenoic acid has the molecular formula C H C0 H.

(ii i)

(1) Describe a test that can be used to show the difference in chemical
property between S and propenoic acid.

(2) Describe a test that can be used to show the similarity in chemical
property between S and propenoic acid.

\ '

Iv\

_..r.~,...... c

fl

I
. . . ,...o....

l-J

(b) Methionine is an amino acid which can undergo condensation polymerization to


form protein, a polyamide.
H
ll lI 11 'N-H
H

H.._ I
_..c ,
fl

methionme

H H

Draw the structural formula of the protein when methionine molecules (1)
undergo condensation polymerization.

(Total: 10)

(1)

(i)

[Turn over

(ii)

CH5072/02/B

This polymer is non-biodegradable.


Explain the term non-biodegradable.

SACSS Preliminary Examination 112011

21

11

22

12

33

23

13

34

24

14

35

25

15

36

26

16

37

27

17

38

28

18

39

29

19

40

30

20

10

St. Anthony's Canossian Secondary School


2011 Prelim II Exam CHEMISTRY 5072
ANSWERS

c
32
A

Paper 1

31

Cl-35 has 18 neutrons while Cl-37 has 20 neutrons

~@J [@f[@J

X = X electron

=chlorine

marks]
Nickel l lll chloride
Silver carbonate
Ammonium sulfate
Aluminium oxide
Potassium nitrate
A, of chlorine = 35 x75/100 + 37 x 25/100 = 26.25 + 9.25 = 35.5

c
rdDC'I'

Section A f50
1
a
b
c
d
e
a
2

b
c

Correct ions shown 1m


Kev 1m
The CFCs in the higher up the atmosphere are decomposed by sunlight
to produce chlorine atoms. These chlorine atoms destroy the ozon e
(molecul es) layer which p rotect the earth from the harmful UV
radiation. UV radiation can cause skin cancer and cataracts in
human.

a
c
a

I I
c

P: CuC0 3
Q: CuO
T: CuCli
S: CaCOa
[Accept both naminq and formula]
CuCOi (s) + 2W (aq) -7 Cu2 (aq) + C02 (g) + H,O (I)
[1 mark for state symbols
1m for equation!
2COS(g) + 30 (g) ___. 2C0 (g) + 2S0 (g)

:::::::::::::::::::::::

L__j

copper (II) nitrate

(i) Cathode

__J_____L_Copper rod

elect rons

~i
j_ '-. ____./--:L ) Flowof

are fewer particles per unit volume/particles further apart and t he


frequency of collision/ number of effective collisions per unit time
decreases leading to a lesser number of effective collisions. Hence rate
of reaction decreases.
The total volume is kept constant to ensure a fair comparison for all unit
volumes for experiments 1 to 6./ fair test .

As the volume of P add:-'e'-:


dLd:---ecre_a_s_e_s_
, -:-:
th_e_c_o_n_c_e_n-,-tr-a"'ti_o_n_of""P""d
__,,e_cr
_ e_a-s e
_s__--=r=::h-e-re

Sulfur dioxide is oxidised as the


Oxidation state of sulfur increases from +4 in sulfur dixiode to +6 in
sulfate ion.
From the information, as -the % of iron increases from 6% to 25%, the HV
value increases from 179HV to 250HV.
Hence, the higher the percentage of iron in the alloy, the higher is the harder is
I the alloy
Zinc is more reactive than iron and loses electron more readily than iron.
Zinc corrodes in place of iron when exoosed to oxygen and water.
Pure iron is soft as the atoms are arranged in an orderly manner and the
atoms can slide past each other when a force is applied.
Steel is an alloy of iron and carbon. The carbon atom disrupts the orderly
arrangement of the atoms. Hence, it is harder as the atoms cannot slide past
each other when a force is applied.
As the volume of P decreases from 50 cm 3 to 15 cm3. t he time for
to
disappear increases from 8 to 89s. This indicates that the rate of reaction has
decreased.
[Quote values from table -1 m

f----+~+--~rrect Conclusion - 1m

Zinc rod

L--

----------

:-:-:-:-:-:-:-:-:-:.-:

==:::::::::::::::::==

1~~tttIIIf~@di-Aqueous

B10

i
Ii

The reading on the voltmeter will decrease_


The further the two metals are in the reaclivit~ series of metals, the greater the
voltage produced. Iron is lower than zinc in the reactivity series of metals, so
irgo is i;;IQser to coi;i~r than zinc is, so the voltage 11roduced i~ malle[.
The elements have different formulae and hence appear to have different
valencies in the oxides. Hence they do not seem to belong to the same Grouo.
Yes, going down Group I, the amount of oxyge'l in the formulae increases

o.

----

2e
02mot
+ ne
04mol

'

Cu
0.2mol
'Ce
0.1 mol

iii
name. chlorine
b I
reason- the element is bromine in Group Vll and is thus more reactive than
bromine
ii Cl, + 2KBr ' 2KCI + Br,
a
When the car is not moving, the petrol in the engine is burning 1n a hm1ted
supply of air in the engine compartment.
Thus incomplete combustion of petrol produces lots of carbon monoxide.
Under the high temperatures in the car engines, nitrogen and oxygen in the
b
air react to form oxides of nitrogen_
Oxides of nitrogen form acid rain which kills marine life/ corrodes building
made of stones and metal.
c
Nitroaen and carbon dioxide
In experiment 1, the mass of silver deposited inrcreases at a constant
a i
rate/proportionally.
In experiment 2, the mass of silver deposited increases at a constant rate
/proportionally but stops at 40 minutes/remains constant after 40 minutes.
II AQ. + e ' AQ
Ill In experiment 1, the anode used is silver.
In experiment 2, the anode used is an inert electrode e.g. platinum or carbon
or Qraohlte.
iv In Experiment 2, the anode is made from an inert material I platinum/ graphite
I carbon.
The silver ions discharged comes from the electrolyte.
When all the silver ions are completely removed I discharged I reduced from
the aqueous silver nitrate, the reaction stops. This explains why reaction stops
when 2.68 g of silver is deposited.
Bubbles are observed.
v
40H' ' Q , + 2H,O + 4e
Number of moles of copper deposited= 12.8/64
b
= 0.2mol
number of moles of cerium = 14/140 = 0.1 mol
cu +

Ce,,.

n=4

81 1

B12
Elth
er

b
c
d

Energy absorbed for bond breaking


= 612 + 4(412) + 3(496)
= 3748 kJ/mol

Energy given out for bond making

'

= 2(803x2) + 2(464x2)
= -5068 kJ/mol
Heat of reaction, t.H
= -1320 kJ/mol

= (-5068 + 3748) kJ/mol

2co, ... 2H2o

The reaction is exothermic


As the energy given out 1n forming the bonds in carbon dioxide and water 1s
greater than the energy absorbed for breaking the bonds in ethene and
oxygen
Enrrc

A<llvolion
Ener11

C,H, Jo,1

Enthalpy

Cllan~e

Progress of Reaction

Musi label
.t' axis
./ exothermic or endothermic must match ca)culation results in (a)
,/ activation energy (one way arrow)
.t' enthalpy change (one way arrow)
.' reactants and products.
IAnv miss1nQ information -1m up to a maximum of 2 marks]
Ethene has simple molecular structure.
little energy is needed to break the weak intennolecular forces of
attraction between the particles. Hence it has a low boiling point
J

CH OCH ,
methoxybutane

CH,OC.H~ n = 1 2 3 __ .
Agree.
They have the same molecular formula but different structural formula
pentanol

612
Or

e
iii

Ii
Ii

a 11
II
iii

b Ii

ii

+ C1 H,OH

Propyl ethanoate
CH~COOH

"
I

CH3COOC3H, + H,O

similarity: Both reactions produce propyl ethanoate


difference: Reaction of ethanoyl chloride with ethanol produces hydrogen
chlonde while reaction of ethanoic acid with ethanol produces water.
P Propene [1]
Q Dibromopropane (1]
R Propanol [1 I
I

'4

+t

f)

l
ff

----u-I - - -
(1) Difference in chemical property:
Add aqueous brormne separately into test tubes containing S and propeno1c
aod and then shake the mixtures.
Aqueous bromine added to S remained reddish brown
However, reddish brown aqueous bromine turns colourless when added to
propeno1c aCld
(2) Similar chemical property:
S and propeno1c acid can be added separately to test tubes containing
aaueous sodium carbonate.
Both S and propenoic acid show the same results.
Effervescence is produced.
When the gas is passed into limewater, a white precipitate is formed indicating
the presence of carbon dioxide.

OR

C -C.+
f

:11

H-C-H

H - C - H

H - C - ~

..

t
H

I
H-C-'i

H-C -

"-C->1

N - C . - C -N
l
I

Add a piece of magnesium ribbon separately into test tubes containing


solutions of S and propenoic acid.
Both S and propenoic acid show the same results.
Effervescence is produced.
When the gas is tested with a lighted/ burning splint, the splint extinguishes
with a 'pop' sound, indicatinq the presence of l}y~rooen_gas.

...nentn'.'l-1n~

I The polymer cannot be broken down by bacteria naturally.

Mdm Mo k L H

- , Class

IIndex Number-

UNITY SECONDARY SCHOOL

1 hour

5072 / 01
16 September 2011

SECONDARY FOUR EXPRESS

PRELIMINARY EXAMINATION 2011

Page 1 of 10

Prelim Exam 2011

This paper consists of 10 printed pages, including this cover page

Sec 4 Express Chemistry

Setter

5 A copy of the Periodic Table is provided.

4 Each question carries 1 mark..

3 Answer all questions on the OAS.

2 This paper consists of 40 multiple choice questions.

Write your name, class and shade the register number in the spaces on the
Optical Answer Sheet (OAS).

READ THESE INSTRUCTIONS FIRST

Additional Materials: Optical Answer Sheet (OAS}

PAPER 1

CHEMISTRY

j Name

-]

1.

2.

3.

4.

5.

Bolling point
Smell
Colour
Solubility in water

Which property of an ester can be used to check its purity before use as a food
flavouring?

A
D

Calcium carbonate and sodium chloride


Copper Oil sulfate and sodium chloride
Ethanoic acid and ethanoi
Zinc carbonate and silver chloride

From which mixture can the underlined substance be obtained by adding water,
stirring and filtering?

Ammonia
Carbon dioxide
Butane
Sulfur dioxide

10
8
16

Ar
18
18

40
40
-

0 2 and Ar?

50 cm of methane gas (CH 4) diffused through a porous pot in 100 seconds. 50 cm


of gas Y took 400 seconds to diffuse under the same conditions of temperature and
pressure.
What could gas Y be?

B..

23

17

o-

24

Na
10
11

Which row correctly shows the total number of electrons for Na,

B.

c.
>-0

C --

Page 2 of 10

Prelim Exam2011

What is the total number of shared electrons in the ethyne molecule shown below?

H--C

10
12

[ ~Sec 4 Express Chemistry

6.

9.

8.

7.

Silicon carbide has the following structure.

key

10.

2.4 dm3
1.2 dm3
0.9dm3
0.6 dm3

A mixture of 0.6 dm3 of oxygen gas and 2.4 dm3 of hydrogen gas is exploded. W hat
is the volume of the remaining gases at room temperature and pressure?

A
D

Which of the following statements concerning acids is/are correct?


All
All
All
All

acids are liquids.


acids consist of oxygen.
acids react with metals to give hydrogen gas.
acids produce hydrogen ions when ionised in water.

I
II
Ill
IV

IV only
II and IV only
I and II only
I, II, Ill and IV

c
D

T he pH of an aqueous solution of hydrochloric acid is 2. What will be the pH of the


acid after the addition of 1O g of sodium chloride?

1
2
9

&

They react with acids.


They contain OH. ions.
They dissolve in water to give solutions of pH greater than 7.
They react with ammonium salts to give ammonia gas.

Which of the following is the most characteristic property of bases?


A
B

Page 4 of 10

Barium carbonate and sulfuric acid


Barium chloride and sodium sulfate
Barium oxide and potassium sulfate
Barium nitrate and calcium sulfate

Prelim Exam 2011

Which two substances would be suitable for preparing barium sulfate using the
precipitation method since barium sulfate is an insoluble salt?
A
B
C
D

Sec 4 Express Chemistry

14.

13.

12.

c
11.

= a silicon atom

0 = a carbon atom

It is a very hard solid and the atoms are bonded in a covalent network.
It is a soft slippery solid with covalent bonding.
It is a very hard solid with metallic bonding.
It is a hard solid with a lattice structure.

Which statement is true for the compound?

B
D

Substance X has the following properties:


melting point higher than soo0c
insoluble in waler
insoluble in organic solvent
conducts electricity when molten

Aluminium oxide
Silicon dioxide
Polyethene
Potassium chloride

What could substance X be?

a
D

Carbon monoxide
Ethanol
Sodium sulfate
Calcium oxide

Which of the following contains both ionic and covalent bonds?

A
B

Page 3 of 10

Sulfuric acid , H2S04


Hydrochloric acid, HCI
A luminium sulfate solution, A'2(S04) 3
Calcium chloride solution, CaCl2

Prelim Exam 2011

Which of the following aqueous solutions contains the greatest number of ions?
A
B
D

Sec 4 Express Chemistry

15.

16.

17.

18.

19.

Add excess sodium carbonate powder to hot nitric acid


Add excess sodium metal to cold nitric acid
Titrate sodium hydroxide solution with nitric acid
Titrate sodium chloride solution with nitric acid

Which is the best way to prepare a sample of sodium nitrate crystals?


A

B
0
An aqueous solution of compound X reacts with aqueous sodium hydroxide to form a
green precipitate and then aluminium powder is added. The mixture is heated and a
gas that turns damp red litmus paper blue is given off.

ammonium nitrate
copper (II) chloride
iron (II) nitrate
iron (Ill) chloride

What is X?
A
B
C
D

ammonia and chlorine


ammonia and hydrogen chloride
carbon dioxide and chlorine
carbon dioxide and sulfur dioxide

Which two gases each change the colour of damp red litmus paper?

A
B
D

I Gas formed- at cathode

Concentration of sulfuric
acid
Increases
Decreases
Increases
No change

Which of the following sets of changes occurs during the electrolysis of dilute sulfuric
acid using platinum electrodes?
Gas formed at anode

Anode (+)
Steel sooon
Steel sooon
Silver rod
Silver rod

Preiiin Exam 2011

Electrolytic solution
Silver nitrate
Silver chloride
Silver nitrate
Silver chloride

Page 5 of 10

Cathode(-)
Silver rod
Silver rod
Steel sooon
Steel spoon

A steel spoon is to be plated with a thin layer of silver. Which of the following
combination gives the most suitable setup for the electroplating cell?

A
B
D

Sec 4 Express Chemistry

20.

21.

22.

23.

the formation of covalent bonds between atoms


the formation of negative ions from atoms
the positive and negative ions coming together
the transfer of electrons from a more reactive to a less reactive element

Which of this gives rise to the electrical energy produced by a simple cell?

t.H = +52 kJ/mol

More bonds are broken than are formed


More bonds are formed than are broken.
Energy involved in the bond breaking is greater than that of the bond
forming.
Energy involved in the bond forming is greater than that of the bond
breaking.

H2 +12 -7 2HI

Why is the following reaction an endothermic process?

A
B

t.H = -570 kJ

The balanced chemical equation for the formation of water from its element is:

2H2 (g) + 02 (g) --> 2H20 (I)

What can you deduce about the value of 570 kJ of energy?

II is gained from the surrounding when 1 mole of water is formed.


It is gained from the surrounding when two moles of water are formed.
It 1s lost to the surrounding when one mole of water is formed.

It 1s lost to the surrounding when two moles of water is formed.

Prelim Exam 2011

Cl2 -7 2CI
HF ~ H + F '
N2 + 3H2 -7 2N + SH
H2 + 02 -7 2H + 20

I
II
Ill
IV

The equations for four reactions are given below:


Reaction
Reaction
Reaction
Reaction

All the reactions


Reactions Ill and IV
Reactions I, II and IV
Reaction Ill only

Page 6 of 10

Which of the above reactions is/are endothermic?


A

B
G

[ -Sec-4 Express Chemistry

24.

Volume of oxvaen aas used


Same in each case
Smaller with zinc
Smaller with zinc
Smaller with copper .

Rate of reaction
Faster with zinc '
Faster with zinc
Faster with copoer
Faster with zinc

10 g each of zinc powder and copper powder were separately burnt in excess air at
the same temperature and pressure. Both metals react with oxygen at a high
temperatu re. Which of the following is correct?

A
B

29.

Y
Z
X
X

is the most likely to form a negative ion among the three elements.
is the most likely to form a positive ion among the three elements.
forms an ampohteric oxide.
and Y are definitely gases.

X , Y and Z are consecutive members of the Periodic table with X having the smallest
atomic number. Y does not form any compounds with any element. From the data
given, it can be predicted that

A
C
D

Catalysts are used in some industrial processes:

Catalvsts
Fe+ Mo

30.

Process
Haber Process

V20s

Dilute sulfuric acid reacts with copper (II) oxide to form copper (II) sulfate and water.
What would not alter the rate of this reaction?

Contact Process

A l(C2Hs)3 + TiCl4

Polymerisation of ethene

The concentration of the sulfuric acid.


The pressure at which the reaction takes place.
The size of particles of copper (II) oxide.
The temperature of the reacting mixture.

Prelim Exam 2011

5
3

A will not displace B from its oxide


C is displaced from a solution of its salt by B.
C will displace A from a solution of its salt.

A, B and C are three metallic elements.

A,B,C
A , C, B
B, A, C
B, C, A

Page 8 of 10

Arrange these three metals in order of increasing reactivity.


A
B

Sec 4 Express Chemistry

Prelim Exam 2011

How many transition elements are shown in the table (as the element or in a
compound)?

C
D

When acidified potassium


dichromate(VI) is reduced
Green to oranae
Oranae to areen
Green to oranoe
Orange to green

31.

Cu Cl

C-7 CO -7 C02
Pb02 -7 PbO -7 Pb
N2 -7 NH3 -7 NO
C2H2 -7 C2H4-7 C2He

Page 7 of 10

Silicone manufacture

Which series of changes include both oxidation and reduction?

B
D

When potassium iodide


is oxidised
Brown to colourless
Brown to colourless
Colourless to brown
Colourless to brown

What are the colour changes observed in the following reactions?

A
0

All groups contain metals and non-metals.


In Group VII, the melting point of the elements increases with proton number.
In Group I, the reactiv ity decreases with proton number.
Atoms of elements in the same group have the same total number of
electrons.

W hich statement about groups in the Periodic Table is correct?

Sec 4 Express Chemistry

28.

27.

26.

25.

32.

33.

34.

35.

36.

Aluminum, Magnesium, Potassium


Copper, Iron, Magnesium
Sodium, Potassium, Zinc
Zinc, Iron, Lead

Which set of three metals are all extracted by reduction with carbon?

A
B
D

None
One
Two
Three

Four beakers of blue copper (II) sulfate solution were prepared. Excess zinc, silver,
iron and magnesium powder were added separately into the beakers of copper (II)
sulfate solution. How many solutions will still be blue after a few days?
A

a
c

Oxygen
Carbon dioxide
Argon
Hydrogen

A sample of arr was passed into aqueous potassium hydroxide and heated copper.
Which gas is still found in the remaining sample?
A

nitrogen
nitrogen dioxide
carbon monoxide
lead (II) oxide

Which one of the following substances, which is present in the car exhaust fumes,
could contribute to the formation of acid rain?

Page 9

ono

ethene, ethanol and butane


ethane, propene and pentane
methane, ethene and butene
ethane, propanol and methanoic acid

Prelim Exam 2011

One of the fractions obtained by the fractional distillation of petroleum oil contains
molecules with 1 to 4 carbon atoms in them. Which of the following molecules can be
found in this fraction?

C
D

Sec 4 Express Chemistry

37_

38.

39.

40.

Determining densities
Checking electrical conductivity
Using blue litmus paper
Mixing with water

Which of the following ways cannot be used to distinguish between dilute ethanoic
acid and ethanol?

The diagram shows the structura l formula of a compound Z.

H H H H H H H 0
I
I
I
I
I
I
I
II
H-c - c - c - c - c = c-c-c-o-H
I

It can undergo esterifcatron.


It can undergo addition polymerisation.
It does not react with magnesium.
It decolourises bromine water.

Which of the following statements about compound Z is NOT correct?

propanol
butanol
propanoic acid
butanoic acid

Process
Addition polymerisation
Neutralisation
Condensation polvmerisation
Condensation polymerisation

END OF PAPER
Page 10 of 10

Prelim Exam 201 f

T
Polyethene
Ethanoic acid
Fats
SuQar

An organic compound Y reacts with calcium carbonate to give a compound with the
formula (C3H1C02) 2Ca. What is Y?

s
Ethene
Ethanol
Amino acids
Starch

Which process is used to convert substance S to T.?

A .

-B

c
D

Sec 4 Express Chemistry

-]

[ Name

CHEMISTRY
PAPER 2

I Class

I Index Number

UNITY SECONDARY SCHOOL


PRELIMINARY EXAMINATION 2011

1 hour 45 minutes

5072 / 02
12 September 2011

SECONDARY FOUR EXPRESS

Additional Materials: 3 Writing Papers

READ THESE INSTRU CTIONS FIRST


Write in dark blue or black ink and on both sides of the writing paper.
Do not use staples, paper clips, highlighters, glue or correction fluid.
Write your name, class and register number in the spaces provided on the writing
paper.
Section A
Answer all the questions.
Write your answers in the spaces provided on the question paper.

Preliminary Exam 2011

[Turn over]

Section B
Answer three questions, the last question is in the forn of either/or.
The answer to each question is to be done on a fresh page of writing paper.
Al the end of the paper, fasten the writing papers to the back of the booklet.

Setter Mdm Mok LH

Page 1 of 14

This paper consi sts of 14 printed pages, including this cover page

I Sec 4 Express Chemistry I

Protons

Neutrons

Electrons

[4]

What are responsible for the conduction of electricity in sodium?

Molecules

a)

What are responsible for the conduction of electricity in aqueous


sodium chloride solution?

Atoms

b)

13
27

What are present in ethyl ethananote dissolved in trichloromethane?

R
12
24

c)

10
20

What are present in different numbers in atoms of carbon 12 and


carbon 13?

19

d)

Ions

Choose from the following words to answer the questions below:

Section A : Structured Questions [50 Marks]


Answer all the questions in the spaces provided.

2
Element
I
1 Proton Number
Nucleon
Number
I

[1]

Preliminary Exam 2011

Which of the above elements is a halogen?

a)

Which pair of elements would form an ionic compound with relative


molecular mass 62?
[1]

Page 2 of 14

b)

I Sec 4 ExpressChemislry I

c)

d)

e)

Draw a dot-and-cross diagram for the ionic compound in


part b). Show all the electrons.

Which element in the table is not likely to form a compound?


Why?

Page 3 of 14

[2)

[1)

[2)

Preliminary Exam 201 1

Which element in the table is suitable to make drink can? What


property makes it suitable for this use?

I Sec 4 Express Chemistry !

No

~,

/
t l'f'"'/'
11 r

,,

c"''

/e"- /"'- ),(

r "I

t'I

I M'/"'

>4

/r"'H'

~'

Experiment
A LSD tablet was added to
aqueous sodium carbonate and
the mixture was shaken

A piece of brown copper metal


was placed in a solution of
LSD.

Page 4 of 14

[2]

Preliminary Exam 201 1 ]

Deduce from the results shown above one chemical property of


LSD.
[1]

Observations
Effervescence.
Colorless, odourless gas
evolved formed white
precipitate in limewater.
No visible reaction

The table below lists the observations recorded for 2 experiments


carried out on L.S.D.

Empirical formula: - - - - - - -

Molecular formula: - - - - - - -

What are the molecular formula and empirical formula of L.S.D?

lj

~---:?"' /I"' /

ti"-/"'- ;i,

Shown in the diagram is the structural formula of a very powerful drug


called L.S.D (lysergic acid diethylamide).

a}

b}

I Sec 4 Express Chemistry I

c)

d)

a)

b)

Circle the portion of the L.S.D. molecule which accounts for the
property exhibited in part (b).
(1)

What would be observed if aqueous bromine is added to L.S.D.?


(2)
Explain.

Preliminary Exam 201_1_ I

[1 J

(1)

(1)

(l]

(1)

Ozone is an oxidising agent. State the colour change you expect to


see when ozone is bubbled into aqueous potassium iodide.
Colour of potassium iodide before bubbling:
Colour of potassium iodide after bubbling:

What is the colour of this salt?

Name the salt formed in the reaction.

Explain, in terms of electrons, why chlorine has been


reduced.

2 Fe2+ (aq) + Cl2 (g)-> 2 Fe 3 (aq) + 2cr (aq)

The following reaction occurs when chlorire 1s bubbled into


aqueous iron (II) chloride.

(i)

(ii)

(iii)

I Sec 4 Express Chemistry I - - page5of 14

Write down the formulae of the ions which take part in each of the
following.
Part (a) has been completed as an example.

a)

The corduction of an electnc current in molten (liquid) sodium


bromide.
Formulae of ions: Na+ and Br-

[2]

The formation of a white precipitate when aqueous sodium


hydroxide is added to zinc nitrate solution.

and-------

and-------

Formulae of ion: - - - - - - - - - - - - - -

The formation of a red colour when blue litmus is added to an


(1)
aqueous solution.

Formulae of ions:

The formation of blue precipitate when aqueous ammonia is added


to aqueous copper (II) nitrate.
[2]

Formulae of ions:

b)

c)

d)

a)

6PbC03 + 02 ~ 2Pb304 + 6C02

Page 6 of 14

Preliminary Exam 201 1

What is the purpose of the above undercoat paint?

[1 J

Red lead or lead tetraoxide (Pb 304) can be used as a pigment


for undercoat paints for iron objects. It can be prepared by
reacting lead (II) carbonate in air.
Below is the chemical equation for this reaction.

(i)

[ Sec 4 Express Chemistry

b)

(ii)

Calculate the volume of air required to react with 40 g of


lead (II) carbonate. (Take percentage composition of
oxygen in air to be 20%)
(2)

Page 7 of 14

Preliminary Exam 2011

The mineral calcite is a carbonate of calcium with the formula,


CaCOa. When 2.0 g of an impure sample of calcite was completely
dissolved in an excess of hydrochloric acid, 0.85 g of carbon
dioxide was evolved. One mole of the carbonate produces 1 mole
of carbon dioxide.
Calculate the percentage purity of the calcite used.
(3)

I Sec 4 Express Chemistry I

zinc
barium
iron
atummmm

time

A student attempted to determine the reactivity of five metals by adding


10g of each of the metals to excess hydrochloric acid and measuring the
volume of hydrogen gas collected. The results were plotted into a graph
as shown below.

ofH2gas

Volume

fom1ed
/cm3

copper

Explain why the graph for the reaction between copper and

(1)

[1 ]

a)

Based on the results in the graph alone, give the order of reactivity

Aluminium is a reactive metal and it tarnishes quickly when exposed to

of the five metals (starting with the least reactive).

hydrochloric acid is a straight line on the 'time' axis.

b)

c)

Page 8 of 14

Preliminary Exam 201 1

not as reactive as expected. Suggest a reason for the above result. (1]

air. But the results obtained by the student shows that aluminium is

I Sec 4 Express Chemistry I

d)

(2)

Using brass as an example, explain why the alloy is hard while its

Brass is an alloy of iron and zinc.

constituent elements are soft.

test tube B

The following apparatus can be used to produce ethanol from sugar by


fermentation.

flask A

One of the raw materials for the process is sucrose, C, 2H220 11. The first
stage of the process is the reaction of sucrose with water to form glucose
(CsH120s).

(1 I

Write a balanced chemical equation for the conversion of sucrose


to glucose.
(1)

Preliminary Exam 2011

a)

Name the process by which sucrose is converted to glucose.

Page 9 of 14

b)

I Sec 4 Express Chemistry I

c)

Explain why boiling would not be an effective method of increasing


the rate at which ethanol is formed.
[1)

Name the other substance added to flask A.

11 I

d)

--

Preliminary Exam 2Q1D

e)

,,_,,,

Page 10of14

Describe what would happen 1f the contents of nask A are left open
in the air. How would you test to see If a change has taken place in
the contents?
[JJ

_________

[ Sec 4 Express Chemistry

i'.H

=-184 kJ

The following reaction takes place in the Haber process used to


manufacture ammonia gas.
Ni + 3H2 ~ 2NH3

Bond energy
(kJ/mol)
436
163

6H1

Bond
N=N
N:=N

2N + 6H

AH1

AH,

Bond energy
(kJ/mol)
409
945

(2)

[2]

(1 )

Preliminary Exam 2011

progress of reaction

2NH1

The table below shows some bond energies, measured in kJ/mol


Bond
H-H
N-N

energy

3H,

2N + 3H1

Ni+

6H.

Page 11 of 14

The bond energy of the N -H bond in kJ/mol

The value of i'.H3.

The activation energy required for the reaction.

Use the information above to calculate


a)

b)

c)

I Sec 4 Express Chemistry I

d)

c)

b)

a)

Salts obtained
(i)

Barium sulfate
Copper(ll) chloride

Preliminary Exam 2011

(4)

[2]

[1]
[1]
{1]

[1]

[1]

[1 ]
[1 ]

[1)

What three steps are required to obtain pure, dry sample of barium
sulfate in its preparation?
(3)
Explain why titration method is suitable for the preparation of
substance (i)
[2)
Explain the importance of adding excess substance (iv) in the
preparation of copper (II) chloride.
(1]

Reactants
Potassium hydroxide
and nitric acid
(ii) and {iii)
Hydrochloric acid and
{iv)

Copy and complete the following table on the preparation of salts,


by naming the substances (i), (ii) and (iii) and (iv).

Section B (30 marks)


Answer all th ree questions
The last question is in the form of either/or and only .Q.!!!lalternative should
be attempted.
10

11

Page 12 of 14

Construct a balanced equation to show the reaction of phosphorus


trichloride and water.
i)
W rite the equation for the reaction between phosphorus and
chlorine.
ii)
Why is it necessary to use dry chlorine in this reaction?
iii)
Suggest a reason why this reaction must be carried out in a
fume cupboard.
iv)
Calculate the mass of phosphorus trichloride which can be
obtained from 0.500 moles of phosphorus atoms.
Describe what you would expect to observe when phosphorus
trichloride was added to:
i)
litmus solution
ii)
aqueous silver nitrate
iii)
aqueous barium nitrate
Draw a diagram to show the arrangement of outershell electrons
in a molecule of phosphorus trichloride.

Phosphorus trichloride, PCl3, is a liquid having a boiling point of 76C. It


can be prepared by passing pure dry chlorine over heated phosphorus.
Phosphorus trichloride reacts vigorously with water producing phosphoric
acid, HJP03 and hydrogen chloride.
a)

b)

c)

d)

I Sec 4 Express Chemistry I

12
Either

0- H

\':::::'/

H- N- l o '- N- H
I

H
monomer B

There are tw o basic types of reactions for forming polymer: addition


polymerisation and condensation polymerisation.

monomer A

H-0
' c- @-c"
,,

0 -'/

The substances shown below are the monomers required to make the
polymer Kevlar that is used to make lightweight helmets.

a)

i)
Explain 'polymerisation' .
Give two differences between addition polymerisation and
condensation polymerisation.
[1)

[2]

Name the functional group present in the monomer A shown above. (1)

ii)

b)
Draw the structure of Kevlar (showing at least three repeating
units).

[1 )

c)

Name the linkage found in Kevlar and give an example of another


polymer that contains the same linkage.
[2)

Preliminary Exam 201 1

d)

e)

Page 13 of 14

Monomer A can be reacted with ethanol to form a new substance.


Draw the structural formula of this substance. State the conditions
for this reaction.
(3)

I Sec 4 Express Chemistry I

12

OR

Which one of the poles, A or B , is the positive pole?

Battery

An electrical circu it is set-up as shown below. A and B are the poles of the
battery. E lectrodes P and Q are both made of platinum. O 12 g of a pink
metal X was deposited on electrode P. At the end of the reaction, the blue
electrolyte turned colourless.

a)

Write an ionic equation for the reaction occurring at electrode P, using


X as the symbol for the metal.
[1)

b)

Suggest the identity of metal X.

c)

Solution of X(N03)i

d)

Name the gas collected in the tube above electrode Q and write an
Ionic equation for the reaction taking place at electrode Q .
Hence,
calculate the volume of gas collected, at room temperature and
pressure, in the tube.
(4)

[1)

[1]

(1)

e)

E lectrodes P and Q are now replaced by two electrodes R and S


respectively. Both electrodes R and S are made of X. Describe the
observations that can be made at the two electrodes, R and S.
[2]

Preliminary Exam 2011

Suggest an industrial use for the reaction in (e).

END OF PAPER

Page 14 of 14

f)

I Sec 4 Express Chemistry I

c
c

Sec 4 Exp Chem Prelim Exam


Pai er 1
1 A
9
2
A
10 A
D
11 A
3
4
A
12 B
13 A
5
14 B
A
6
15
A
7
16
8

2011
17
18
19
20
21
22
23
24

25
26
27
28
29
30
31
32

D
B
B
B

Mar king Scheme


A

c
c
D

A
A

33
34
35
36
37
38
39
40

c
c
D
A

--

Marks
1
1
1
1

1
1
1, 1
1

a. noble gas configuration


No mark for full electronic structure, should be full valence shell
s

1
1
1
1

1
1

C1sH1sN202
C1sH 15N202
Acidic or contains H+ ions in water
Reacts with a base to form salt and water or reactive metal to
produce H2
Circle on diagram COOH
Aqueous bromine which 1s reddish brown becomes colourless
Presence of C=C bond

1
and OH-

colourless
brown
Electron gain
Iron (Ill) chloride
yellow

Resistant to corrosion

p
Pand R
Correct dot and cross diaaram of R,. 2p

Paper2
Section A
No Answer
1a
electrons
b
ions
molecules
c
d
neutrons

2a
b
c
d

c
d

3a

...___

4a
bi
ii
iii

5a
-b - Zn"'

c
d

Cu,. and OH"

H'

2
1

1
1

1
1
1

=
=

Copper is unreactive, so no reaction

C12H22011 + H,O -7 2CsH120s


hydration
yeast
the hiah temo will denature the enzvmes in the veast
Turns sour or oxidised to ethanoic acid
Add litmus paper to it
Litmus oaoer turns red

1
1
1
1
1
1
1

2
2

945 +[436 x 3) =2253 kJ


-(2253 + 184) -2437 kJ
(1 mark for m inus sign and 1 mark for value)
2437/6 406 kJ

1
1

Cu, A l, Fe. Ba and Zn


The layer of aluminium oxide prevents the alumm1um from
reacting with the acid readily
Different sized atoms in alloys break the regular pattern of the
layers of same sized atoms, hence alloys are harder
Pure metals have equal sized atoms and are arranged in
re1:1ular pattern so the layers easily slide over one another

=
=

Mr of lead carbonate= 267


No of moles = 0.150
3
Vol of 02 = 0.150/6 x 24 =0.600 dm
3
Vol of air= 0.600 x 5 3.00 dm
Mr of CaCQ3 100
No of mol 2/100 0.02
No of mol ofC02 0.85/44 0.019318181
% puritv 0.019318181/0.02 x 100 96.6%

6ai
To prevent surface from contacting oxygen in air
I (no malKs for writing prevent rusting)
Mass of lead carbonate 40 g
ii

7a
b
c
d

Ba
b
c
d
e

9a
b
c

Section B

Either
Polymerisation is the process of many small molecules joining
topether to form a lara_e molecule
1 diff : addition polymerisation occurs with no loss of atoms or
molecules but in condensation polymerisation a small molecule
is,iroduced
2 diff : monomers in addition polymerisation have C=C bond
while monomers in condensation polymerisation have functional

PCh + 3H20 -? HJP03 + 3HCI


2P + 3Cl2 -? 2PC'3
The PC1 3 formed will react vigorously with water if the chlorine
contains water
The aases are toxic
Mass of PCb = 0.5 x 137.5 =68.75 g
Turns red
White orecipitate
No visible change or the barium nitrate solution remains
colourless (No marks for writina no chanae or no reaction)
Correct diagram of covalent molecule

10ai Potassium nitrate


ii
Barium nitrate or barium hydroxide
iii
Sulfuric acid or sodium sulfate
iv Copper (II) oxide or Coooer(ll) hydroxide
b
Filter the mixture and collect the residue of barium sulfate
Wash the residue with distmed water
Press dry the residue with filter paper
c The acid, alkali and salt formed are all soluble in water
At end of titration, acid is completely neutralised by alkali
(End ooint can ensure acid is completely neutralised bv alkali)
To completely neutralise the hydrochloric acid
d

11a
ii

bi
iii
iv
ci
ii
iii
d
12
ai
ii

1
1
1
1
1
1
1
1
1
1
1
1
1
1
1
1

1
1
2

1
1

A - carboxvl arouo

Correct structural formula showing 3 repeat units

Amide linkage
Nylon
Cone H, so.

rbl groups at each end of molecule


c
d
e

12
a
b
c
d

e
f

heatina
Correct formula of ester
OR
A

X + 2e-? X
Coooer
40H" -? Q, + 4H, Q + 4e
Oxvaen aas
No of mol of Cu= 0.12164 = 0.001875
No of mol of aas 0.5 x 0 .001875 = 0.000938
Vol of aas 0.000938 x 24 0.0225 dm0
At R, pink metal is deposited and R increases in mass
At S, electrode becomes smaller and dissolves to form X" ions
Electrolytic purification of copper

1
1

y,
y,

1
1
1
1

1
1
1
1
1

Second ary Four Express

PRELIMINARY EXAMINATION I
General Certificate of Education Ordinary Level

CHEMISTRY
PAPER 1

Penodoc: Tabte

OTAS ''"''"'"sheet

Add1110nal matenals

1 hour

Clas$

I
~

JULY 2011

5072/1

WESTWOOD SECONDARY SCHOOL

Cmdodate Name _______________ _ _

TIME
INSTRUCTIONS TO CANDI DATES

Candidate
Number

Write your name, class and candidate number in the spaces at the top of this page and in the
provided on the answer sheet.

o/

'

heel

) .

cmu;yr<noe

Sherice wanted to find out t he percentage of oxygen in the air using the apparatus
shown. Air is passed over a heated copper wire till no further decrease in volume is
observed.

. - ,, .,,;.

=i I""' .')''

9<1u.\'lingll ....cop;>er

.~ir

~ ..~ J,.t
,,..

The tube contaming the copper should be removed.

The measurements should all be taken at the same temperature.

Both syringes should contain the same volume of air at the start.

All the copper should have reacted

Which precaution should be taken by Sherice before measuring the initial volume of air and
before measuring the final volume of gases remaining in the apparatus?

Add water and distill

Add ethanol and distill

Add water and filter

Add ethanol and filter

Which one of the following methods can be used to obtain sodium carbonate from a solld
mixture of sodium carbonate and iodine?

hydrocnlo'~

....,.,t

Cc1t<.tiH~O

lurrrp:it c1f ..;hr:i1rctlf\I

To dry the gas.

To remove carbon dooxode from the gas.

To remove hydrogen chloride from the gas.

What is the function of Y?

C!lrt':Ol~

waler

To prevent water being su~ked back on to the hot carbon.

c
D

[Turn over

The diagram below shows an apparatus set-up used to obtain carbon monoxide gas.
cii!ute

s~aces

There are fo rty questions in lhis paper. Answer all questions. For each question, there are four
possible answers. A. B. C and D. Choose the one you consider correct and record your choice in
soft pencil on the separate answer sheet.

acid

[Turn over

Read very c arefully the i nstructions on t he answer sheet.

Each correct answer will score one mark. The total marks for this paper is 40.

INFORMATION FOR CANDIDATES


Any rough working should be done in the question paper.
The OTAS should be handed in separately at the end of the paper.
Electronic calculators are allowed.

A copy of Data Sheet is printed on page 16.

This ques t ion p aper consists of 16 pri nted pages.


Setter . Mn Th10-S1m Slaw Cheng

4 EXPICHEM/507211/PRELIM~/2011

er
Br-

3
In which one of the following sels do all three particles have the same total number of
electrons?

Na

Ne

H
Na

SO/Ve.'ltX

t
8
7
6

5
4

a
2

: I

-r-r

l'

I l i l 1 !
sotveni front -- ~r-i

9 10

II)

cs.,,,__

Q)

c::

++--+--:-++
-~ ' -f-t-!1-1
++-J ;::
W.--~

I '!rl . l
, it'
t-14---+-
r-t-
-- --,--,

I
,
l
I

, -A '

=t=t--+-t-r+--.:..-+-i-tI
'
'
'
I ' '

--.-_r-+--:r-r- +-+-

-1.-1.....l.-.--i--L'

I l ! i i I t l I
, -t-i-1-+-[-t---,
-r
, '
I ,
,

solventY-+

0.30
0.26

R, v alue In solvent Y

0.20

0.38
0.50

0.73

R, value i n solvent X

start _,,.#

o
~

A mixture of three amino acids is separated by two-way chromatograp1y. The positions of


three spots A, B and C, afte1 treating with a locating agent, are indicated in the
chromatogram below.

Amino acid

Glycine

0.91

0.70

Glutamic acid

Leucine

Arginine

Spot B and C only

Spot A and B only

Spot B only

SpotAonly

[Turn over

Using the information from the graph and the table above, which of the following spot(s)
cannot be identified?

4EXP/CHEM/5072/11PRELlM-ll2011

z.

II

"x

/'.y'-...

x"//

y~ /y~

4
A stable molecule containing atoms of the element X, Y and Z has the following structure.

z/

Ge

Si

Cl

~ -

W hich of the following is a possible combination of the elements?

Which substance does not have a macromolecular structure?

Graphite

Diamond

Silicon tetrachloride

B
C

Silicon dioxide

(II)

(I)

Any water in the conical flask will affect the reading of the end-point.

The last drop of solution is to be blown out from the pipette.

The conical flask is to be rinsed with the solution to be contained.

I, II and Ill only

[Turn over

Xin Hui prepared a sample of sodium chloride by titrating aqueous sodium hydroxide with
dilute hydrochloric acid. Which one of the following are misconceptions about titration
experiment?

(Ill)

11, Ill and IV only

I, II and IV only

(IV) The filler funnel is to be left on the burette during titration.

All of the above

C
D

4EXPICHEMl507211/PRELIM-l/2011

10

..

- ~ ~----M

s
thne

Carbon dioxide gas was cooled under pressure until it changes into a liquid. The graph below
shows how the temperatre of ca1bon dioxide changed with time.

lernpernturra

0
0

0
0

they contain carbon atoms joined to one another in different ways.

diamond is non-metallic in nature whereas graphite is metallic in nature.

diamond has covalent bonds whereas graphite has ionic bonds.

At stage QR, which one of the following diagrams best represents the structural arrangement
of carbon dioxide molecules?

o. O
0

o:QODo
o o coorrooo

O = carbon dioxide molecule


A

'r-.9-io0 ..o 8 oo
oo
v-oou-0

they contain different isotopes of carbon.


[Turn over

Diamond and graphite has different physical properties because._ _ _ __ _ _ _ __

4EXP/CHE M/5072/1/PREUM4/2011

11

12

13

14

15

The number of valence electrons decreases in the atoms.

The ability of the elements to conduct electricity increases.

The number of neutrons in the atoms decreases.

The ability to fonn x ions decreases.

6
Which one of the following statements correctly describes a trend in the properties of the
elements, going from left to right of Penod 2 of the Periodic Table?

Mg2

crl

Mn2'

All

Jade is a gemstone with a green colour. Which one of the following ions is responsible for its
colour?

Magnesium sulfate

Ethyl ethanoate

Copper(ll) bromide

Which compound has both ionic and covalent bonds?

Ozone

potassi1;1m chloride, potassium bromide. bromine and Iodine.

Excess bromine was shaken with a mixture of potassium chloride and potassium iodide
solutions. The final solution will contain - - - -- - - -- -- - -- - A

potassium bromide. bromine and iodine.

potassium chloride, potassium bromide and iodine.

potassium bromide, potassium iodide, bromine and chlorine.

B
D

Weak acid

Strong acid

Weak base

Strong base

[Turn over

A black powder is burned in air The gas produced dissolved in water to fonn solution R
The pH of solution R is close to 7. The gas is re<idily absorbed in aqueous sodium
hydroxide. What type of substance is present in solution R?

4EXP/CHEW5072111PRELIMl/2011

16

17

i
I

l-1

ri
I

11

Ii

1/
11 I !!
l I ; Ii
..... __...'r~-' --{~~17 z,-~2 Pb;,

-,
I

j____l__.-p.
Pb:

..

!:

r--,.
l

i
!I

Bas ic

Neutral

r...,

I iI

..

[Turn over

il
11

Fe' Cu' z,,: ~i

L,.___1....- - - . - -... -

Ii

f
I

I
. I.

Amphoteric

I .1 1 I.
_.__J
_
_.__
bl' Pb2 '
Fe,. Ci.;''

..-...--_..

7
Four separate solutions are prepared so that each contained 1 g of the ions Fe3 , cu2. Zn2
and Pb2' . To each solution, an excess of ammonia solution was added and the mass of any
resulting precipitate found. Which one of the following diagrams. A . B , C or 0 illustrates the
results?

....

l)l''.'.- z~l...

Ln __
f:cl'

Na,o ' ZnO ' Al20l co, ' co ' so,

The formulae of some oxides are shown below.

H,O '

Which entry in the table gives the correct number of each type of oxide?

Number of each type of oxide

Acidic
A

2
0

3
2

--- 2 2

t~

4EXPICHEM/507211/PREUM-ll2011

18

19

20

Mass of test tube

"29.04 g

= 21 .28 g
= 29.84 g

The following results were obtained from an experiment involving the reduction of an oxide
of iead to lead metal.
' "

M ass of test tube + lead

Mass of test tube + lead oxide

Pb203

PbO

Pb30,

Pb~

From the experimental results obtained, what is the empirical formula of this oxide of lead?

The liquid limonene can be extracted from oranges and has the following structure:

'c,,....

'-H

'H

,c~ _,....H

H
C/
'H

,.....c~ /H

H- C-H

H ......_

t-i_... I

H/I
H- C

w,....

H"',.-

H......._

1.5 mol

0.5 mol

2.0mol

1.0mol

How many moles of limonene will it take to react with one mole of hydrogen gas?

Calcium chloride

Magnesium oxide

Aluminium oxide

At the anode

2v2 - + Y2 +4e

[Turn over

Element X is extracted by the electrolysis of a molten compound of elements X and Y. The


electrode reactions are as shown.
At the cathode
x2+2~

Potassium sulfide

What 1s the identity of the molten compound of elements X and Y?

4EXP/CH EM/507211/PREl.lMl/2011

21

22

Concentrated
aqueous sodium
bromide

9
Chun Zong conducted an electrolysis experiment as shown in the diagram below.

c==;-

Graphite electrodes

23

Volume of sulfuric acid


3
required I cm

Mass of sodium
hydroxide I g

20
40

Volume of water I
cm3

1O cm3

:rn :!~

~-t:(H!lf

:m_-;;~

{.

:c

j }.

K.~}

Time/~

Cc~.:)il fl - - -

(~:rap.h

Diluting the glue with more solvent.

Using a catalyst.

Using Jess pressure on the glue during the setting process.

[Turn over

T(.t~:~~~3:!~1 :::- : ~~::ol~t'-'.>

Smoothening the surfaces of the pieces of wood before the glue is applied.

4EXP/CHEM/5072/11PREUMli2011

Which changes can be made to the experiment to obtain Graph B?

,;1:

A chemical reaction takes place when glue sets. An experiment was set up to measure the
effect of temperature on the time it takes for the glue to set when applied to two pieces of
wood. Graph A was obtaineod as shown below:

3s
5s

10
Lithium carbonate is reacted with dilute hydrochloric acid and the volume of carbon dioxide
evolved is recorded. The val ues are then plotted and the following graph is obtained.

Va!urn~

c:rrbon

{.i1uxide

'

nt r.tp_/
.;n:

(!

At which time is the rate of reaction the highest?

Experiment

20
40

40 cm3

B
0

pH increased

At positive electrode

40

2s
4s

Effect on electrolyte

24

At negative electrode

Colour1ess solution
turns orange
Silvery droplets

pH decreased

pH Increased

Effervescence of a
colourless gas

Colourless solution
turns orange
Effervescence of a ,
colourless gas

A
C

Effervescence of a
colourless gas
Silvery droplets

pH decreased

Effervescence of a
colourless gas

What is observed?

A
B

c
~

20
40

The table below shows the results of experiments to measure the volume of sulfuric acid that
is needed to neutralize the mass of sodium hydroxide dissolved in the volume of water.

5 cm3
20 cm'

[Turn over

From this table, what volume of sulfuric acid is needed to completely neutralize the sodium
hydroxide in Experiment 4?

4EXP/CHEM/507211/PRELIM ll2011

25

26

27

12

The raw materials are obtained from air.

Fe2'

A black solid and a colourless liquid.

P10<Wna

l'fC>llttU Of ttlc4at\

Pr-

30

\\

; L

\\

\iH~l2i in

I\

!y

\I I
i-0

The slag should float on top of the Iron.

Only iron ore and limestone should go In through the charger.

Hot air should enter at X, and waste gases should leave at points marked Y.

Graphite instead of coke should be put into the charger at the top of the furnace.

Sulfur dioxide

PhosphorusM oxide

Nttrogen dioxide

Carbon monoxide

4 EXPICHEM/S072111PREUM-ll2011

[Turn over

Waste gases from a coal-burning power station are passed through powdered calcium
carbonate to reduce air pollution. Which one of the following waste gases will not react with
the powdered calcium carbonate to be removed?

Which one of the following changes would correct the error?

'-~~--

Yi._;,~
' I
, ,,.,_,,,.,.. : 'i<;,1 '
:roo~~~
:_.-.t...~/-;
.......
.
.
,,.
::.,
s.;1.:s;
O~I
\
<
C"'
\
"'

\c.',

H.i"" j I

jl

,,

,..,/~~ /

"""'i1'' I ""'~! ..... ,


t~~{

llldtokin

ir<>A ote, lilri~or.t

The diagram below of a blast furnace has been incorrectly labelled.

Which one of the following statements is correct about Haber process?

The reaction does not go to completion.

28

Shertlet. a frozen fruit juice, feels cold when you placed it in your mouth. It is made from a
mixture of citric acid and sodium ~.ydrogencarllonate. The following reaction takes place in
your mouth.

i;I

11

Sodium citrate + carbon dioxide +water

1 volume of hydrogen reacts w ith 3 volumes of nitrogen to form 2 volumes of ammonia

~f

~I

Whicln energy profile diagram shows the reaction when sherbet is placed in your mouth?

Citric acid + Sodium hydrogencarbonate -

Increasing the temperature will increase the yield but operating cost is high.

Ptacsucu

l'fbv<tN ol <Hcii...

llt1onh

l'ro0f41 91 .. ..-..~

ti r..fb
...r/'I.

.., i

H20

A black solid and a blue liquid.

29

r
c

R.ict:anu

.. ~~ . \P'O<lll<H

f
. . ----'r
P1oy:ou el r-K".1c.n

Which one of following acts as the re,!!_~ent in the equation shown?

H'

Fe" + N02 + H20

A
N03-

Fe2' + NO) + 2H' -

A blue hquid only.

[Turn over

Copper(ll) oxide catalyses the decomposition of aqueous hydrogen peroxide to give oxygen
and water In a laboratory experiment, aqueous hydrogen peroxide was shaken with
copper(ll) OXJde in a test tube.

A light blue solid and a colourless liquid.

What did the tube contain when the reaction was complete?

4EXP/CHEM/507211/PRELl~l/2011

31

32

33

34

~lOO
tin

~-00

silver

magne!Uum

~-oo

-zinc

~"~

13
The diagram below show four pieces of metal strips of equal size tied together in dilute
hydrochloric acid . After five minutes, which beaker will contain the least amoun t of iron ions
in the solution?

35

Cotton
wool

Anhydrous
calcium
chloride

2
Did not rust

Rust after 1
week

Tap
water at
25C

Rust after 3 days

Sodium
chloride
solution

14
Kai Xiang set-up the four test tubes below to investigate the conditions for rusting of metals
to occur.

Oil
Boiled
tap
water at

2s c

Old not rust

1and 3

2and 3

1and 4

Which two expenments can be used to show that water is needed for iron to rust?

2 and 4

where n > 1 _Which one of the following

2,

Alkynes are saturated hydrocarbons.

Alkynes have the general formula C.H2n


statements is correct?

The boiling point decreases as the number of carbon atoms in the alkyne increases.

36

Jia Quan placed a sample of powdered metal M in aqueous lead(ll) nitrate. A grey residue
was obtained. The temperature of the reaction mixture rose and some M remain unchanged.

Alkynes decolourlse bromine water rapidly.

The relative molecular masses of successive members in the series differ by 12.

Which one of the following conclusions cannot be deduced from this information?
The reaction is exothermic.

A
M has the same valency as lead-

Petrol

Hydrogen

Density
Mass

Boiling point

Molecular formula

H,

CH3Coo- Na'

CH3CH2 0.Na

CH,COOH

Which one of the following substances could be X?

4EXP/CHEM/5072/11PREUM-l/2011

[Turn over

When a glass of wine is left standing in air for some bme. they become acidic. When the acid
Is removed and reacted with sodium, a substance X is formed_

During the polymerization of ethane to form poly(ethene), which of the following does not
change?

37

M was in excess.
M is more reactive than lead

Methane

Diesel

When ~ burns easily in air or in oxygen.

38

When 1t contains carbon and hydrogen only.

[Turn over

Which one of the following fuels does not produce any pollutants when they undergo
combustion reaction?

When it has the formula C.H,,,_,.

Under which condition can we identify a substance as an alkane?

When 11is generally unreactive_

B
0

4EXP/CHEM/5072/1/PRELIM1/2011

39

40

15
Compound W , which has the molecular formula,

CiH,COOC,H 7,

went through the following

Water was added to compound W which formed an acid X and an alcohol Y .

reactions:

A
C2HCOOH

C2HCOOH

CiHsOH

C3H70H

C3H70H

C2~

C2H

C3He

C3He

/H

(II)

I
H

[Turn over

0-H

'\

/0
~

r-0-H

c-6-1

0 ~
H-0/

(IV)
0 ,

c-c-c-c

'\."-

H-0/

Y was heated with pieces of broken porous p ot to form Z, which can decolourise aqueous
bromine.

B
C,H,COOH
C2HsOH

Deduce the identity of compounds X, Y and Z.

c
C3H1COOH

~-

H-C =c-c-H
H

I
H

'\

Which of the following compounds would undergo polymerisation on their own?

(I)

(II)

0 ,

c-c-c-N

'\."I

(I) and (II) only

(II) and (Ill) only

(1), (II) and (Ill) only

All of the above

H-0/

End of Paper

4EXP/CHEM/5072/tlPREllM-V201t

16

5072 CHEMISTRY (NEW PAPERS WITH SPA) 0 LEVEL (2008)

aluminium hydroxide

white

white

Colours of Some Common Metal Hydroxides

calcium hydroxide

light blue

red-brown

green

copper(tl) hydroxide

iron(ll) hydroxide

oron(lll) hydroXJde

white

--

lead(ll) hydroxide

white

zinc hydroXJde

4EXP/CHEMIS07211/PRELIM-l/2011

[Turn over

Secondary Four Express


PRELIMINARY EXAMINATION I
General Certificate of Education Ordinary Level

CHEMISTRY
PAPER 2

Wnung papers (3 sheets)

Additional matenals
A copy of PerlOdlc Table

1 hour 45 minutes

Class

-~

JULY 2011

5072/2

WESTWOOD SECONDARY SCHOOL

Candidate Name _ _ _ __ _ _ __ _ _ _ _ __ __

TIME
INSTRUCTIONS TO CANDIDATES

Candidate
Number

Marks

Wnte your name, class and candidate rumber in the spaces at the top of this page and on any
separate writing paper(s) used. Do not use staples, paper clips, highlighters, glue or
correction fluid.

Section

Section A
Answer all questions.
Write your answers in the spaces provided on the Question Paper.

Section B

B
Total

Answer three questions in the wrrting papers provided. The last


question is 1n the form either/or and only one of lhe alternatives
should be attempted.
At the end of the examination. fasten any separate answer paper
used securely to the question paper.

INFORMATION FOR CANDIDATES

[Turn over

You may use a pencil for any diagrams. graphs or rough working.
The number of marks is given in brackets [ ) at the end of each question or part question
Electronic calculators can be used in this paper. If your answer in Section A or B involves
mathematical calculations. show your working. Where necessary, round off numencal answers to 3
Significant figures.
A copy of Data Sheet is printed on page 16.

This question paper consists of 16 printed pages.


Sette< , Mrs Thlo-S1m Slaw Cheng

-I

Section A [SO marks]

Answer all questions in this section in the spaces provided.

Freezer X

-1 c

-sc

~ ~ Iii
2c

2c

-1

~~

This 1s what the tubes look like In Freezer Y.

Melting
point:

Freezer Y

-s c

Solid

Liquid

The temperature of the fndge is below ___ C and above ___ C.

[Turn over

The contents of the tubes are made of particles. The diagram below shows the
arrangement of particles in tube B.

[ 11

----------- - - - - - - - - - - ---- I 1 I

The temperature inside lhe freezer IS - 2 C. Which tube(s) will contain solid at this
temperature?

Melting
point:

Freezer X has three tubes t o determine if the temperature gets too high. Each tube contains
a solid that melts at a different temperature. The solid stays on top of the tube. When it melts.
it drops to the bottom.

(a)

(b)

(c)

EXPICHEMIS07212JPREUMl/2011

3
Given that particles in tube B sublimes at -1
tube B at oc.

._ ,~,..,

~ '

[1l

c. draw the arrangement of particles in

. _,

The diagram below was obtained from an analysis of naturally occurring magnesium.
. .- . .. ,,.-- _,.,.
H. ..

.
-e~tt-~~iv&~~~f
--,_, ,, ~--'"
;-;-,.~ -;. ~

',

-l---........
"--- ... ..

.. . {_~--- ' .,..,.- ~--~


.,...,
' ..
j l~- :- .."" ~ ' : ;:_.
:_. ~

:fr }1'Jj\;'+:
40
30
2Q

wl .J. l - , .
, I / . t_;-c_;\_
24

25 25
.....,. -

In the space below, represent one of the isotopes found in naturally occurring
magnesium. using the following representation: ~-Mg
[1I

,~

(a)

Using the diagram, calculate the relative atomic mass of a sample of naturally
occurring magnesium
[2I

[Turn over

Use an appropriate diagram to show the bonding and electron arrangement in


magnesium chloride, showing only the outermost electrons
( 2)

chloride.

A sample of naturally occurring magnesium is reacted with chlorine to form magnesium

(b)

(e)
(I)

4EXP/CHEM/5072/2JPRBJM-ll2011

(ii)

4
Suggest one difference that you would expect to see in the reaction of the
different isotopes of magnesium with chlorine.

----------------------------------------------------------------- [ 1 )

3C/2 + 6Na0H -

Describe a lest for chlorine gas.

5NaC/ + NaCIO, + 3H,O

-------------------- -------------------------------

-------------! 1 I

[Turn over

[2

Predict two observations that can be made by Priscilla if an aqueous solution of


bromine were used instead of chlorine gas.

--------------------------------- --------------- [ 2 )

Explain your answer in part (b) above.

Whal is the property of chlorine in the reaction when it forms sodium chlorateM?

-- I 1 I

Priscilla conducted an experiment to react chlorine gas with hot aqueous sodium hydroxide
to form sodium chloride and sodium chlorate(V). The following equation represents the
reaction:

(a)

(b)

(c)

(d )

4EXP/CHEMl50nl2/PREUM-l/2011

re!ativ~

Jeve.Js

',X/
,

---

__ / ......

fodconsumptkm

o.:drte$ of n~og~n anissioo

'>~._ ........._~

1'-,>~

,
,,.-\,../
i ' ..

, .;

~
/

. ______ [2 J

[Turn over

......_......................________ [ 2 I

Describe how oxides of nitrogen affect the environment.

Explain why as the air : fuel ratio increases, the level of oxides of nitrogen increases.

----------------------- [ 2 J

Explain why the carbon monoxide level decreases with increased air: fuel ratio.

air: i1,'l~I ratio

~-,,..,~_:ooox~ omissiOn -..:~:-:-:-.-.---

.\

The graph below shows the results obtained of air : fuel ratio and the fossil fuels (e.g. petrol
and diesel oil) consumption in road vehicles. The relative levels of carbon monoxide and
oxides of nitrogen emissions from the exhaust pipe of the vehicles are shown.

(a)

(b)

{c)

4EXP/CHEM/507212JPRELl~l/2011

::Jfl

l1t

o---------- -

6
The diagram below shows an electrolytic set-up for two solutions:

! .

r -- .._..______ -O
f

-"'~

-----,
~

Ccl!2

[ 2]

T-- -----~graphite

~ . hydroxlcle solutionr!
I
md<.el
\\.. ../ ;
...._.-/

d;:ute Cillcium

:~:~~ t~' /: ~ -- .. ---- ~~~-sulphate \


'--

.
Cch 1

Write the half equation for the reaction at the anode of Cell 1.

Identify the gas observed.

Suggest how this g:,is could have formed at the anode of Cell 2.

..... I 1 I

A colourless gas was observed at the graphite anode of Cell 2. This was followed by
the appearance of a white precipttate around the anode.

. ........... [ 1 J

What would happen to the concentration of the nickel(ll) sulfate solution in Cell 1?
Explain your answer.

Cell 1 has nickel electrodes dipping into aqueous nickel(ll) sulfate solution.
Cell 2 has graphite electrodes dipping into dilute calcium hydroxide solution.
{a)

{b)

(c)

{i)

(ii)

Identify the white precipitate formed .

------------------[ 1)
{iii)

Write the half equation to show the reaction which took place at the cathode in
Cell 2.

[Turn over

.........___________ [ 1 ]

........___________________ I 1 I
(iv)

4EXP/CHE M/5072121PRELl~l/2011

S!ep 1

heat

C r,o~ (s}

nitrogen gas &


water vapour

$lep4

Add di!vte HiSO, and

/1ydrogen peroxide

Step 2
fuse with N aOH in air.

( 2)

Na2 C;O,(aq)

Step3

~.dd dilute H,so,

Cool arid add water

Cr20i(aq}

cr,o,2-: - - - - - - -

State the role of hydrogen peroxide in step 4.

[ 1l

Describe the observations Benjamin would likely to see of the aqueous solution
containing Cr2 0/- ions in step 4.

---------------------------------------------------- ( 2

Explain in terms of changes in oxidation states, why (NH4 ),Cr20, is both oxidized and
reduce<! in step1.

Na2Cr04 : _ _ _ _ _ _ __

What is the oxidation state of chromium in Na,CrO, and Cr,o,2- respectively?

Cr+(aq}

(NH4)2Cr101(s}

Benjamin carried out a sequence of tests on compounds of chromium. The diagram below
shows the reaction scheme involving compounds of chromium.

(a)

(b)

(c)

(d)

[Turn over

---------------------------------------------------- [ 1 I

4EXPICHEM/507212/PRELJM-112011

N2 (g) + 3H2 (g)

t.H = +92 kJ

Hydrogen gas can react wrth nitrogen gas under suitable conditions to form ammonia
gas via Haber process. Give the optimum conditions and catalyst used for this process.

[ 2)

Is the above decomposition of ammonia an exothermic or endothermic reaction?


Explain your answer in terms of bond breaking and bond forming.

2NH3 (g) -

8
Ammonia gas is decomposed back to nitrogen and hydrogen gas according to the reaction . ._

(a)

(b)

Draw a suitable diagram illustrating the electron arrangement in a molecule of


ammonia. Show the full electronic structure.
( 2)

l
(c)

One of the uses of ammonia is in the manufacture of nitric acid. This is done by a twostage process.

- ------------------------- ( 2

(d)

4NH3 (9) + 502 (g)

4NO (g) + 6H20 (g)

Stage 1: Ammonia is converted to nitrogen(ll) oxide.

4NO (g) + 2H20 (I) + 302 (g) _ _ . 4HN03 (aq)

Stage 2: Nitrogen(ll) oxide is converted to nrtric acid.

(i)

[Turn over

[1l

During the reaction in stage 1, ammonia gas and oxygen gas are passed
through a powdered catalyst. Explain why the catalyst is used in the form of the
powder.

4EXP/CHEM/5072121PREUM-112011

(e)

9
(c)

Calculate the maximum mass of nitric acid whicl1 can be produced from 720 dm
of ammonia measured at room temperature and pressure.
[ 3)

(d)

(ii)

Ammonia and nitric acid can react to form ammonium nitrate which can be used as an
explosive. A terrorist was suspected of carrying a sample of ammonium nitrate in a
plane. Suggest how the police can conduct simple laboratory tests to show if the
sample indeed contains ammonium nitrate.

I2 I

Ethan al

Methanal

CH3CHO

HCHO

Formula of aldehyde

HCOOH

Form ula of compound


formed when aldehyde is
oxidised

Amira.h was given a task to research on the homologous series of aldehydes. The table
below shows some information about the first five members of the aldehyde homologous
series.

Propanal

C 3 H7 COOH

Aldehyde

C2 H5CHO

Number of
carbon (n)

C 4 H9 COOH

CH,COOH

C 3 H,CHO
Pentanal

C, H,CHO

Suggest the general formula for aldehydes.


----------

I1I

[Turn over

------------

-------------------------- - - - ---- ---- --- ------I 1 I

Write the name of the aldehyde with the formula C 3H7CHO.

Use the information in the table to answer the following questions.


(a)

(b)

---------- -----------------4EXP/CHEMl5072/21PREUM-U2011

10
Draw the full structural formula of the compound formed, when propanal is oxidised.

[1]

[Turn over

Aldehydes can be reduced to form alcohols. When propanal is reduced. propanol is


formed. Draw the st ructure of the compound formed when propane! reacts with
methanoic acid .
[ 1]

4EXP/ CHEM/507212/PRELIMl/2011

11
S~tion B [30 mark$]
Answer all t hree questions.
The last question is In the form either/or and only one of the alternatives should be attempted.
Question 9: Data-based question
9

Bromine Is an important industrial chemical used for making name relardanls. Bromine is
extracted from sea water. which is naturally rich in bromine ions. Chlorine can be bubbled
through sea waler to convert bromide ions into bromine.
Dessy carried out an experiment in a laboralory to investigate the reaction between bromide
ions and chlonne gas.
She bubbled Chlorine gas through dilute. aqueous potassium bromide for 6 minutes. She
took samples of the reaction every 30s and measured the colour intens~y of each sample
using a calorimeter.

.:

= :
:esled

$a!l'",Ple being

abeortmnce

A calonmeter measures light absorbed when the light passes through a coloured solution.

The diagram shows how a calorirreter works.

-0,~g!lt
~rec

telJdll\Q

The darker the colour of the solution, the more is light absorbed and the higher the
absorbance reading on the calorimeter.
Aqueous bromine absorbs more light than aqueous chlorine or potassium bromide.

I
2
ltne!mnde:$

[Turn over

Explain why aqueous bromine has a high absorbance value but aqueous chlorine has
a low absorbance value.
(1J

oo +-~~~~~~~~~~~~~~~~~

01;

02

~:j

*'"'"""'O.Si

::~

0.6

::l

Dessy plotted her results on a graph.

(a)

4EXPICHEMl5072121PRELIM-1/2011

(b}

(c)

12
Describe the rate of reaction in the first minute of the displacement.

Explain what causes the absorbance to Change in this way.

(i}
(ii)

Describe and explain the changes in speed of reaction shown by the graph. [ 4 )

[11

I1I

(iii}

Time taken for reaction to


finish I mlns
5.00

0.8

0.4

Absorbance reading at
end of reaction
0.8

Dessy carried out four more experiments to determine the time taken for each reaction
to finish. She used lhe same volume of potassium bromide solution each time. She
recorded the lime taken and the absorbance reading at lhe end of each reaction in a
table.

6.00

Which expenment uses potassium bromide of a lower concentration than in


experiment 1.
[ 1J

Experiment

2.50

2.50

(i)

If the starting concentration ol reactants is the same, how does the time taken
for the reaction to finish relate to the rate of the reaction.
(1)

0.9

(ii)

Which two experiments show the same concentrations of reactants being used
but at different temperatures?
[ 1)

[Turn over

[Total: 10 M)

(Iii)

4EXP/CHE M/507212/PR~UMl/401 1

10
(a)

(b)

Gu

,l\.g

13

ILJ

Ca

lr

1:
i

M\,I

t~

CutNO,h \aq)

H 1---1

lI

\/
Mg(NO;i, {aq)

Ca{NO~n {aq;

I J' I
~

h
i

The following experiment was set up by Marcus to investigate on the reac1iv1ty of


metals.

Wir1

1\1(,:01

AgN03(aq)

[2 ]

In which beaker was there no reaction? Explain your answer.

[2)

(i)
Descnbe what M arcus should observe in beaker 0 .

[ 2]

(Ii)
Construct a chemical equation, with state symbols, for beaker A.

(Iv)

(Iii)

The experiment was subsequently repeated . All the metal strips in the four
beakers were replaced with zinc strips. Which beakers would Marcus expect to
see a reaction?
[ 1I

Using only two of the following metals:

Magnesium, Zmc or Aluminium


Draw a diagram of a simple cell that can be set up to produce the largest amount of
electricity.

[Turn over

[Total: 10 M)

Indicate the sotutlon(s) and electr odes that you would use and the direction of flow
of electrons in your diagram.
[ 3J

4EXPICHEM/5072121PREUM-l/2011

EITHER
11

14

One mole of hydrocarbon A is cracked to give one mole of propane and two moles of
hydrocarbon B only.
Hydrocarbon B can be :onverted to an alcohol C by reacting hydrocarbon B with steam and
phosphoric(V) acid.

(II)

(i)

Explain why cracking 1s an important process in the petrochemical industry. [ 1 ]

Construct a chemical equation for the cracking of hydrocarbon A to propane and


hydrocarton B.
[1)

Determine the molecular formula of alcohol C .

Gas

Liquid

Physical State

- 2220

-1367

Ent halphy change of


combustion I kJ per mole

I1I

Liquified Petroleum Gas (LPG) and ethanol can be used as fuels for vehicles instead of
petrol. LPG contains mainly propane.

(iii)

[3]

Alcohol C w as analyzed and It was determined to contain 60.0% carbon, 13.3% hydrogen
and 26.7% of another element. The molecular mass of alcohol C is 60.
(a)

(b)

Boiling

The table below shows some informalton about propane and ethanol.
Fuel

Polnt/C

78

Give an advantage of using ethanol rather than propane as a fuel for motor
vehicles .
[1)

-42
(i}

Which fuel gives out more energy per kg? Use data from the table to support your
answer.
[ 1]

Ethanol

(ii)

In a car engine, a spark plug ignites a mix1ure of air and ethanol. The spark is
needed because combustion of ethanol needs activation energy.

Propane

(iii)

10 M}

[Turn over

[Total:

Draw an energy profile diagram for the combustion of ethanol, indicating the
activation energy and onthal phy change for the reaction. Label the reactants
and products.
[3]

4EXPICHEM/S07212/PRELIM-li2011

OR
11

(a)

(b)

..

I
,

15

.H
,

.,

.,

,0

:1

!
!

!.
'

_Jn

i~
1

c- N--c---o --c-c--O-l-

I H ,_.
\

}>=C
...

'H

~ /.!lj
I c . -~y ~",,,,

I1,'l'

One of the constituents of Lycra, a fabric used in swimwear and sports clothing is
polyurethane with the following formula:

'

.
j

!!

I
I
--L...c-N---"c
1

ri
I. u
0

Assuming that this polymer is made by the loss of water, draw the full stnuctural
formulae of the monomers found in this polyurethane.
[2]

polyuretlume

(i)

Draw the linkage that is found in this particular polyurethane.

(ii)

Name and d raw the structure of another synthetic polymer that uses a similar
linkage, showing two repeating units.
[2]

[1 ]

(iii)

[Turn over

[Total: 10 M)

Brass is an alloy of copper and zinc. Using brass as a starting material, describe how
you would obtain a sample of dry and pure zinc(ll) carbonate solid.
[5]

End of Paper

4EXPICHEM/5072121PREUM-1/2011

16

5072 CHEMISTRY (NEW PAPERS WITH SPA) 0 LEVEL (2008)

aluminium hydroxide

white

white

Colours of Some Common Metal Hydroxides

calcium hydroxide

iron(!ll) hydroxide

iron(ll) hydroxide

Whrte

red-brown

green

light blue

lead(ll) hydroxide

white

copper(ll) hydroxide

zinc hydroxide

4EXPICHEMIS07Z/21PRELIM-<12011

[Turn over

Secondary Four Express


PRELIMINARY EXAMINATION I
General Certificate of Education Ordinary Level

PAPER2

CHEMISTRY

Additional materials:

Writing papers (3 sheets)

A copy of Periodic Table

1 hOu r 45 minutes

Class

JULY 2011

5072/2

WESTWOOD SECONDARY SCHOOL

Candidate Name ___________________

TIME
INSTRUCTIONS TO CANDIDATES

Candidate
Number

Marks

Write your name, class and candidate number in the spaces at the top of this page and on any
separate writing paper(s) used. Do not use staples, paper clips, highlighters, glue or
correction fluld.

Section

Section A
Answer all questions.
Write your answers in the spaces provided on the Question Paper.
Section B

Total

Answer three questions in the wrtting papers provided. The last


question is in the form either/or and only one of the alternatives
should be attempted.
At the end of the examination, fasten any separate answer paper
used securely to the question paper.

INFORMATION FOR CANDIDATES


You may use a pencil for any diagrams, graphs or rough working.
The number of marks is given in brackets [ ] at the end of each question or part question.

[Turn over

Electronic calculators can be used in this paper. If your answer in Section A or B involves
mathematical calculations , show your worKing. Where necessary, round off numerical answers to 3
significant figures.
A copy of Data Sheet is printed on page 16.

This question paper consists of 16 printed pages.


Setter : Mrs Thio-Sim Siaw Cheng

Section A [50 marks]

Answer all questions in this section in the spaces provided.

2 c

-1 c

~ ~

-1 C

.4

-5 C

-5C

Tubes A and B - Both to be correct to award 1m

2C

~~

This is what the tubes looK like in Freezer Y.

Melting
point:

Solid

[Turn over

The contents of the tubes are made of particles. The diagram below shows the
arrangement of particles in tube B.

The temperature of the fridge is below .:1.C and above .:2_C. (both must be correct to
award 1m

Liquid

The temperature inside the freezer is -2 C. Which tube(s) will contain solid at this
temperature?

Melting
point:

Freezer X has three tubes to detennine if the temperature gets too high. Each tube contains
a solid that melts at a different temperature. The solid stays on top of the tube. When it melts,
it drops to the bottom.

(a)

(bl

(c)

4EXPICHEM1507212/PREUM-1/201 1

0 0
0

Given that particles in tube B subijmes at -1 "C, draw the arrangement of particles in
[ 1]
tube B at o "C.

0
Draw arrangement of gaseous particles

or

;:Mg

or :~Mg

i Mg

(1)
(1)

(1 I

Use an appropriate diagram to show the bonding and electron arrangement in


[ 2)
magnesium chloride. showing only the outennost electrons.

A sample of naturally occurring magnesium is reacted with chlorine to form m<19nesh,1m


chloride.

= 24.3

Relative atomic mass of Mg= (0.8x24) + (0.1x25) + (0.1x26)

Using the diagram. calculate the relative atomic mass of a sampl& of naturally
occurring magnesium
[2)

:;Mg

magnesium, using the following representation:

In the space below, represent one of the isotopes found in naturally occurring

The diagram below was obtained from an analysis of naturally occurring magnesium.

(a)

(b)

{c)

(i)

[Turn over

(1) correct number of electrons in each ion and correct transftr of electrons
(1) correct charges, brackets

4EXP/CHEM/507212JPREUM-l/2011

(II)

Suggest one difference that. you would expect to see in the reaction of the
different isotopes of magnesium with chlorine.

Isotopes with higher number of neutrons will react slower with chlorine
compared to Isotopes with lower number of neutrons [1)

Priscilla conducted an experiment to react chlorine gas with hot aqueous sodium hydroxide
to form sodium chloride and sodium chlorate(V}. The following equation represents the
reaction:

Describe a test for chlorine gas


Test chlorine gas with moist bl ue litmus paper['!.)
Litmus paper will turn red t hen bleached. ('!.l

SNaC/ + NaCIO, + 3H20

(a)

What is the property of chlorine in the reaction when it forms sodium chlorate(V}?

3C'2 + 6NaOH -

(bl

Explain your ansv.er 1n part (b) above.

Predict two observations that can be made by Priscilla if an aqueous solution of


bromine were used instead of chlonne gas.

Chlorine increases In oxidation state (11 from 0 to +S [1).

Chlorine has the property of a reducing agent/ Chlorine is oxidised [11


(c)

(d)

Br2 + 2NaOH -

NaBr+ NaBrO +H 20

[Turn over

Aqueous bromine turns from reddish-brown to colourless/ pale vellow. !11


White solid formed 111

4 EXP /CHEW ei07212/PREUM-l/2011

rulatlve levels

:'
/

_L - -

t....~ coosumpllon

-:-:-::7:---

Ol(lde-1 of mt09en troisiai

-.

~~'-._

'>' . ,_

\--'~~
._,

~
,
I
l
I

~:fl.lelrtio

More complete combustion fuels occur resulting in more heat


generated. increasing the temperature of the combustion engine 11.
Thus at high temperature, nitrogen and oxygen from the al r In
combustion engine will react to form more oxides of nitrogen 11.

Explain why as the air : fuel ratio increases, the level of oxides of nitrogen increases.

As the alr:fuel ratio Increases, there are more oxygen avallable 11


for the fossil fuels to burn more completely to form water and
carbon dioxide (11 and hence less pollutants, carboo monoxide
produced.

Explain why the carbon monoxide level decreases with increased air : fuel ratt0

eatbc:i nor-oxid!:i {;rn~lOO --

l
1

\
"

5
The gr aph below shows the results obtained of air : fuel ratio and the fossil fuel s (e.g. petrol
and diesel oil) consumption in road vehicles. The relative levels of carbon monoxide and
oxides of nitrogen emissions from the exhaust pipe of the vehicles are shown.

(a)

{b)

{c)

Describe how oxides of nitrogen affect the environment.


Oxicl_e.s of nitroqen will dlssolve in rain water to form acid rain. Acid rain
corrodes metal structures/ chang_e_~.ID'f of soil and water hence affect agricultural
or aquatic lives (11

[Turn over

Sunllght acting on oxides of nitrogen in photochemical reaction results in


formation of photochemlcal smog which irritates eyes and lungs. 111

4EXP/CHEM/5072/21PREUM-l/2011

I
I :-i ;.........:

:::::'.

11i~l-,el

CeH2

'

.:_=-oraoh"

-:::::

___.../

\\,,\'-.../, l!

--.-------.. .ti ~-----j

calci~m ~ f~_.I

(}-----..----..~----.

The diagram below shows an electrolytic set-up for two solutions:

t.._} .,

1 1

dilute
r!
hyctroxid~
solution!~.

r--..-----.. --0
ij

aqu~.:.us

L....... _,l - -

L~.~

nlckei'li)
1
'
sulpn;te ... t...
\...
~I
C~I!

What would happen to the concentration of the nickel{ll) sulfate solution in Cell 1?
Explain your answer.

Cell 1 has nickel electrodes d1pp1ng into aqueous nickel(ll) sulfate solution.
Cell 2 has graphite electrodes dipping into dilute calcium hydroxide solution.
(a)

Write the half equation for the reaction at the anode of Cell 1.

Concentration of nic kel!lll sulfate will remain unchanged [1) because there is no
net loss of Ni2 ions. At the anode of Cell 1, nickel dissociates to form Ni2 ions
where as at the cathode. Nl2 Ions are discharged to form a layer of nickel on the
cathode. (1)
(b)

A colourless gas was observed at the graphite anode of Cell 2. This was followed by
the appearance of a white precipitate around the anode.

H, !glf11

[Turn over

Wnte the half equation to show the reaction which took place at the cathode in
Cell 2.

Calcium carbonate 111

Identify the white precipitate fonned.

ill

OH' Ions are d ischarged forming water and oxygen. As the reaction is
exothermic. oxygen wlll react with carbon from the electrodes to form CO,

Suggest how this gas could have formed at the anode of Cell 2.

Carbon dioxide (1 I

Identify the gas observed.

Ni Isl - + Ni1 !aql + 2e 111


(c)

(i)

(ii)

(Ill)

(iv)

2W !aal +2e -

4EXPICHEM/507212/PREUM~/2011

Sl"P 1

re.at

Cr203(S)

nitrogen gas &


waler vap<>ur

Sli?p4
Add dir.ite H,so, and

l1ydro11m f!E:roxlcle

Step 2

I 21

+ Ufl:lCrO, (aq)

fuse with NaOH in air.

Slep3

fuct d!Me 1-1,so,

Cool a nd 00-J wa ter

Cr20/- : +6

State the role of hydrogen peroxide in step 4_

Orange acidified Cr1o,2- solution turns green. 111

[Turn over

Describe the observations Benjamin would likely to see of the aqueous solution
containing Cr2 0,2- ions in step 4.

il'!!:!._4 ),Cr2 0 7 Is reduced to cr._03_ as the oxidation state of chromium decreases


from +6 In !NH,)iCr10z to +3 in Cr20, [1)

{NH 4 ),Cr,07 is o>Cidlsed to N, as the oxidation state of nitrogen increases from -3


in (NH,J,Cr,o, too in N, 111

Explain in terms of changes in oxidation states, why (NH,)zCr2 0, is both oxidized and
reduced in step1 _

Na 2Cr04 : +6

What is the oxidation state of chromium in Na 2Cr04 and Cr20,2- respectively?

,/

Cr3"{aq} _,.,_ _ _ _ _ __ Cr20/-(aq)

{NH~)2Cra01(s)

Benjamin carried out a sequence of tests on compounds of chromium_ The diagram below
shows the reaction scheme Involving compounds of chromium.

(a)

(b)

(c)

{d)
Reducing agent [1)

4 EXP/CHEM/5072/2/PRELIM-112011

N2 (g) + 3H2 (g)

t.H

=+92 kJ

Is the above decomposition of ammonia an exothermic or endothennic reaction?


Explain your answer ill terms of bond breaking and bond forming.

2NH 3 (g) -

Ammonia gas is decomposed back to nitrogen and hydrogen gas according to the reaction.

(a)

Endothermic [1]. More energy is absorbed in the breaking of N-H bonds in


ammonia than energy released In making HH and N=N bonds [11
(b)

Hydrogen gas can react with nitrogen gas under suitable conditions to form ammonia
gas via Haber process. Give the optimum conditions and catalyst used for this process.

c and 200 aim 111

Draw a suitable diagram illustrating the electron arrangement in a molecule of


ammonia. Show the full electronic structure.
[ 2]

One of the uses of ammonia is in the manufacture of nitric acid. This is done by a twostage process.

(1) correct number of atoms for each element

[1] full eh.ictronh;: ~tructure with cQrrect number of electrons shared

Iron catalyst (1), 450


(c)

(d)

4NH 3 (g) + 502 (g)

4NO (g) + 6H2 0 (g)

Stage 1: Ammonia is converted to nitrogen(ll) oxide.

[Turn over

Calculate the maximum mass of nitric acid which can be produced from 720 dm3
of ammonia measured at room temperature and pressure.
{3]

Greater surface area of catalyst exposed, increases rate of reaction (1)

through a powdered catalyst. Explain why the catalyst is used in the fonn of the
powder.

During the reaction in stage 1, ammonia gas and oxygen gas are passed

4NO (g) + 2H20 (I)+ 302 (g) - - - + 4HN03 (aq)

Stage 2: Nitrogen(ll) oxide is converted to nitric acid.

(i)

(ii)

No of mole of NH3 = 720/24 = 30 moles [1]


Mole ratio of NH3 : HN03 = 1: 1
No of motes of HN03 30 moles [1]
Mass of nitric acid= 30 x (1+14+48) = 1890 g (1]

4EXP/CHE M/507212/PRELIM l/2011

(e)

9
Ammonia and nitric acid can react to form ammonium nitrate which can be used as an
explosive. A terrorist was suspected of carrying a sample of ammonium nitrate in a
plane. Suggest how the police can conduct simple laboratory tests to show if the
sample indeed contains ammonium nitrate.
Add aqueous sodium hydroxide to the sample solution. Warm the mixture gently.
Gas evolved turns moist red litmus paper blue. Gas is ammonia. NH; Jons are
present 1)
Add aqueous sodium hydroxide and al uminium metal to the sample solution.
Warm the mixture gently. Gas evolved turns moist red litmus blue. Gas is
ammonia. NO," ions present. (1]
Amirah was given a task to research on the homologous series of aldehydes. The table
below shows some information about the first five members of the aldehyde homologous
series.

CH3CHO

HCHO

Fonnula of aldehyde

HCOOH

Formula of compound
fonned when aldehyde is
oxidised

Aldehyde

Ethanal

Methanal

Number of
carbon (n)

c.H9CHO

C 3H 7CHO

C4H9COOH

C 3H 7COOH

CH 3COOH

2
C2HsCHO

Pentanal

Propanal

3
5

l1J

Draw the full structural formula of the compound fonned, when propanal is oxidised.

CJ:l...:tt 1CHO where n= 0,1,2... OR Cr?-1!:!z.,.1CHO where n=0.1.2 .... [11

Suggest the general fonnula for atdehydes.

Butanal [11

Write the name of the aldehyde with the formula C,H,CHO.

Use the infonnation in the table to answer the following questions.


(a)

(b)

(c)

(d)

[Turn over

Aldehydes can be reduced to form alcohols. When propanal is reduced, propanol is


formed. Draw the structure of the compound fonned when propanol reacts with
methanoic acid.
[1J

4EXPIC HE Ml5072121PREUMU2011

10

Section B [30 marks)


Answer all three questions.
The last question is in the form either/or and only one of the alternatives should be attempted.
Question 9: Data-based question

Bromine is an important industrial chemical used for making flame retardants. Bromine is
extracted from sea water, which is naturally rich in bromine ions. Chlorine can be bubbled
through sea water to convert bromide ions into bromine.

Dessy carried out an experiment in a laboratory to investigate the reaction between bromide
ions and chlorine gas.

She bubbled chlorine gas through dilute, aqueous potassium bromide for 6 minutes. She
took samples of the reaction every 30s and measured the colour intensity of each sample
using a calorimeter.

-- !

<Jet~c<

readi">l

;:ibsortiance

A calorimeter measures light absorbed when the light passes through a coloured solution.

II

}mi

The diagram shows how a calorimeter works.

-.o:light

soorc..

~8mple being
testoo

The darker the colour of the solution, the more is light absorbed and the higher the
absorbance reading on the calorimeter.

tme-/m!ntites

[Turn over

Aqueous bromine absorbs more light than aqueous chlorine or potassium bromide.

Oessy plotted her results on a graph.

101
o.~l

OB-.

(t,6-i

011

oi

- o.s-l

().21!

U.3-i

0.11

'

oo~.'--~~~~~~~~~~-.-~~-.~~-.

4EXPICHEIW5072121PRELIMll201 I

(a)

(b)

(c)

11
Explain why aqueous bromine has a high absorbance value but aqueous chlorine has
a low absorbance value.
[1]

Rate of reaction Is the highest in the first minute [1]

Desclibe the rate of reaction in the first minute of the displacement


[ 1]

Bromine is red/brown so is more strongly coloured I is a darker colour


than chlorine which is pale green/yellow chlorine so is paler/ less strongly
coloured [11
(i)

(ii)

Explain what causes the absorbance to change in this way.


[1J
more bromine is made I idea that solution is becoming more
concentrated with bromine [1J

(iii ) Describe and explain the changes in speed of reaction shown by the graph. [ 4]

reaction slows (1)


as concentration of bromide decreases (1 I
reaction stops I absorbance no longer increasesJ.!l
because bromide ions have all been used up (1 I

Experiment

6.00

5.00

Time taken for reaction to


flnish/mins

0.9

0.8

0.4

0.8

Absorbance reading at
end of reaction

Dessy carried out four more experiments to determine the time taken for each reaction
to finish. She used the same volume of potassium bromide solution each time. She
recorded the time taken and the absorbance reading at the end of each reaction in a
table.

2
2.50

2.50

(II)

(i)

If the starting concentration of reactants is the same, how does the time taken
for the reaction to f inish relate to the rate of the react.ion.
[1]
Shorter time, faster rate 111

Which experiment uses potassium bromide of a lower concentration than in


experiment 1.
[1 ]
Experiment 2

3
4

(Ill)

(Turn over

[Total: 10 M)

Which two experiments show the same concentrations of reactants being used
but at different temperatures?
[ 1I
Expt 1 and 3 111

4EXP/CHEM/50nl21PREUM-l/2011

10
(a)

(b)

Cu

Ag

u,

'I

12

Ca

LL--)

1 I

irw dW
ru
;

r--] ,

Mg

II,,

Cu(NC}c,i, (aq)

~I

The following experiment.was set up by Marcus to investigate on the reactMty of


metals.

Mg(NO")' (<><:ii

C9(NO,}L (aq)

Cu(s) + Ag(NO,j, (aq) Cu(N0 3 )2{aq) +Ag (s)


[1) correct formula for reactants and products
[1] correct state symbols

Construct a chemical equation. with state symbols, for beaker A

Ma strips dissolves into the solution/ becomes smaller 111

Blue solution turns colourless [1]

Describe what Marcus should observe in beaker 0 .

[ 2]

[ 2]

[2]

Beaker B [1) because silver is less reactive than calcium ions in calcium
nitrate solution hence unable to displace calcium ions. [1)

In which beaker was there no reaction? Explain your answer.

,;gNO~ (aQ)

' - /'

l-Ur--r

~l

strip

Me~a1

(i)

(ii)

(iii)

(iv)

The experiment was subsequently repeated. All the metal strips in the four
beakers were replaced with zinc strips. Whieh beakers would Marcus expect to
see a reaction?
[ 1]
Beakers A and O (1)

Magnesium, Zinc or Aluminium

Using only two of the following metals:

Draw a diagram of a simple cell that can be set up to produce the largest amount of
electricity.

[Turn over

[Total: 10 M]

Indicate the solution(s) (1] and electrodes [1] that you would use and the direction
of flow of electrons [1] in your diagram.
[ 3]

4EXP/CHEM/507212/PRELIM-1/2011

EITHER
11

13

One mole of hydrocarbon A is cracked to give one mole of propane and two moles of
hydrocarbon B only.
Hydrocarbon B can be converted to an alcohol C by reacting hydrocarbon B with steam and
phosphoric(V) acid .

c
13.3

1.66875

26.7

(iii)

0"

14

In a car engine, a spark plug ignites a mixture of air and ethanol. The spark 1s
needed because combustion of ethanol needs activation energy.

C=C

H,

'

,,

.,.

'

..

~n

' - c - - oI c-N-c--o-c
,.,
I
!
j

'H

c,

\\ - //
/C

f~

Assuming that this polymer Is made by the loss of water, draw the f ull structural
formulae of the monomers found in this polyurethane.
[ 2]

p1J!JUrctltane
(i)

[Turn over

[1l

Draw the linkage that is found in this particular polyurethane.

4 EXPICHE Ml507212JPRELIM-l/2011

(II)

j ::

! II

- I c---N-c,,

One of the constituents of Lycra. a fabric used in swimwear and sports clothing is
polyurethane with the following formula:

[Total: 10 Mj
OR
(a)

Draw an energy profile diagram for the combustion of ethanol, indicating the
activation energy and enthalphy change for the reaction. Label the reactants
and products.
[3]
(1) axes and labolll n9
(1) E. and CIH

Determine the molecular formula of alcohol C.

60.0

13.3
1

Enthalphy change of
combustion I kJ per mole

11

Alcohol C was analyzed and it was determined to contain 60.0% carbon, 13.3% hydrogen
and 26.7% of another element. The molecular mass of alcohol C is 60.
(i)

% by mass

5
8

Element

No of moles

as Mr is 60 (1)

[1)

Simpl est ratio [ 1)


Empirical formula=

C3 H10

=C,H, O

(Ii)

Construct a chemical equation for the cracking of hydrocarbon A to propane and


hydrocarbon B.
[1]
C9 H20 2C 3 H, + C3 H 0 (1)

Molec ular formula

(iii)

Physical State

-1367

Boiling

Liquid

I1I

Liquified Petroleum Gas (LPG) and ethanol can be used as fuels for vehicles Instead of
petrol. LPG contains mainly propane.

Explain why cracking is an important process in the petrochemical Industry. [ 1 I


To convert long chain hydrocarbon into usefu l. short chain hydrocarbo ns
t hat are high in demaod_l 1l

[3]

(a)

(b)

Fuel

78

-2220

Poi nt/ c

Gas

T he table below shows some information about propane and ethanol.

I
Ethanol

-42

(I)

Give an advantage of using ethanol rather than propane as a fuel for motor
vehicles.
[1I
Ethanol has a lower percentage of carbon 152.2"6>1 than pro pane (81 .1%1.
thus it burns more c leanly !11

Propane

(II)

=-

4 EXPICHEMl5072121PRELJM-l/2011

[Turn over

WhlCh fuel gives out more energy per kg? Use data from the lable to support your
answer.
[ 1]
0.046 kg of ethan ol produces -1367 kJ
1 kg of ethanol produces= 136710 .046
29717 kJ (1)
0.044 kg of propane produces -2220 kJ
1 kg of propane produces = 22201 0.044 = -50455 kJ (1)
Propane produces more energy per kg

(b)

{Ill)

15
Name and draw the structure of another synthetic polymer that uses a similar
linkage. showing two repeating units.
[2)

Brass is an alloy of copper and zinc. Using brass as a starting material . describe how
you would obtain a sample of dry and pure zinc(ll) carbonate solid.
[ 5)

[1)

17

11
12
13
14
15
16

No

A
A

c
c

A
A

Answer

18
19 '
20

No

21
22
23
24
25
26
27 '
28
29
30

Answer

38

31
32
33
34
35
36
37-

No

B
B
B

c
c

B
A

39
40

WESTWOOD SECONDARY SCHOOL


PRELIM EXAMINATION 2011 - SEC 4 EXPRESS
5072 CHEMISTRY ANSWER SCHEME

End of Paper

Answer
D
B

c
D

c
B
c

(Turn over

[Total: 10 M)

Add HCllH,SO, to brass. Zinc will react with the acid and copp.r wlll nol (1)
Fiiter the mixture to remove copper (1)
The filtrate formed ia ZnCh IZnSO, [1]
Add aqueous Na,co, to the filtrate obtain Insoluble znco. (1)
Filter to get the Insoluble ZnC01 and dry the insol uble ZnC03 between filter paper

Answer
B
B

- - cB

Paper 1:
No

1
2
3
4
5
6
- 7
8
9
_!Q_ _

4EXPICHEM/507212!PREUM-l/2011

Potrebbero piacerti anche